Endocrinology
Endocrinology
A 72-year-old man has had fatigue and loss of interest in daily activities over the past 4 months. He sleeps poorly and has had a
4.5 kg (10-lb) weight gain during this period. He states that he has probably lived long enough. His blood pressure is 110 /78 mm
Hg, and pulse is 50/min. Examination shows a slow return of deep tendon reflexes. Measurement of which of the following serum
levels is the most appropriate next step in management?
A) Parathyroid hormone
B) cosyntropin stimulation test
C) Glucose
D) Aldosterone to renin ratio
E) Thyroid-stimulating hormone
Explanation
Answer Description
Primary hyperparathyroidism associated with finding of hypercalcemia and present with fatigue, weakness, nephrolithiasis, and
fractures.
Patients typically develop gradual fatigue, weakness, borderline hypotension, and electrolyte abnormalities.
Summarized Points:
Hypothyroidism may mimic depression (lack of interest, poor sleep) that’s why always first exclude hypothyroidism by measuring
TSH (Thyroid Stimulating Hormone) level. Classically, Hypothyroidism symptoms are: Fatigue, cold intolerance, constipation,
weight gain, hair loss.
More Detail
Patient with symptoms (loss of interest, fatigue, sleeplessness, weight gain, bradycardia, slow deep tendon reflex) are
suggestive of hypothyroidism. Best initial test is measuring TSH (Thyroid-stimulating hormone).
References
Garber JR, Cobin RH, Gharib H, et al. Clinical practice guidelines for hypothyroidism in adults: cosponsored by the American
Association of Clinical Endocrinologist and the American Thyroid Association. Thyroid. 2012;22(12):1200-1235. Objective
number 118-1 (Weight gain/Obesity) of Objectives for the Qualifying Examination, 3rd edition.
Question #2
A 73-year-old man comes to the physician due to fatigue and lethargy for several weeks. He has non-ischemic cardiomyopathy
with a left ventricular ejection fraction of 25%, an automated implantable cardioverter-defibrillator, and a history of ventricular
tachycardia that has been controlled with amiodarone over the past year. The neck examination is unremarkable. Laboratory
evaluation shows TSH of 22 mIU/L (0.4-5.0) and low T3/T4 levels. Which of the following is the best next step in the
management of this patient?
Answer Description
Anti-thyroid peroxidase antibodies are present in patients with chronic autoimmune (Hashimoto's) thyroiditis. This test would be
helpful for evaluating patients with subclinical hypothyroidism (TSH <10 µU/mL) but would not alter the management of this
patient.
In this patient, amiodarone should be continued to prevent recurrent ventricular tachycardia, and hypothyroidism can be easily
treated with levothyroxine replacement treatment.
Thyroid ultrasound with color Doppler is a very useful tool for differentiating type I (increased vascularity) from type II (absent
vascularity) amiodarone-induced thyrotoxicosis. However, it would not be helpful in the management of amiodarone-induced
hypothyroidism.
Summarized Points:
Long-term amiodarone use can cause thyroid dysfunction (hypothyroidism or hyperthyroidism). Patients with overt
hypothyroidism may continue amiodarone but should initiate thyroid replacement therapy.
More Detail
This patient's clinical presentation — fatigue, lethargy, elevated TSH, and low free T4 — is consistent with amiodarone-induced
hypothyroidism. Long-term amiodarone use can cause thyroid dysfunction (hypothyroidism or hyperthyroidism) in 5%-30% of
patients. The risk of thyroid dysfunction depends on the patient's underlying thyroid status and dietary iodine intake.
Amiodarone-induced hypothyroidism is more likely than hyperthyroidism in iodine-sufficient areas and in patients with underlying
autoimmune thyroid disease. In contrast, hyperthyroidism is more common in iodine-deficient areas and in patients with
multinodular goiter or latent Graves' disease.
This patient has severe left ventricular (LV) systolic dysfunction and history of nonsustained ventricular tachycardia
(VT); treatment with amiodarone to prevent recurrent VT and automated implantable cardioverter-defibrillator placement is
indicated. Amiodarone should be continued in this patient, and hypothyroidism can be easily corrected by thyroid replacement
therapy with levothyroxine.
References
Objective number 63(Neck Mass, Goiter, Thyroid Disease) of Objectives for the Qualifying Examination, 3rd edition. Bartalena L,
Bogazzi F, Chiovato L, et al. 2018 European Thyroid Association (ETA) Guidelines for the Management of Amiodarone-
Associated Thyroid Dysfunction. Eur Thyroid J 2018; 7:55.
Question #3
A 23-year-old man with no past medical history comes to the physician due to breast enlargement. The patient is 6' 2" (188 cm)
tall and weighs 85 kg (187 lb). He has mild bilateral gynecomastia along with long extremities, sparse facial hair, and small, firm
testes. Serum LH and FSH are elevated and testosterone is low. Serum prolactin and TSH are normal. Which of the following is
most likely to establish the diagnosis in this patient?
A) 17-hydroxyprogesterone level
B) Genotyping for fibrillin-1 mutation
C) Karyotype analysis
D) MRI of the brain
E) Testicular biopsy
Explanation
Answer Description
Choice A is not correct:
17-hydroxyprogesterone is used primarily as a screening test for congenital adrenal hyperplasia (CAH). Classic CAH presents in
infancy with adrenal insufficiency, virilization, and ambiguous genitalia.
Marfan syndrome is an autosomal dominant disorder of the fibrillin-1 gene. Marfan syndrome may present with a tall body
habitus but is also characterized by joint hypermobility, arachnodactyly, pectus excavatum, and scoliosis/kyphosis. Gonadotropin
and testosterone levels are normal.
Pituitary tumors, such as prolactinoma, may cause hypogonadotropic hypogonadism and gynecomastia. This patient's normal
prolactin and TSH levels and elevated gonadotropins do not suggest a pituitary disorder.
Testicular biopsy in patients with Klinefelter syndrome may show abnormalities of seminiferous tubules and Leydig cells.
However, these abnormalities are not diagnostic, and biopsy is more invasive than chromosomal analysis.
Summarized Points:
Klinefelter syndrome is characterized by delayed puberty, tall stature, gynecomastia, small firm testes, and hypogonadism with
elevated gonadotropins. Karyotype analysis confirms the diagnosis.
More Detail
This patient has characteristic features of Klinefelter syndrome, which may include delayed puberty, tall stature, gynecomastia,
testosterone deficiency, and elevated gonadotropins. The testes are small and firm due to severely damaged seminiferous
tubules. In contrast, patients with gonadotropin deficiency have small and soft testes. Karyotype analysis confirms the
diagnosis; the most common genotype is 47XXY. Treatment involves testosterone replacement.
Patients with Klinefelter syndrome are at increased risk for breast cancer. Although there is no consensus, some clinicians
order periodic mammograms after age 45. These patients also have a higher risk of developing extra-gonadal germ cell
tumors, non-Hodgkin lymphoma, and lung disease (e.g., chronic obstructive pulmonary disease and bronchiectasis).
References
Objective number 10-3 (Gynecomastia) of Objectives for the Qualifying Examination, 3rd edition. Groth KA, Skakkebæk A, Høst
C, et al. Clinical review: Klinefelter syndrome--a clinical update. J Clin Endocrinol Metab 2013; 98:20.
Question #4
A 4-day-old infant is in the nursery with poor feeding and Intermittent vomiting. The infant was born vaginally after an otherwise
uncomplicated pregnancy. Gender has not been assigned due to ambiguous genitalia. Temperature is 36.7 C (98 F), blood
pressure is 49/18 mm Hg, pulse is 192/min, and respirations are 42/min. The patient's pulse oximetry shows 98% on room air.
Examination shows slow capillary refill and mottled extremities. There is clitoromegaly, partially fused labia, and bilaterally non-
palpable gonads. The rest of the examination is normal. Intravenous access is established and resuscitation with normal saline is
initiated. Which of the following is the most appropriate next step in the management of this infant?
Serum testosterone level can be considered as part of the workup for ambiguous genitalia and is elevated in classic 21-
hydroxylase deficiency. However, it does not aid in the diagnosis or immediate management of CAH, and is therefore not a
priority for this neonate.
Congenital hypothyroidism is generally asymptomatic in the immediate neonatal period. In rare cases, infants may present with
jaundice, lethargy, or poor feeding. However, overt signs of shock (as seen in this infant) are not consistent with congenital
hypothyroidism.
Pyloric stenosis presents with non-bloody, nonbilious projectile vomiting and may gradually progress to hypovolemic shock if
undiagnosed. However, it usually presents at age 4-8 weeks and rarely manifests in the first week of life or with rapid
deterioration.
Echocardiography is used to diagnose congenital heart disease. Cardiogenic shock due to closure of the ductus arteriosus could
present in a similar manner, but the lack of hypoxia, murmur or diminished pulses make heart disease unlikely in this patient
Summarized Points:
Classic "salt-wasting" congenital adrenal hyperplasia (CAH) causes virilization in females and hypotensive shock. All infants with
ambiguous genitalia should be screened for CAH by 17-a hydroxyprogesterone testing. Emergency treatment of adrenal crisis
consists of volume expansion and glucocorticoids.
More Detail
This infant's vomiting, lethargy, and hypotensive shock in the setting of ambiguous genitalia are suggestive of classic
congenital adrenal hyperplasia (CAH). CAH is primarily caused by 21-hydroxylase deficiency, which results in elevated
17-α hydroxyprogesterone levels (usually >3500 ng/dL [105 nmol/L]). Affected infants have decreased glucocorticoid and
mineralocorticoid production ("salt-wasting") but a relative increase in androgen production. Males have normal genitalia, but
affected females can present with partial virilization (seen in this patient). CAH by measurement of 17-α hydroxyprogesterone
levels is included in newborn screening in all states, but symptoms may manifest in the first week of life before the newborn
screening results are available. Once CAH is suspected, 17-α hydroxyprogesterone should be measured followed by immediate
treatment with glucocorticoids (i.e., hydrocortisone). Mineralocorticoids must also be replaced but are not as urgent as
glucocorticoids. Psychosocial support is critical due to the ambiguous genitalia and life-long need for steroid supplementation.
References
Objective number 36-1 (Ambiguous Genitalia) of Objectives for the Qualifying Examination, 3rd edition. Merke DP, Auchus RJ.
Congenital Adrenal Hyperplasia Due to 21-Hydroxylase Deficiency. N Engl J Med 2020; 383:1248.
Question #5
A 62-year-old man with diabetes and hypertension comes to the office for a routine 6-month follow-up visit for his blood pressure.
He is currently taking lisinopril and metformin. On examination, you discover a 1.5-cm mobile and nontender nodule on the right
lobe of a thyroid gland. There is no lymphadenopathy. On further questioning, you find that he has no history of radiation to the
neck and no family history of cancer. He feels well and review of systems is negative. Thyroid stimulating hormone level is
normal. Which of the following is the most appropriate next step in management?
Answer Description
A TSH in an asymptomatic patient is usually sufficient as a first step. T3 and T4 are likely to be of less yield than an FNA, as the
patient clinically is euthyroid. A low TSH may be helpful, however, in suspecting a hyperfunctioning adenoma; a high TSH may
be indicative of autoimmune (Hashimoto) thyroiditis.
A computed tomography scan is inappropriate because the patient has no family history of cancer and has never had radiation to
the neck.
Given the utility of FNA, either proceeding directly to surgery for a possible benign lesion or follow-up alone for a potential
carcinoma are not acceptable.
Summarized Points:
Once TSH is normal, next step is Fine needle aspiration (FNA). A needle is inserted into the nodule, and cells are aspirated and
then examined under a microscope. This is the only test that can reliably differentiate between benign and malignant nodules.
More Detail
Asymptomatic thyroid nodules are often found on clinical examination. Most nodules are benign; however, approximately 5% are
malignant. Risk factors for thyroid cancer include age >60 or <30 and patients with a history of radiation. For a given nodule,
males are also more likely to have malignancy. Fine-needle aspiration (FNA) biopsy is often recommended as the first step in
the evaluation, and may also be the most cost-effective. FNA may have an overall accuracy of as high as 95% in good centers.
References
Kundel A, Zarnegar R, Kato M, et al. Comparison of microarray analysis of fine needle aspirates and tissue specimen in thyroid
nodule diagnosis. Diagn Mol Pathol. 2010 Mar. 19(1):9-14. Objective number 63(Neck Mass, Goiter, Thyroid Disease) of
Objectives for the Qualifying Examination, 3rd edition.
Question #6
A 32-year-old woman presents to her primary care provider with menstrual abnormalities. Over the past few months, she has
had irregular menses and periods of amenorrhea. She also noted weight gain and fatigue over this time. Initial evaluation reveals
a pulse of 80/min, a blood pressure of 160/90 mm Hg, and an oxygen saturation of 98% in room air. Physical examination is
notable for hyperpigmentation of the skin and striae. Low-dose dexamethasone is administered without a significant change in
serum cortisol. Serum adrenocorticotropic hormone (ACTH) is elevated. High-dose dexamethasone is subsequently
administered, leading to a decrease in serum cortisol. Which of the following is most likely responsible for this patient's
condition?
A) Pituitary adenoma
B) Adrenal adenoma
C) Exogenous cortisol administration
D) Non-pituitary ACTH-producing tumor
Explanation
Answer Description
Summarized Points:
Cushing's disease, caused by an ACTH-producing pituitary adenoma, leads to hypercortisolism that is responsive to high-dose
dexamethasone administration.
More Detail
Cushing syndrome is a collection of clinical manifestations resulting from chronic excess cortisol exposure. Symptoms of
hypercortisolism include menstrual irregularities, easy bruising, striae, skin atrophy, glucose intolerance, central fat redistribution,
and hypertension. Hyperpigmentation may occur with increased ACTH, although this is not directly related to hypercortisolism. In
women with adrenal or ACTH-secreting tumors, hyperandrogenism may develop as well.
Cushing syndrome can occur in the setting of exogenous cortisol administration, a cortisol-producing adrenal adenoma, an
ACTH-producing pituitary adenoma (Cushing disease) or a non-pituitary tumor producing ectopic ACTH. Low ACTH suggests
either an adrenal adenoma or exogenous cortisol administration. Dexamethasone suppression testing is used in the
diagnosis of Cushing syndrome and can suggest the etiology. Low-dose dexamethasone administration should lead to a
decrease in cortisol; if cortisol remains high, it is suggestive of Cushing syndrome. High-dose dexamethasone administration
will lead to a decrease in cortisol in Cushing's disease, in which excess ACTH is being produced from a pituitary tumor (as in
the patient described here). Ectopic ACTH production from a non-pituitary source will not be suppressed by high-dose
dexamethasone.
References
Objective number 118-1 (Weight Gain, Obesity) of Objectives for the Qualifying Examination, 3rd edition. Lacroix A, Feelders RA,
Strotokis CA, Nieman LK. Cushing's syndrome. Loncet. 2015;386:913. Lacroix A, Raff H. Dexamethasone suppression tests.
Post TW, ed. UpToDate. Waltham, MA: UpToDate Inc. Accessed April 25, 2020.
Question #7
A 22-year-old woman with type 1 diabetes mellitus is evaluated in the emergency department for diabetic ketoacidosis. She uses
an insulin pump. On physical examination, blood pressure is 100/60 mm Hg, pulse rate is 110/min, respiration rate is 20/min, and
oxygen saturation is 98% with the patient breathing ambient air. Her mucous membranes are dry, and she has mild abdominal
discomfort to palpation. Other than tachycardia, the remainder of the physical examination is normal. Laboratory studies:
Blood Electrolytes
Potassium 3.2 mmol/L (3.5‐5.0)
Sodium 138 mmol/L (135‐145)
Chloride 99 mmol/L (98‐106)
Bicarbonate 10 mmol/L (24‐30)
Phosphorus 0.97 mmol/L (0.81–1.58)
Glucose 27.8 mmol/L
β- Hydroxybutyrate Elevated
pH, venous 7.2
Fluid resuscitation with 0.9% saline is initiated. Which of the following is the most appropriate intravenous treatment?
A) Insulin
B) Phosphorus
C) Potassium chloride
D) Sodium bicarbonate
Explanation
Answer Description
Initiating insulin before potassium replacement could cause life-threatening hypokalemia because insulin shifts potassium into
the intracellular space. Therefore, potassium should be replaced to above 3.3 mmol/L before initiating insulin.
Because DKA is often associated with hypophosphatemia, phosphorus levels should be monitored. However, routine
replacement of phosphorus is not indicated because no therapeutic benefit is evident, except in severe hypophosphatemia
(<0.32 mmol/L). Adverse effects include hypocalcemia and hypomagnesemia. Following the correction of DKA,
hypophosphatemia can generally be treated with food rich in phosphorus, such as dairy products.
Sodium bicarbonate is only considered in patients with severe acidosis (pH <6.9). Alkali therapy may prolong the recovery from
DKA and may result in a residual metabolic alkalosis once DKA is resolved.
Summarized Points:
In diabetic ketoacidosis, potassium should be replaced to greater than 3.3 mmol/L before initiating insulin. Initiating insulin before
potassium replacement will worsen hypokalemia because insulin shifts potassium into the intracellular space and may result in
life-threatening hypokalemia.
More Detail
Glucosuria in diabetic ketoacidosis (DKA) causes an osmotic diuresis and severe volume depletion, which may progress to
lethargy, obtundation, and death if the hyperglycemia, dehydration, and electrolyte abnormalities are not treated aggressively
and early. Aggressive intravenous volume replacement with 0.9% saline is indicated. Electrolyte deficits, such as
potassium, should be replaced, and hyperglycemia should be corrected with intravenous insulin. In DKA, total body potassium
levels are depleted because of shifts from the intracellular to extracellular space caused by ketoacidosis and insufficient
insulin. Potassium urinary losses are generated by glucose osmotic diuresis and result in potassium depletion. Normal or low
serum potassium levels indicate a depletion of body stores and require supplementation before insulin therapy is initiated.
The most appropriate next step is to administer potassium chloride, 20-30 mmol/L until the potassium level is greater than 3.3
mmol/L; then 20-30 mmol/L of potassium can be added to each liter of saline thereafter.
References
Objective number 37-2(Diabetes Mellitus) of Objectives for the Qualifying Examination, 3rd edition. Diabetes Canada Clinical
Practice Guidelines Expert Committee. Diabetes Canada 2018 Clinical Practice Guidelines. Can J Diabetes
2018;42(S1):S1-S296.
Question #8
A 28-year-old gravida 2 para 2 woman comes to the clinic 2 months after the birth of her second child. She complains of the
return of her menses being heavier than normal and of some occasional palpitations. She has no significant past medical history.
Her blood pressure is 130/85 mm Hg, pulse is 110/min, and she is afebrile. Chart review indicates that she has lost 22.7 kg (50
Ib) since her delivery, and she had only gained 16 kg (35 Ib) with her pregnancy. She seems nervous and hyperactive compared
to how she behaved during her pregnancy. She is taking no medications, as she is breastfeeding. As the doctor follows her
closely over the next 2 weeks, her symptoms of tachycardia and weight loss significantly worsen. Laboratory results are as
follows:
Thyroid-stimulating
0.01 mIU/L (0.5-5.0)
hormone
Thyroxine, total T4 160 nmol/L (66-155)
Anti-thyroid peroxidase
35 IU/mL (<9)
antibody
Serum thyroglobulin 68 ng/mL (<33)
A) Propranolol
B) Methimazole
C) Radioiodine ablation
D) Levothyroxine
E) Monitor thyroid function every 4 to 8 weeks
Explanation
Answer Description
In the hyperthyroid phase, radioiodine treatment and antithyroid drugs (i.e., methimazole) are of no value because the synthesis
of T4 and T3 is decreased, not increased as in most other disorders causing hyperthyroidism.
In the hypothyroid phase, women with symptoms should be treated with T4 (levothyroxine). In asymptomatic women, treatment
is recommended when the TSH exceeds 10 µU/L. However, this patient presents with thyrotoxicosis features.
Asymptomatic patient with mild thyroid function test abnormalities should be monitored every four to eight weeks to confirm
resolution of biochemical abnormalities or to detect the development of more severe hypothyroidism, indicating possible
permanent hypothyroidism. However, this patient is complaining of thyrotoxicosis features.
Summarized Points:
Postpartum thyroiditis occurs within a year of pregnancy and is characterized by autoimmune inflammation of thyroid follicles and
release of preformed thyroid hormone. Many of the clinical features in postpartum thyroiditis are mediated by increased
responsiveness to catecholamines, and patients with significant symptoms should be treated with beta-blockers.
More Detail
This patient's clinical features (e.g., heavy period, palpitations, weight loss, anxiety) and suppressed TSH are consistent with
thyrotoxicosis. In light of her recent pregnancy, this is most likely due to postpartum thyroiditis (PT). PT is destructive
thyroiditis induced by an autoimmune mechanism within one year after parturition. Many women experience an initial
hyperthyroid stage, followed by a hypothyroid stage prior to the resolution of symptoms.
PT is associated with a positive thyroid peroxidase (TPO) antibody assay and is considered a variant of chronic
lymphocytic (Hashimoto) thyroiditis. PT is more common in women with preexisting autoimmune thyroid disorders and other
autoimmune endocrinopathies (e.g., type 1 diabetes mellitus) and women may have recurrences with subsequent pregnancies.
Women who have bothersome symptoms of hyperthyroidism can be treated with propranolol or atenolol or metoprolol daily
until their serum T3 and serum free T4 concentrations are normal. For women who are breastfeeding, propranolol is preferred
because, due to high plasma protein binding, it is not concentrated in breast milk as much as other beta-blockers.
References
Objective number 63 (Neck Mass, Goiter, Thyroid Disease) of Objectives for the Qualifying Examination, 3rd edition. Burman
KD. Postpartum thyroiditis. Post TW, ed. UpToDate. UpToDate Inc. Accessed June 24, 2021.
Question #9
A 54-year-old female presents to the hospital with abdominal pain. She is diagnosed with cholecystitis and has a
cholecystectomy performed. The following day she complains of palpitations. On questioning she has had an irregular heartbeat
off/on for the past few months but acutely worsened during this episode. She also notes a 6 Kg weight loss over the past month.
She denies dizziness, syncope, dyspnea, or chest pain. She has a known history of hyperlipidemia but has otherwise been
healthy. Medications include aspirin and atorvastatin. Her blood pressure is 155/88 mmHg, pulse is 134/min, respiration rate is
14/min, temperature is 39.1 C (102.4 F). Oxygen saturation at room air is 96%. Physical examination: the patient is agitated and
appears confused, sometimes answering questions inappropriately. Skin is diaphoretic. Neck is supple without tenderness and
no palpable nodules or enlargement. Cardiac examination reveals an irregular rhythm, tachycardia, and no murmurs. ECG
confirms atrial fibrillation with the rapid ventricular response. TSH is <0.01 mU/L (0.4‐5.0). What is the most appropriate next
step in the management of this patient?
Radioactive iodine uptake (RAIU) at this point will delay treatment which is not appropriate given the morbidity/mortality
associated with thyroid storm.
Iodine is a treatment option used in thyroid storm; however, it should be given AFTER methimazole or propylthiouracil (PTU) to
avoid its use as a substrate for more thyroid hormone synthesis.
Thyroidectomy should only be performed once the hyperthyroidism is treated given the mortality rate associated with surgery in
thyroid storm.
Summarized Points:
Thyrotoxic shock, or thyroid storm, is a complication of thyrotoxicosis that usually is preceded by infection, trauma, surgery, or
diabetic ketoacidosis. β-Blockers such as propranolol are mainstays in the initial management.
More Detail
The patient is presenting with thyrotoxic shock, or thyroid storm. This is a complication of thyrotoxicosis that is usually
preceded by infection, trauma, surgery, or diabetic ketoacidosis. Patients typically present with jaundice, nausea and vomiting,
diarrhea, seizure, delirium, fever, tachycardia, and hypertension. If untreated, thyrotoxic shock can be lead quickly to death, and
even with treatment, mortality is as high as 30%. Death is usually due to cardiac failure, arrhythmias, and hyperthermia. β
Blockers such as propranolol are mainstays in the initial management of patients with thyrotoxic shock. β Blockers mediate
symptoms of hyperthyroidism such as palpitations, trembling, and anxiety. Propranolol also inhibits thyroxine deiodinase, which
blocks the conversion of T4 to T3.
References
Burch HB, Wartofsky L. Life-threatening thyrotoxicosis. Thyroid storm. Endocrinol Metab Clin North Am. 1993 Jun. 22(2):263-77.
Objective number 63 (Neck Mass, Goiter, Thyroid Disease) of Objectives for the Qualifying Examination, 3rd edition.
Question #10
A 23-year-old woman comes to the physician due to lethargy and palpitations during the first trimester of pregnancy. Thyroid
function tests are ordered. Which of the following changes are expected during a normal pregnancy?
Increased estrogen levels raise serum thyroxine-binding globulin, resulting in higher total triiodothyronine (T3) and total thyroxine
(T4) levels.
β-hCG increases thyroid hormone synthesis during the first trimester by binding to thyrotropin (TSH) receptors on thyroid
follicular cells. As a result of the higher T3 and T4 levels, TSH secretion is suppressed.
Summarized Points:
Pregnancy induces a number of changes in thyroid physiology: increased serum thyroxine-binding globulin increased thyroid
hormone production and higher total thyroid hormone levels. Patients with primary hypothyroidism will require an increase in
levothyroxine dose during pregnancy.
More Detail
Thyroid physiology changes during pregnancy to meet the increased metabolic demands. During the first trimester, β-hCG binds
to thyrotropin (TSH) receptors on thyroid follicular cells and stimulates thyroid hormone production. The resulting elevations
in T3 and T4 levels suppress TSH secretion. This transient hyperthyroid state is subclinical and considered a normal finding
of pregnancy. Later in pregnancy, as β-hCG levels decline, TSH and thyroid hormone levels return to the normal range.
Concurrently, higher estrogen levels cause an increase in serum thyroxine-binding globulin, which leads to higher total
triiodothyronine (T3) and total thyroxine (T4) levels. However, free thyroxine is only mildly increased.
Patients with primary hypothyroidism are unable to increase thyroid hormone production and will typically require an increase
of 25%-50% in levothyroxine dose during pregnancy. Free T4 assays in pregnancy are subject to artifact and are frequently
unreliable; total T4 increases predictably during pregnancy and should be followed using pregnancy-specific norms.
References
Objective number 80(Pregnancy) of Objectives for the Qualifying Examination, 3rd edition. Moleti M, Trimarchi F, Vermiglio F.
Thyroid physiology in pregnancy. Endocr Pract. 2014 Jun;20(6):589-96.
Question #11
A 76-year-old male is brought to the emergency department in a coma. He was delivered to the emergency department from a
nursing home and was reported by the nursing home staff to have had a seizure that lasted less than 1 minute. He was
subsequently confused and soon thereafter entered a comatose state. His medical history is significant for type II diabetes
mellitus requiring insulin, hypertension, and mild congestive heart failure. In the emergency department, the patient is very
lethargic and responds only to pain stimuli. His vital signs are stable, and complete blood count and electrolyte levels are normal.
Serum glucose is 0.88 mmol/L and serum insulin is elevated. C-peptide is 0.2 ng/mL (normal range, 0.5 to 3 ng/mL). Urine
sulfonylurea level is undetectable. What is the likely cause of this patient’s condition?
Since this patient presents with a decreased C-peptide level, this is not consistent with an ENDOGENOUS source of insulin
since pancreatic ß-cells produce proinsulin (which breaks down into insulin and C-peptide). Therefore, insulinoma is not the
diagnosis since there are decreased levels of C-peptide in this patient. Note that surreptitious sulfonylurea use will also produce
elevated insulin and C-peptide levels (similar to insulinoma) since this drug essentially stimulates proinsulin secretion. That is
why it is critical to order a urine sulfonylurea level, which is undetectable in this patient.
Summarized Points:
The patient in this question is presenting with clinical symptoms and laboratory findings consistent with surreptitious injection of
insulin (elevated insulin, decreased glucose, and decreased C-peptide).
More Detail
The patient in this question is presenting with clinical symptoms and laboratory findings consistent with surreptitious injection of
insulin (elevated insulin, decreased glucose, and decreased C-peptide).
References
Garza H. Minimizing the risk of hypoglycemia in older adults: a focus on long-term care. Consult Pharm. 2009 Jun. 24 Suppl
B:18-24. Objective number 37-1 (Glucose Abnormalities) of Objectives for the Qualifying Examination, 3rd edition.
Question #12
A 44-year-old man is evaluated for left breast enlargement. Breast enlargement started about 3 months ago and has been
progressively worsening. His mother and sister have had breast cancer. On physical examination, vital signs are normal. BMI is
30 kg/m2. Palpation of the left breast reveals a firm, nontender 3-cm subareolar mass. The right breast examination is normal.
The testicular examination and the remainder of the physical examination are normal. Which of the following is the most
appropriate test?
A) Mammography
B) Luteinizing hormone
C) 8 AM Serum testosterone
D) Testicular ultrasonography
E) Human chorionic gonadotropin
Explanation
Answer Description
Gynecomastia is diagnosed by physical examination. Palpation of subareolar glandular tissue greater than 0.5 cm in diameter is
consistent with gynecomastia. Gynecomastia is typically bilateral and tender to palpation, particularly early in its course of
development. This patient does not have a physical examination consistent with gynecomastia. The initial laboratory evaluation
for gynecomastia includes measurement of 8 AM serum testosterone, human chorionic gonadotropin (hCG), luteinizing
hormone, and estradiol. These tests are not indicated for this patient because he does not have gynecomastia on examination
but rather a breast mass that requires imaging and biopsy.
Because testicular germ cell tumours can cause gynecomastia, evaluation should include testicular examination and hCG
measurement. Testicular ultrasonography is recommended if hCG is elevated or a testicular mass is palpated on physical
examination. In this case, the patient did not have gynecomastia on examination, so neither hCG measurement nor testicular
ultrasonography are indicated.
Summarized Points:
A unilateral, nontender, fixed breast mass in a male patient raises concern for breast cancer, and mammography is indicated.
The highest rates of male breast cancer are found in those with Klinefelter syndrome and in persons with a family history of
BRCA2-positive breast cancer.
More Detail
A unilateral, nontender, fixed breast mass should raise concern for breast cancer, and imaging with mammography and
subsequent breast biopsy is warranted. Other physical examination features of male breast cancer include nipple involvement/
retraction (approximately 40%-50% of cases) overlying skin changes or ulceration, and axillary adenopathy. Gynecomastia is a
benign proliferation of glandular tissue in males, and patients with gynecomastia of relatively recent onset may experience
breast tenderness, but this finding is unusual in male breast cancer. Only 0.2% of all cancers in men are male breast cancer. The
highest rates of male breast cancer are found in those with Klinefelter syndrome and those with a family history of
BRCA2-positive breast cancer, which is likely in this patient. Other conditions that may predispose to male breast cancer
include abnormally high estrogen states that may occur in patients with obesity, liver disease, or testicular disorders. Chest
irradiation is also a risk factor. Male breast cancer has been associated with gynecomastia, particularly if unilateral or
asymmetric, and should always be included in the differential diagnosis.
References
Objective number 10(Breast Disorders) of Objectives for the Qualifying Examination, 3rd edition. Kanakis GA, Nordkap L, Bang
AK, et al. EAA clinical practice guidelines-gynecomastia evaluation and management. Andrology. 2019 Nov;7(6):778-793.
Gradishar WJ, Ruddy KJ. Breast cancer in men. Post TW, ed. UpToDate. Waltham, MA: UpToDate Inc. Accessed June 24, 2022.
Question #13
Patient presents to the Emergency Department for altered mental status and decreased level of consciousness. Labs indicate
that he has a glucose level of 1.6 mmol/L (3.3‐5.8) and calcium level of 2.9mmol/L (2.18‐2.58). Intact parathyroid (PTH) level
is increased. You administer Dextrose 5% fluid to the patient and his level of consciousness improves. What diagnostic or lab
test is indicated at this time?
Pro-insulin and C peptide level tests) is not needed at this time. You would check these tests in an individual if you suspected
surreptitious injection of insulin.
Thyroid function tests is also not necessary at this time because the patient has low sugar and high calcium - which would not
affect thyroid function tests.
Calcitonin level and Urine metanephrine level may be indicated with MEN IIA and MEN IIB.
Summarized Points:
Patient who has low glucose, high calcium, and a high intact PTH, the top differential diagnosis we should be concerned
about is multiple endocrine neoplasia (MEN) type I. This condition is associated with three Ps:
• Parathyroid hyperplasia
• Pancreas islet cell tumors
• Pituitary tumors
More Detail
With this patient who has low glucose, high calcium, and a high intact PTH, the top differential diagnosis we should be
concerned about is multiple endocrine neoplasia (MEN) type I. MEN type I, also known as Wermer's syndrome, is a rare
disorder causing tumors in the endocrine glands and portions of the gastrointestinal tract. This condition is associated with three
Ps:
• Parathyroid hyperplasia
• Pancreas islet cell tumors
• Pituitary tumors
The pancreas may be secreting a lot of insulin that is causing the low sugar. The PTH is likely increased from parathyroid
hyperplasia that is leading to an elevated calcium level. Now that we have two of the "Ps", we need to visualize the pituitary to
look for pituitary tumors. The best way to visualize the pituitary is MRI of the pituitary.
References
Objective number 12-2 (Calcium Disorders) of Objectives for the Qualifying Examination, 3rd edition. Thakker RV, Newey PJ,
Walls GV, et al. Clinical practice guidelines for multiple endocrine neoplasia type 1 (MEN1). J Clin Endocrinol Metab 2012;
97:2990.
Question #14
A 67-year-old man is evaluated for management of type 2 diabetes mellitus, diagnosed 6 years ago. For the past 4 months, he
has had fatigue and unsteadiness while walking. Medications are metformin and glipizide, which he has been taking since his
diagnosis. On physical examination, vital signs are normal. BMI is 28 kg/m2. He has decreased vibratory sense in the great toes
and loss of patellar and Achilles reflexes. The remainder of the examination is normal. Laboratory studies:
The depressed reflexes are also inconsistent with a process limited to the spinal cord; therefore, an MRI of the spine is
unnecessary.
Vitamin B6 (pyridoxine) deficiency presents as nonspecific stomatitis, glossitis, cheilosis, confusion, bilateral distal limb
numbness and burning paresthesia. Distal limb weakness is rare. This patient's presentation is not consistent with this
deficiency, so vitamin B6 measurement is unnecessary.
Although electromyography and nerve conduction studies would likely be abnormal in this patient, they would not help determine
the cause of peripheral polyneuropathy because these tests are not specific for subacute combined degeneration. This patient's
presentation is not consistent with a compressive spinal or nerve root process such as a herniated disc or an epidural mass and
cannot account for the patient's macrocytic anemia.
Summarized Points:
Long-term use of metformin is associated with an increased risk for developing vitamin B12 deficiency. Vitamin B12 levels should
be measured annually in patients receiving long-term metformin therapy.
More Detail
This patient with type 2 diabetes mellitus has new neuropathic symptoms that have developed during the past 4 months.
His examination findings of decreased vibratory sense and areflexia suggest posterior column disease. Laboratory findings
reveal macrocytic anemia. The combination of neurologic findings and anemia in a patient taking metformin for several years is
consistent with vitamin B12 deficiency. Although the mechanism is not entirely understood, it is believed to be related to
interference with the absorption of food-derived B12 at the level of the ileum. A high level of suspicion for vitamin B12
deficiency is needed in a patient with type 2 diabetes taking long-term metformin who develops peripheral polyneuropathy.
Anemia and macrocytosis may not be present, and neurologic symptoms and findings may be the only manifestation of vitamin
B12 deficiency.
A serum vitamin B12 level test is most often used for the initial assessment. Treatment is oral or parenteral vitamin B12
replacement; metformin may be continued. The prevalence of vitamin B12 deficiency approaches 20% in patients taking
metformin for 5 years. Therefore, vitamin B12 levels should be checked annually in patients receiving long-term metformin
therapy.
References
Objective number 37-2(Diabetes Mellitus) of Objectives for the Qualifying Examination, 3rd edition. Aroda VR, Edelstein SL,
Goldberg RB, et al; Diabetes Prevention Program Research Group. Long-term Metformin Use and Vitamin B12 Deficiency in the
Diabetes Prevention Program Outcomes Study. J Clin Endocrinol Metab. 2016 Apr;101(4):1754-61. Wexler DJ. Metformin in the
treatment of adults with type 2 diabetes mellitus. Post TW, ed. UpToDate. Waltham, MA: UpToDate Inc. Accessed May 31, 2022.
Question #15
A 27-year-old athlete comes to the physician with a 2-month history of fatigue, erectile dysfunction, and muscle weakness. On
examination, he has well-developed musculature but mild gynecomastia and small, soft testes. Which of the following laboratory
abnormalities would confirm surreptitious androgen abuse in this patient?
Men with prolactinomas have decreased libido, erectile dysfunction, and symptoms of tumor impingement (e.g., headache,
peripheral vision loss). Case reports have noted increased prolactin levels in prolactinoma patients taking exogenous androgens,
but prolactin is not established as a reliable marker of androgen use.
Exogenous androgens have been found to have a small effect on thyroid function, but TSH levels usually remain within normal
limits.
Serum insulin-like growth factor-1 (IGF-1) may be used to screen for illicit growth hormone use, which is sometimes used as a
performance-enhancing substance. Exogenous androgen use will further increase IGF-1 levels, but IGF-1 is not a specific
marker of androgen abuse.
Exogenous androgens can cause erythrocytosis by increasing erythropoietin and decreasing serum hepcidin levels. However,
transferrin saturation will be normal.
Summarized Points:
Laboratory testing in patients with anabolic-androgenic steroid abuse shows increased hematocrit and decreased levels of
endogenous testosterone, LH, and FSH.
More Detail
The correct answer is E.
This patients’ clinical features (fatigue, erectile dysfunction, and testicular atrophy) are suggestive of testosterone deficiency,
likely due to cessation of anabolic-androgenic steroid use. Exogenous androgens (from natural or synthetic sources) suppress
the pituitary-testicular axis, resulting in decreased testicular size and function (decreased testosterone secretion and
spermatogenesis). With ongoing use of anabolic steroids, patients will have low levels of gonadotropins (LH, FSH) and
endogenous testosterone. During this time, they may develop infertility due to testicular failure but may not have erectile
dysfunction due to adequate levels of circulating exogenous androgens. The suppression is reversible when anabolic steroids
are stopped, but recovery of normal gonadotropin and testosterone levels may take several months. Some features, such as
gynecomastia, may not be reversible.
References
Objective number 46 (Infertility) of Objectives for the Qualifying Examination, 3rd edition. Abbate V, Kicman AT, Evans-Brown M,
et al. Anabolic steroids detected in bodybuilding dietary supplements - a significant risk to public health. Drug Test Anal 2015;
7:609.
Question #16
A 65-year-old female presents to your clinic with questions regarding her new diagnosis of osteoporosis. She had a hip fracture 6
months ago after a fall. Subsequently, she had a prolonged hospital course complicated by a deep vein thrombosis for which she
completed therapy with warfarin. She has had no further falls and is back to her baseline level of activity after a 1-month
rehabilitation stay. She had a DEXA scan which showed a T-score of - 2.6 in the hip and - 2.3 in vertebrae. Her past medical
history includes achalasia for which she gets dilatations as needed. Currently she only takes omeprazole on a daily basis. Labs
including renal function, calcium, and vitamin D are all normal. She is inquiring about further options to prevent her from having
fractures. What is the appropriate management of this patient besides recommending continued supplementation with vitamin D
and calcium?
A) Zoledronic acid
B) Alendronate
C) Raloxifene
D) Calcitonin
E) Estrogen
https://afkebooks.com
Explanation
Answer Description
Alendronate is an oral bisphosphonate and would be contraindicated given achalasia and the risk of pill esophagitis.
Raloxifene is a SERM and has evidence only for reduction of vertebral fractures and would thus not be an ideal choice; in
addition, it is contraindicated given the prior history of thromboembolic disease.
Estrogen is no longer used for management of osteoporosis given many risks, one of them deep vein thrombosis/ pulmonary
embolism, and this medication would be contraindicated in this patient.
Summarized Points:
Pharmacologic therapy for osteoporosis is indicated for patients with a T-score -2.5 and for those with fragility fractures
regardless of bone mineral density. Bisphosphonates are first-line treatment in most cases.
More Detail
The patient has osteoporosis and a history of a fracture; thus, every attempt would be made to start the patient on a medication
to reduce her risk of fractures. Zoledronic acid is a bisphosphonate which will reduce both vertebral/nonvertebral fractures; in
https://afkebooks.com
addition, it is given intravenously and is most appropriate in this patient with achalasia.
References
Bono CM, Einhorn TA. Overview of osteoporosis: pathophysiology and determinants of bone strength. Eur Spine J. 2003 Oct. 12
Suppl 2:S90-6. Objective number 67-1-2-4 (Local Pain, Spinal Compression/Osteoporosis) of Objectives for the Qualifying
Examination, 3rd edition.
https://afkebooks.com
Question #17
A 39-year-old male-to-female transgender woman comes to the office for a preventive care visit. She has been receiving
feminizing hormonal therapy for the last 5 years and underwent gender reassignment surgery with augmentation mammoplasty a
year ago. Her past medical history is otherwise unremarkable. Current medications include spironolactone and conjugated
estrogens. Blood pressure is 128/84 mm Hg, pulse is 80/min, and BMI is 23 kg/m2. Postoperative changes are noted at the
breasts and genitalia, but the remainder of the examination is normal. Laboratory studies show a normal comprehensive
metabolic panel and lipid panel. HIV screen is negative. Which of the following additional studies is most appropriate at this
time?
https://afkebooks.com
Explanation
Answer Description
MTF patients may experience bone loss similar to natal women of the same age. However, peak bone density is typically higher
than that of natal women due to male androgen levels in early life. Bone density screening is recommended after age 65, or
sooner if the patient had an orchiectomy and has not been on hormone replacement.
https://afkebooks.com
MTF patients on estrogen have a slightly increased risk of breast cancer. However, mammography in MTF patients has a higher
false-positive rate than in natal women. In general, MTF patients on estrogen should be offered mammography at age 50, but
recommendations should be individualized.
Papanicolaou testing is recommended to screen for cervical cancer in FTM patients who still have a cervix. It is not needed in
MTF patients.
The prostate gland is not removed in standard gender reassignment procedures. However, the risk of prostate cancer is likely
lower in MTF patients who have had orchiectomy than in men with intact gonads.
Summarized Points:
Transgender patients on hormone therapy should have regular monitoring of sex hormone levels, with dose adjustment to
maintain levels within the physiologic range for the desired gender.
More Detail
Prior to gender reassignment surgery, male-to-female (MTF) transgender patients require high-dose estrogens and anti-
androgens to achieve feminizing effects. Following removal of the testes, the doses can be decreased substantially. However,
even at lower doses, sex hormone supplementation can be associated with adverse effects (e.g., venous thromboembolism).
MTF patients on hormonal therapy should have estrogen and testosterone levels measured once or twice yearly, with doses
adjusted to maintain levels within the normal range for premenopausal women. Hepatic enzymes, glucose, and lipids can
also be affected by hormone therapy and should be monitored periodically. If spironolactone is being used as an anti-androgen,
electrolytes should be checked regularly as well.
Female-to-male (FTM) transgender patients often receive substantial doses of androgens. These patients should be monitored
with testosterone levels, blood counts (to rule out erythrocytosis), hepatic markers, and glucose and lipid testing.
https://afkebooks.com
References
Objective number 94(Sexually Concerned Patient) of Objectives for the Qualifying Examination, 3rd edition. Hembree WC,
Cohen-Kettenis PT, Gooren L, et al. Endocrine Treatment of Gender-Dysphoric/Gender-Incongruent Persons: An Endocrine
Society Clinical Practice Guideline. J Clin Endocrinol Metab 2017; 102:3869.
https://afkebooks.com
Question #18
A 13-year-old soccer player is brought to the clinic for follow-up on type 1 diabetes mellitus. He was diagnosed 1 month ago after
presenting with polyuria, polydipsia, weight loss, and random blood glucose levels of 28 mmol/L and 27.5 mmol/L. He was
started on a subcutaneous insulin regimen and initially had stable blood sugars ranging between 5.7-8 mmol/L. Over the past
few weeks, his blood glucose levels have been consistently <4.8 mmol/L before each meal and at bedtime, and his insulin dose
has decreased significantly. He is back in school but has not resumed sports yet. Which of the following is the most likely
explanation for the changes in this patient's blood glucose levels?
A) The patient is entering adolescence, an insulin-sensitive state, and his insulin requirement will continue to decrease
B) The patient is likely miscalculating the insulin dose and administering too much insulin
C) The patient is in the remission phase of diabetes mellitus and his insulin requirement will likely increase again
D) The patient was incorrectly diagnosed with diabetes mellitus and can discontinue insulin therapy
E) The patient's insulin requirement is decreasing because he is not exercising as much as usual
https://afkebooks.com
Explanation
Answer Description
Adolescence is a period of insulin resistance rather than sensitivity. During this period, tight glycemic control is difficult to achieve
and insulin requirements often increase rather than decrease.
Though it is possible that the patient is miscalculating his dose, this is less likely as he is checking his blood glucose frequently
and decreasing his insulin dose accordingly.
Given that this patient's clinical presentation was classic for T1 DM and he had 2 separate elevated blood glucose
measurements, it is unlikely that he was diagnosed incorrectly. It is more likely that he is in the "honeymoon" phase of T1 DM.
Exercise induces hypoglycemia by increasing glucose utilization and insulin sensitivity. The fact that this patient is not exercising
should not decrease his insulin requirement or induce hypoglycemia.
Summarized Points:
The "honeymoon" or "remission" phase of type 1 diabetes mellitus (T1 DM) occurs several weeks after the diagnosis of new
onset TIDM and is characterized by an increase in endogenous insulin production. It presents with decreased insulin
requirement and can last for months to years.
More Detail
https://afkebooks.com
This patient's clinical presentation is consistent with new onset type 1 diabetes mellitus (T1 DM). TIDM is typically diagnosed
before age 19; males and females are equally affected. It classically presents with polyuria, polydipsia, weight loss, and fatigue.
The weight loss is due to increased catabolism in the form of muscle and fat breakdown, which occurs because the body is
unable to utilize glucose properly. TIDM treatment consists of subcutaneous insulin injections or as a continuous infusion through
an insulin pump. Insulin regimens often have to be titrated depending on nutrition and exercise to achieve euglycemia. However,
after blood glucose levels have stabilized, nearly all children who are diagnosed with TIDM experience a "honeymoon" or
"remission" phase several weeks after the initial diagnosis and initiation of insulin treatment. This phase occurs when the
few remaining functional beta cells begin secreting some insulin, which decreases the exogenous insulin requirement. This
phase may last for several months to years and the patient should be monitored very closely for hypoglycemia. After the
"honeymoon" phase, the patient will become hyperglycemic again and require higher doses of exogenous insulin.
References
Objective number 37-2 (Diabetes Mellitus) of Objectives for the Qualifying Examination, 3rd edition. Sokołowska M, Chobot A,
Jarosz-Chobot P. The honeymoon phase - what we know today about the factors that can modulate the remission period in type
1 diabetes. Pediatr Endocrinol Diabetes Metab. 2016;22(2):66-70.
https://afkebooks.com
Question #19
A 23-year-old woman is evaluated in the office for a 6-month history of intermittent nausea, anorexia, and occasional
constipation. She does not smoke cigarettes, drink alcohol, or use recreational drugs. She otherwise feels well and takes no
medications. Family history is unremarkable. Vital signs and physical examination are normal. Hypercalcemia was noted on an
initial metabolic profile. Repeat laboratory studies:
A) Chest radiography
B) Neck ultrasonography
C) Parathyroid hormone-related protein measurement
D) Urine calcium-creatinine ratio determination
https://afkebooks.com
Explanation
Answer Description
Neck ultrasonography may be reasonable to consider in a patient with PTH-dependent hypercalcemia to locate an adenoma
before surgery. However, this patient's PTH is suppressed, making hyperparathyroidism unlikely.
Tumor-produced PTH-related protein (PTHrP) is the most common cause of hypercalcemia of malignancy. As in this patient,
PTH would be suppressed but 1,25-dihydroxy vitamin D would not be elevated and serum phosphorus would be low. Most
patients with humoral hypercalcemia of malignancy have advanced cancer associated with severe hypercalcemia; tumour-
produced PTHrP is an unlikely mechanism of hypercalcemia in this otherwise well young patient. Therefore, PTHrP
measurement is inappropriate for this patient.
Urine calcium-creatinine ratio determination is useful to confirm the diagnosis of familial hypocalciuric hypercalcemia (FHH).
Patients with this disorder are asymptomatic, have a history of hypercalcemia since childhood, and have a family history of
hypercalcemia. In this condition, the PTH level is elevated and the urine calcium excretion is low, resulting in paradoxical
hypocalciuria in the setting of hypercalcemia. This patient's clinical and biochemical profiles are inconsistent with FHH, so
measurement of the urine calcium-creatinine ratio is unnecessary.
Summarized Points:
Vitamin D-dependent hypercalcemia is associated with a suppressed parathyroid hormone level, hypercalcemia, a high or high-
normal serum phosphorus level, and an elevated 1,25-dihydroxy vitamin D level. Unregulated conversion of 25-hydroxyvitamin D
to 1,25-dihydroxy vitamin D and resultant hypercalcemia may occur in granulomatous tissue associated with fungal infection,
tuberculosis, sarcoidosis, and lymphoma.
More Detail
https://afkebooks.com
The correct answer is A.
This patient has mildly symptomatic hypercalcemia and a high-normal serum phosphorus level, suppressed parathyroid
hormone (PTH), and an elevated 1,25-dihydroxyvitamin D level. Unregulated conversion of 25-hydroxyvitamin D to
1,25-dihydroxyvitamin D may occur in granulomatous tissue associated with fungal infection, tuberculosis, sarcoidosis, and
lymphoma, leading to increased intestinal absorption of calcium. Vitamin D-dependent hypercalcemia is associated with
normal to elevated serum phosphorus levels because vitamin D enhances intestinal absorption of phosphorus, and suppressed
PTH secretion reduces kidney phosphorus excretion. In the absence of an established cause of vitamin D-dependent
hypercalcemia, such as documented ingestion, a chest radiograph to diagnose sarcoidosis, fungal infection, tuberculosis,
or lymphoma is reasonable. In this young otherwise healthy patient, pulmonary sarcoidosis causing vitamin D-dependent
hypercalcemia is probable.
References
Objective number 12-1(Hypercalcemia) of Objectives for the Qualifying Examination, 3rd edition. Gwadera Ł, Białas AJ, Iwański
MA, Górski P, Piotrowski WJ. Sarcoidosis and calcium homeostasis disturbances-Do we know where we stand? Chron Respir
Dis. 2019 Jan-Dec;16:1479973119878713. Shane E. Diagnostic approach to hypercalcemia. Post TW, ed. UpToDate. Waltham,
MA: UpToDate Inc. Accessed June 07, 2022.
https://afkebooks.com
Question #20
A 68-year-old woman comes to the physician for 5 weeks of persistent pain and mild jaw swelling after extraction of a left lower
molar. She was diagnosed with multiple myeloma 8 months ago and treated with dexamethasone, lenalidomide, and zoledronic
acid. Physical examination shows gingival ulceration at the site of the extraction with exposed bone and mild inflammation.
Which of the following is the most likely diagnosis?
A) Actinomycosis
B) Osteonecrosis
C) Gingival hyperplasia
D) Necrotizing fasciitis
E) Plasmacytoma
https://afkebooks.com
Explanation
Answer Description
Cervicofacial actinomycosis occurs most commonly in the mandible. Patients typically develop a slowly progressive and non-
tender mass that extends through tissue planes to form multiple abscesses, fistulas, and purulent drainage containing yellowish
"sulfur" granules.
Drug-induced gingival hyperplasia is most commonly due to phenytoin, calcium channel blockers (e.g., diltiazem), or
cyclosporine. Patients usually develop gingival overgrowth that can cause difficulty with chewing.
Necrotizing fasciitis is a deep-tissue infection that progressively destroys muscle fascia rapidly over several days. Patients
usually have pain out of proportion to the examination findings. This patient's exposed bone and subacute course make this less
likely.
A plasmacytoma may form a destructive lesion in the bone but more commonly presents with pain or pathologic fracture in the
axial skeleton (e.g., vertebrae, pelvis, ribs), rather than in the face or mandible.
Summarized Points:
Osteonecrosis of the jaw is a serious but uncommon complication of bisphosphonates or denosumab that occurs most often in
patients with malignancy. Patients can develop pain, loosening of teeth, bony enlargement, gingival swelling or erythema, and
exposed bone. Symptoms are more common at the site of dental extractions.
More Detail
https://afkebooks.com
The correct answer is B.
This patient's clinical features are consistent with osteonecrosis of the jaw (ONJ). Medication-related ONJ can occur with any
bisphosphonate therapy, but it is most common in patients with malignancy (e.g., breast cancer, multiple myeloma) treated
with high-potency bisphosphonates (e.g., zoledronic acid) or denosumab. Patients can be asymptomatic or develop local
pain, loosening of teeth, bony enlargement, gingival swelling or erythema, and exposed bone. Symptoms can occur
spontaneously but are more common following dental extraction. Current guidelines recommend a comprehensive dental
examination and preventive dentistry (e.g., teeth extraction) before starting high-potency antiresorptive agents. Treatment of
medication-related ONJ includes stopping the offending agent and local conservative management.
References
Objective number 67-1-2-4 (Local Pain, Spinal Compression/Osteoporosis) of Objectives for the Qualifying Examination, 3rd
edition. Yarom N, Shapiro CL, Peterson DE, et al. Medication-Related Osteonecrosis of the Jaw: MASCC/ISOO/ASCO Clinical
Practice Guideline. J Clin Oncol 2019; 37:2270.
https://afkebooks.com
Question #21
A 51-year-old overweight man with type 2 diabetes mellitus comes to the physician for an ulcer on his right foot. His vital signs
are within normal limits. Examination shows a 1.3 x 2 cm ulcer on the plantar surface beneath the head of the fifth metatarsal.
Lower extremity skin examination is otherwise normal. Which of the following is the best test to identify the primary underlying
risk factor for this patient's ulcer?
https://afkebooks.com
Explanation
Answer Description
The 6-minute walk test is an assessment of functional status that measures how far a patient can walk in a standardized time. It
is typically used for patients with chronic lung or heart disease or with chronic pain conditions (e.g., fibromyalgia, osteoarthritis).
It is not as useful for patients with diabetic neuropathy or foot ulcers.
Peripheral arterial disease (PAD) can be assessed by calculating the ankle-brachial index (ABI). However, ABI is primarily a
measure of large vessel PAD and does not accurately assess small vessel disease, which often contributes to ulcers in diabetic
patients. Also, arterial ulcers are usually located on the tips of the toes rather than the plantar surface.
Delayed capillary refill time (>3 seconds) indicates impaired limb perfusion and may suggest underlying PAD. However, a
delayed capillary refill is a nonspecific finding and is also seen in hypotension and volume depletion.
Diabetic neuropathy can affect the large nerve fibers in the lower extremities, causing decreased vibratory sensation,
paresthesia, and decreased/absent ankle reflexes. However, the knee reflex is usually not affected.
Summarized Points:
Risk factors for diabetic foot ulcers include diabetic neuropathy, previous foot ulceration, vascular disease, and foot deformity.
Diabetic neuropathy is the most important contributing factor and is found in >80% of patients with ulcers. Monofilament testing
predicts the risk of future ulcers.
More Detail
https://afkebooks.com
The correct answer is B.
This patient has a diabetic foot ulcer, which can lead to eventual nontraumatic lower-limb amputations. Risk factors for
diabetic foot ulcers include diabetic neuropathy, previous foot ulceration, vascular disease, and foot deformity. Diabetic
neuropathy is the most common underlying cause and is found in >80% of patients with diabetic foot ulcers. Neuropathy
decreases pain sensation and pressure perception causes muscle imbalance leading to foot deformities and impairs the
microcirculation and integrity of the skin.
Neuropathic ulcers most commonly occur in the feet under bony prominences, such as the metatarsal heads. They typically
have a punched-out or undermined border. Peripheral sensory neuropathy can be documented accurately by testing for pressure
sensation with a 10-g monofilament (placed on the plantar surface at a right angle with increasing pressure until filament
buckles). Patients with neuropathy have a higher pressure threshold and loss of monofilament sensation, which are
associated with an increased risk of foot ulcerations. Other sensory deficits may include decreased vibratory sensation
(tested with a tuning fork), decreased pinprick pain, and decreased temperature sensation.
References
Objective number 37-2(Diabetes Mellitus) of Objectives for the Qualifying Examination, 3rd edition. Feng Yuzhe , Felix
Schlösser, Bauer E Sumpio . The Semmes Weinstein monofilament examination is a significant predictor of the risk of foot
ulceration and amputation in patients with diabetes mellitus. J Vasc Surg 2011;53:220.
https://afkebooks.com
Question #22
A 73-year-old woman is brought to the physician by her daughter due to intermittent dizziness, weakness, and confusion. The
patient was seen for fever and dysuria 3 days ago and started on trimethoprim/sulfamethoxazole. Her other medical problems
include type 2 diabetes mellitus, hypertension, and chronic kidney disease with a baseline serum creatinine of 114 µmol/
L(50-90). Her chronic medications include hydrochlorothiazide, amlodipine, glyburide, atorvastatin, calcium, vitamin D, and
alendronate. Her temperature is 36.8 C (98.2 F), blood pressure is 140/85 mm Hg supine and 144/89 mm Hg standing, pulse is
84/min, and respirations are 20/min. BMI is 19 kg/m2. There are no focal neurologic findings. Her Mini-Mental State Examination
score is 24/30. Laboratory results are as follows:
https://afkebooks.com
Hemoglobin A1C 6.1%
Which of the following is the best next step in the management of this patient?
A) Discontinue glyburide
B) Discontinue hydrochlorothiazide
C) Order CT scan of the head
D) Order renal ultrasound
E) Send urine cultures and adjust antibiotics
https://afkebooks.com
Explanation
Answer Description
Diuretic use in elderly patients can cause orthostatic hypotension, dizziness, and falls. However, this patient has no signs of
hypovolemia or orthostasis. Loop diuretics are preferred over thiazides for patients with hypertension and advanced CKD
(glomerular filtration rate <30 ml/min/1.73 m2). However, this patient has good blood pressure control and her antihypertensive
regimen need not be changed at this time.
In the absence of focal neurologic findings, brain imaging studies (CT or MRI) are not recommended at this visit but may be
indicated if her symptoms continue.
Renal ultrasound can detect obstructive uropathy (from kidney stones) or perinephric abscess, which can present with persistent
costovertebral angle pain and pyuria after urinary tract infections or pyelonephritis. However, this patient's stable vital signs and
normal white blood cell count make these less likely.
Absence of fever and resolution of dysuria make persistent urinary tract infection unlikely in this patient. Her minimal pyuria and
bacteriuria on urinalysis may indicate asymptomatic bacteriuria, which does not require further evaluation.
Summarized Points:
Sulfonylureas (e.g., glyburide) can accumulate in elderly patients with chronic kidney disease (CKD) and cause hypoglycemia;
this effect is potentiated by trimethoprim/sulfamethoxazole. Goals for diabetes control should be relaxed in elderly individuals
with CKD.
https://afkebooks.com
More Detail
This elderly woman has features suggesting symptomatic hypoglycemic episodes that are likely due to a drug interaction
between trimethoprim/sulfamethoxazole (TMP-SMX) and glyburide in the setting of chronic kidney disease (CKD). Glyburide
is a long-acting sulfonylurea medication that can accumulate in patients with CKD and carries a significant risk of
hypoglycemia in elderly patients. TMP-SMX can potentiate the effect of sulfonylureas and cause unexpected hypoglycemia in
previously stable patients. This patient's hemoglobin Are (HbA1C) indicates well-controlled blood glucose over the past 3
months, and she is likely lacking adequate carbohydrate intake. In addition, her CKD decreases gluconeogenesis and
glycogenolysis and impairs her defenses against hypoglycemia. The most appropriate action would be to discontinue
glyburide and readjust her glycemic control to target a higher HbA1C. Many experts recommend an HbA1C of 7.0%-7.9% in
elderly patients with longstanding diabetes and an anticipated life expectancy of <10 years.
References
Objective number 37 (Glucose Abnormal, Serum/Diabetes Mellitus/Polydipsia) of Objectives for the Qualifying Examination, 3rd
edition. Hohl CM, de Lemos J, Abu Laban RB. Emergency hospitalizations for adverse drug events. N Engl J Med. 2012 Mar
1;366(9):858-9; author reply 859-60.
https://afkebooks.com
Question #23
A 56-year-old woman with a history of lung cancer presents to the emergency department with confusion, nausea, and vomiting.
She is unable to provide much history due to her confusion. Her vital signs are normal. Laboratory studies reveal a serum
calcium of 3.5 mmol/L (2.12–2.52) and creatinine of 106 μmol/L (50‐90) What is the next step in her management?
A) Administer bisphosphonates
B) Administer calcitonin
C) Administer furosemide
D) Administer intravenous fluids
https://afkebooks.com
Explanation
Answer Description
Administration of bisphosphonates is a later step undertaken after the patient has been fluid resuscitated. Bisphosphonates
inhibit bone resorption and lead to a sustained decrease in serum calcium as early as 12 hours after administration with an effect
of up to 4 weeks.
Administration of calcitonin will lower serum calcium levels, but tachyphylaxis develops in a few days without prolonged benefit.
Administration of furosemide can result in further diuresis and dehydration without significant changes in serum calcium levels.
Summarized Points:
Hypercalcemia of malignancy is most commonly caused by the secretion of parathyroid hormone-related protein (PTHrP).
Management includes intravenous normal saline (0.9%), calcitonin, and bisphosphonates (denosumab if bisphosphonates are
contraindicated). It may need hemodialysis if refractory to medical treatment.
More Detail
Malignancy induced hypercalcemia can occur in individuals with advanced cancer. Breast cancer, lung cancer, and multiple
myeloma are the most common malignancies associated with hypercalcemia. The elevated calcium is due to parathyroid-
related hormones secreted by the tumour, bony destruction due to osteoclast activation, and tumour production of
vitamin D analogues. Symptoms of malignancy induced hypercalcemia are the same as hypercalcemia from other causes,
including lethargy, confusion, anorexia, constipation, and nausea. The increased calcium concentration leads to an osmotic
https://afkebooks.com
diuresis that can severely dehydrate a patient. Administration of intravenous fluids (isotonic saline) is the initial management
in symptomatic patients, with the goal of replenishing intravascular volume and moderately decreasing calcium concentrations. A
reasonable regimen, in the absence of edema, is the administration of isotonic saline at an initial rate of 200 to 300 mL/hour that
is then adjusted to maintain the urine output at 100 to 150 mL/hour.
References
Objective number 12(Calcium/Phosphate Concentration Abnormal, Serum) of Objectives for the Qualifying Examination, 3rd
edition. Brock P, Cruz-Carreras MT. Emergency Complications of Malignancy. In: Tintinalli JE, Ma OJ, Yealy DM, et al., eds.
Tintinalli’s Emergency Medicine: A Comprehensive Study Guide. 9th ed. McGraw-Hill; 2020:(Ch) 240. Shane E, Berenson J.
Treatment of hypercalcemia. Post TW, ed. UpToDate. Waltham, MA: UpToDate Inc. Accessed April 08, 2022.
https://afkebooks.com
Question #24
A 50-year-old man consults a physician because of episodic weakness and paresthesias. On one occasion, he experienced
transient paralysis. The patient has also been experiencing polyuria and polydipsia. Vital signs demonstrate a blood pressure of
140/80 mmHg with normal temperature, pulse. and respirations. The remainder of the physical examination is unremarkable and
specifically shows no significant abnormal neurologic findings and no peripheral edema. Routine screening chemistry studies are
remarkable only for serum potassium of 2.1 mmol/L. CT scan demonstrates a small adrenal mass. Which of the following is the
most appropriate pharmacotherapy to treat this patient’s hypertension?
A) Captopril
B) Furosemide
C) Hydrochlorothiazide
D) Spironolactone
E) Propranolol
https://afkebooks.com
Explanation
Answer Description
Captopril is an angiotensin-converting enzyme (ACE) inhibitor that works well to reduce blood pressure in many patients by
blocking the renin-angiotensin system; however, it is less effective when there is already an excess of aldosterone present.
Furosemide, a loop diuretic, and hydrochlorothiazide , a thiazide type diuretic, would tend to make the patient’s hypokalemia
worse.
Propranolol is a beta-blocker that would reduce blood pressure but would not specifically counter the cause of this patient’s
hypertension.
https://afkebooks.com
Summarized Points:
The presence of hypertension and hypokalemia is consistent with primary hyperaldosteronism (conn's syndrome) Primary
hyperaldosteronism is commonly caused by adrenal hyperplasia. which is treated with an aldosterone blocking medication such
as spironolactone.
More Detail
Hypertension with an adrenal mass should bring to mind two distinct conditions:
• An epinephrine/norepinephrine-secreting Pheochromocytoma
• An aldosterone-secreting adrenocortical tumor (which
can produce Conn syndrome).
Although both of these can produce hypertension, other aspects of the clinical presentations are quite different.
Pheochromocytoma can produce either episodic severe hypertension or continuous hypertension. In either situation, electrolyte
studies are usually normal. In contrast, Conn syndrome (which this patient has) often shows prominent symptoms related to
hypokalemia, including weakness, paresthesias, and transient paralysis. The treatment of hypertension related to high
aldosterone levels is to block the aldosterone activity with the potassium-sparing diuretic. (and anti-aldosterone agent)
spironolactone.
References
Cruz DN, Perazella MA. Hypertension and hypokalemia: unusual syndromes. Conn Med. 1997 Feb. 61(2):67-75. Objective
number 9-1(Hypertension) of Objectives for the Qualifying Examination, 3rd edition
https://afkebooks.com
Question #25
A 55-year-old patient with a history of sarcoidosis with pulmonary fibrosis and hyperlipidemia is admitted for community-acquired
pneumonia. The day after admission, she suddenly develops confusion and hypotension. Her blood pressure is 80/45 mmHg,
and her pulse is 105 /min. Laboratory results are as follows:
She is immediately given 1 ampule of 50% dextrose and is started on intravenous fluids, but her blood pressure remains the
same after a 2 L infusion. What should be the next step?
A) Intravenous dobutamine
B) Intravenous dexamethasone
C) Intravenous methylprednisolone
D) Intravenous sodium bicarbonate
https://afkebooks.com
Explanation
Answer Description
Dobutamine is used in the treatment of heart failure and cardiogenic shock; it is not indicated in this situation. Vasopressors like
norepinephrine would be reasonable to give after dexamethasone therapy.
Methylprednisolone is indicated to prevent the adrenal crisis on the day of scheduled surgery.
Sodium bicarbonate is also used for hypotension refractory to fluid hydration. It increases cardiac contractility and is often used
in the treatment of hypotension induced by cyclic antidepressant overdose.
Summarized Points:
Adrenal Crisis is an acute and severely symptomatic stage of adrenal insufficiency that can include severe hypotension and
cardiovascular collapse, abdominal pain (can mimic an acute abdomen), acute renal failure, and death. Any stress (e.g., trauma,
infection, surgery) can precipitate an adrenal crisis. It can be fatal if untreated. Treat with Large volumes (1 to 3 liters) of 0.9
percent saline solution or 5 percent dextrose in 0.9 percent saline (if there is hypoglycemia)
• In a patient without a previous diagnosis of adrenal insufficiency, dexamethasone, which is not measured in cortisol
assays, should be used rather than hydrocortisone while biochemical testing is performed.
• For patients with a previously known diagnosis of adrenal insufficiency, IV hydrocortisone or any other
glucocorticoid preparation may be used because diagnostic testing is not necessary.
More Detail
Treatment of patients who present in possible adrenal crisis should not be delayed while diagnostic tests are performed. Blood
for serum cortisol, adrenocorticotropic hormone (ACTH), aldosterone, renin, and serum chemistry should be drawn and therapy
initiated immediately. The serum chemistry results guide initial treatment; the hormone values return later and are used to
confirm the diagnosis (cortisol and aldosterone) or evaluate the differential diagnosis (ACTH and renin).
Treatment includes 1 to 3 liters of 0.9 percent saline solution or 5 percent dextrose in 0.9 percent saline (to correct possible
hypoglycemia) should be infused intravenously (IV) within the first 12 to 24 hours based on assessment of volume status and
urine output. Hypotonic saline should not be used because it can worsen hyponatremia.
References
Objective number 9-2 (Hypotension, Shock) of Objectives for the Qualifying Examination, 3rd edition. Nieman LK. Treatment of
adrenal insufficiency in adults. Post TW, ed. UpToDate. Waltham, MA: UpToDate Inc. Accessed May 16, 2022.
Question #26
A 34-year-old woman comes to the office due to darkening and thickening of the skin over her neck and groin areas. The patient
otherwise feels well. Blood pressure is 130/80 mm Hg and pulse is 83/min. BMI is 31 kg/m2. Skin examination findings are
shown in the image. Similar lesions are found on the axilla and groin. The remainder of the physical examination is
unremarkable. This patient's condition is most likely associated with which of the following?
A) Addison disease
B) Gastric adenocarcinoma
C) Hemochromatosis
D) Polycystic ovary syndrome
E) Pellagra
Explanation
Answer Description
Hyperpigmentation in primary adrenal insufficiency (Addison disease) is generalized, although it may be more prominent in areas
of friction (e.g., elbows, knees) and sun exposure (e.g., face, dorsum of the hands). Other features of Addison's disease include
fatigue, dehydration, and hypotension, which are not seen in this patient.
Malignant AN is an uncommon condition associated with underlying tumor, notably of the gastrointestinal and genitourinary
systems. The abrupt emergence of such skin abnormalities in middle-aged or elderly persons is a sign of underlying cancer.
Diabetes mellitus and diffuse hyperpigmentation of the skin are features of hemochromatosis ("bronze diabetes"). Excessive
melanin and hemosiderin deposits cause generalized skin discoloration in hemochromatosis that is typically brownish or bronze
and at times slate gray.
Niacin deficiency (i.e., pellagra) typically presents with photosensitive dermatitis, erythematous tongue, diarrhea, and neurologic
symptoms (e.g., insomnia, dementia, confusion). AN has been reported with high-dose niacin therapy for dyslipidemia.
Summarized Points:
Acanthosis nigricans is characterized by symmetric, hyperpigmented, velvety plaques in the axillae, groin, and neck. It is
commonly associated with insulin resistance (e.g., diabetes mellitus, polycystic ovary syndrome, obesity) in younger patients
and, in rare cases, with gastrointestinal malignancy in older individuals.
More Detail
• Benign AN, typically seen in younger individuals, is associated with insulin resistance (e.g., diabetes mellitus,
obesity, polycystic ovary syndrome [PCOS]). Increased levels of insulin and/or insulin-like growth factors are thought to
stimulate epidermal and dermal proliferation. Similarly, skin tags (acrochordons), pedunculated outgrowths of normal
skin, are also commonly present on regions affected by AN.
• Malignant AN is a rare finding associated with underlying neoplasm, especially of the gastrointestinal and
genitourinary tracts. The sudden appearance of such skin changes in middle-aged or elderly patients is suggestive of
underlying malignancy.
PCOS typically occurs in obese women with menstrual irregularities (e.g., infertility), hyperandrogenism (e.g., acne,
hirsutism), and insulin resistance (e.g., AN). Patients with PCOS are at risk for type 2 diabetes mellitus and therefore require
glucose tolerance screening.
References
Objective number 97(Skin Rash / Papules) of Objectives for the Qualifying Examination, 3rd edition. Kutlubay Z, Engin B,
Bairamov O, Tüzün Y. Acanthosis nigricans: A fold (intertriginous) dermatosis. Clin Dermatol. 2015 Jul-Aug;33(4):466-70.
Question #27
A 31-year-old woman is evaluated for heat intolerance and palpitations. She has no past medical problems. Nine months ago,
she delivered a healthy boy via an uncomplicated vaginal delivery. A month ago, she was treated symptomatically for an upper
respiratory infection. Blood pressure is 144/70 mm Hg, and pulse is 110/min. Examination shows no proptosis. The thyroid gland
is diffusely enlarged without nodules or tenderness. Laboratory results are as follows:
Radioactive iodine uptake at 24 hours is 32% (10%-30%) with a diffuse pattern. Which of the following is the most likely
diagnosis?
Euthyroid sick syndrome is an abnormal pattern of thyroid markers seen in patients with severe acute illness. T3 is typically low,
free T4 and TSH are normal to low, and reverse T3 (inactive metabolite) is elevated. This patient has elevated T3 and T4 with an
overtly suppressed TSH level.
Silent thyroiditis and postpartum thyroiditis (<1 year of pregnancy) are variants of chronic lymphocytic (Hashimoto) thyroiditis that
can present with a transient hyperthyroid phase and diffuse goiter. However, hyperthyroidism in these disorders is due to the
destruction of the gland and the release of preformed thyroid hormone rather than the increased metabolic activity of the gland.
Radioiodine uptake is diminished rather than increased.
Subacute thyroiditis classically follows a viral syndrome and may have a hyperthyroid phase. However, it typically presents with
fever and a painful, tender goiter.
Summarized Points:
Graves’ disease is caused by antibodies to the TSH receptor. It typically presents with hyperthyroidism and diffuse goiter.
ophthalmopathy is present at the time of diagnosis in about 25% of cases. Radioiodine uptake is increased with a diffuse pattern.
More Detail
This patient has hyperthyroidism (elevated thyroid hormone levels, suppressed TSH) and a diffuse goiter consistent with
Graves’ disease. Graves’ disease is an autoimmune disorder caused by antibodies against the TSH receptor (thyroid-
stimulating immunoglobulin [TSI]) that cause increased metabolic activity of the thyroid, increased iodine uptake, and increased
production of thyroid hormone. About 25% of patients with Graves’ disease have ophthalmopathy (e.g., proptosis, periorbital
edema, irritation or discomfort in the orbit) at the time of diagnosis; ophthalmopathy in the setting of hyperthyroidism is virtually
diagnostic for Graves’ disease. For patients without ophthalmopathy, such as this patient, diffusely increased radioiodine
uptake can establish the diagnosis. For patients with suspected but unconfirmed Graves’ disease (e.g., indeterminate
radioiodine uptake, ophthalmopathy with normal thyroid function), assay of serum TSI is useful in diagnosis. Treatment options
include antithyroid drugs, radioiodine ablation, and surgical thyroidectomy.
References
Objective number 63(Neck Mass, Goiter, Thyroid Disease) of Objectives for the Qualifying Examination, 3rd edition. Kahaly GJ,
Bartalena L, Hegedüs L, et al. 2018 European Thyroid Association Guideline for the Management of Graves' Hyperthyroidism.
Eur Thyroid J 2018; 7:167.
Question #28
A 36 year old female presents to your office complaining of weakness, especially with activities that require muscular force, such
as climbing stairs. Her symptoms have developed gradually over the past year and she has largely ignored them. She reports a
recent weight gain of 11Kg (25 lb) over the past year and has been feeling melancholy for the past 5 months. She has also had
back pain for the past several months. Her medical history is significant for hypertension, and Diabetes. She takes metoprolol
and insulin. Physical examination reveals mild obesity, with fat deposition mainly around the trunk and the posterior neck. You
note some facial hair and scattered purple striae on the abdomen. Radiographs reveal a compressed fracture at the level of T11.
Her blood pressure is 140/85 mm Hg, pulse is 70/min. What would be the best initial test in this patient?
A) Serum ACTH
B) Magnetic resonance imaging of the brain
C) Computed tomography scan abdomen
D) 24-hour urine-free cortisol
E) Corticotropin-releasing hormone stimulation test
Explanation
Answer Description
Serum ACTH is not used as best initial test and is only helpful in determining the etiology location once the diagnosis of Cushing
syndrome has been made.
MRI brain and CT abdomen may help to localized pituitary adenoma and adrenal adenoma respectively but this test is premature
at this point as the diagnosis of Cushing has not yet been confirmed.
Corticotrophin releasing Hormone (CRH) stimulation test is only used once hypercortisolism has been established and ACTH is
elevated
Summarized Points:
Cushing syndrome presented with bone pain (osteoporosis), bruising, hirsutism, buffalo hump, central obesity, acne. The best
initial test to diagnose Cushing syndrome is either Low-dose dexamethasone or 24-hour urinary cortisol level. Both of them are
same, but 24-hour urinary cortisol level is gold standard.
More Detail
Cushing’s syndrome is a disorder caused by prolonged exposure to high levels of cortisol. Typical signs and symptoms of
Cushing’s syndrome include upper body obesity, a rounded face, striae on the skin of the abdomen, thighs, and breasts, thin skin
with easy bruising, weakened bones (Osteoporosis), excess body hair growth(Hirsutism), and menstrual irregularities in women,
and decreased fertility in men. The best initial test would be either low-dose dexamethasone or 24-hour urinary cortisol no
different. Among the choice, only 24-hour urinary cortisol level available.
References
Nieman LK, Biller BM, Findling JW, Newell-Price J, Savage MO, Stewart PM, et al. The diagnosis of Cushing's syndrome: An
Endocrine Society Clinical Practice Guideline. J Clin Endocrinol Metab. 2008 May. 93 (5):1526-40. Objective number 118-1
(weight gain, obesity) of Objectives for the Qualifying Examination, 3rd edition.
Question #29
Our patient's results are ready, His lab shows a High 24-hour urinary cortisol level. What would be the most appropriate next test
for this patient?
A) Serum ACTH
B) MRI brain
C) CT abdomen
D) Low-dose dexamethasone
E) CRH stimulation test
Explanation
Answer Description
In order to understand what imaging modality CT or MRI to use we need to order 2nd test either High-dose dexamethasone or
CRH-stimulating test.
Low-dose dexamethasone test is the best initial test to confirm Cushing syndrome. This has been done, it is not necessary to
repeat it.
First, we need to measure the ACTH level, if the ACTH level is high then. CRH-stimulating test is necessary.
Summarized Points:
After confirming the Cushing syndrome diagnosis, the next step is to order the ACTH level.
More Detail
Confirmation of Cushing syndrome diagnosis requires a high 24-hour urinary cortisol level. The next step is to determine the
location of the tumor or disease, which can be achieved by ordering an ACTH level.
Low ACTH levels indicate adrenal adenoma, while high levels are indicative of two conditions:
1. Cushing disease
2. Ectopic production
If the ACTH level is high, a second test is required to further localize tumors. The second test can be either a high-dose
dexamethasone test or a CRH-stimulating test, which are essentially the same.
References
Aymes S. Endocrine Society releases guidelines on the treatment of Cushing’s Syndrome. Endocrinology Advisor. Aug 26, 2015.
Question #30
A 14-year-old boy comes to the office because his breasts have recently become tender and slightly swollen. He is worried that
he is undergoing feminization and will grow up to become a “freak.” Upon examination a tender, 1.5-cm mass is found to be
palpable in the subareolar region of both breasts. Which of the following describes the best course of action?
if the mass is greater than 5 cm or does not regress by 16 to 17 years of age, a subcutaneous mastectomy should be performed.
Incision, aspiration, and topical steroids are not indicated under any circumstances.
Summarized Points:
Pubertal gynecomastia occurs in most adolescent boys, peaking at age 13-14 during Tanner stages Ill and VI. The breast tissue
is usually <2 cm in diameter and resolves spontaneously within a year.
More Detail
Gynecomastia is the presence of breast tissue in boys. Although it is often of concern to adolescents, it is usually benign and
physiologic in neonatal, pubertal, and elderly men. The trimodal distribution reflects periods of hormonal transition when
estrogen activity is increased and testosterone activity is decreased. Pubertal gynecomastia affects up to 70% of adolescent
boys, peaking at age 13-14 during Tanner stages III and IV. The presentation consists of unilateral or bilateral palpable breast
tissue extending <2 cm beyond the areola. Patients can expect spontaneous regression within a year. Because this patient
has no history of medication or illicit drug (e.g., anabolic steroids, marijuana) use and examination is otherwise normal,
reassurance should be provided. Further workup and treatment are not warranted.
References
Carlson HE. Approach to the patient with gynecomastia. J Clin Endocrinol Metab. 2011 Jan. 96(1):15-21. Objective number
10-3(Gynecomastia) of Objectives for the Qualifying Examination, 3rd edition
Question #31
A 66-year-old man without significant past medical history has decreased libido, decreased morning erections, and fatigue. His
serum testosterone level on repeated testing is consistently low and he requests testosterone replacement therapy. Before
initiating therapy, which of the following should be screened for in this patient?
Answer Description
TRT is associated with an increased risk of VTE. Patients should be evaluated with a careful medical and family history of VTE
prior to starting TRT but do not need routine screening for deep venous thrombosis.
TRT is associated with erythrocytosis, but not hemochromatosis or worsening of obstructive lung disease. Patients should have
a complete blood count prior to initiation of therapy but do not require specific screening for hemochromatosis.
Summarized Points:
Contraindications to testosterone replacement therapy include prostate or breast cancer, untreated severe sleep apnea, severe
lower urinary tract symptoms, hematocrit >50%, uncontrolled heart failure, and elevated prostate-specific antigen levels. All
patients should undergo prostate cancer screening prior to starting treatment.
More Detail
Testosterone replacement therapy (TRT) is indicated for men with clinical hypogonadism (e.g., decreased libido, fatigue,
reduced morning erections, loss of body hair, gynecomastia) and low testosterone levels (confirmed on 2 separate levels
drawn between 8 and10 AM). Contraindications for TRT are listed in the table. All patients should undergo prostate cancer
screening prior to starting TRT. TRT should not be initiated if the prostate-specific antigen level is >4.0 µg/L or >3.0 µg/L in
higher-risk individuals (e.g., African American, 1st-degree relative with prostate cancer). Patients with obstructive sleep apnea,
hematocrit >50%, or benign prostatic hyperplasia with severe lower urinary tract symptoms can be considered for TRT, but only
after these conditions are first treated and under control. Adverse effects of TRT include erythrocytosis, venous
thromboembolism (VTE), worsening of sleep apnea, and possibly an increased risk of cardiac events.
References
Objective number 46 (Infertility) of Objectives for the Qualifying Examination, 3rd edition. Bhasin S, Brito JP, Cunningham GR, et
al. Testosterone Therapy in Men With Hypogonadism: An Endocrine Society Clinical Practice Guideline. J Clin Endocrinol Metab
2018; 103:1715.
Question #32
A 12-year-old girl with type 1 diabetes mellitus is brought to the physician because of shortness of breath and fatigue for 1 day.
Since menarche began 4 months ago, she has had one episode of diabetic ketoacidosis per month; prior to that she had been
stable. Her blood pressure is 110/70 mmHg, pulse is 140/ min, and respirations are 36/min. physical examination reveals no
murmurs or extra heart sounds. Her lungs are clear to auscultation, and her abdominal examination is benign. She appears to be
moderately dehydrated. Laboratory studies show:
Serum
132 mmol/L
Na+
(135‐145)
Which of the following is the most likely underlying cause of this patient’s respiratory disorder?
A) Acute interstitial nephritis
B) Increased plasma ketone level
C) Diabetic nephropathy
D) Hyperinsulinemia
E) Acute renal failure
Explanation
Answer Description
Acute interstitial nephritis is an inflammatory process of the renal tubules presents with maculopapular rash, fever, and
eosinophilia.
Diabetic nephropathy is characterized by persistent albuminuria > 300 mg/day confirmed on at least 2 occasions taken 3 to 6
months apart.
Type 1 DM results from autoimmune destruction of pancreatic cells, often following a viral illness, which results in insulin
deficiency.
Summarized Points:
Type 1 diabetes mellitus associated with Diabetic Ketoacidosis(DKA). patient breath deep and shallow which called Kussmaul
breathing. It is physiological compensation response to metabolic acidosis caused by excessive ketoacid formation.
More Detail
This patient has diabetic ketoacidosis (DKA), which is typically seen in patients with type 1 diabetes mellitus. Her laboratory
data are significant for hyperglycemia (shown by the fact that she is spilling glucose in her urine), glycosuria, and ketonuria.
Respiration in DKA is Kussmaul pattern meaning it is rapid shallow breathing. It is the physiologic response to metabolic
acidosis caused by excessive ketone.
References
Agus MS, Wolfsdorf JI. Diabetic ketoacidosis in children. Pediatr Clin North Am. 2005;52(4):1147-1163. Objective number 37-2
(Diabetes Mellitus) of Objectives for the Qualifying Examination, 3rd edition.
Question #33
A 56-year-old man presents himself to a walk-in clinic complaining about tiredness, muscle weakness, polyuria, and nocturia.
The triage nurse finds his blood pressure is 220/125 mm Hg. Recognizing a potential medical emergency, she immediately
introduces him to the physician on duty. That physician treats him with nitroglycerine, which brings the readings down to 175/90
mm Hg. He also gives him a few extra nitroglycerine pills, plus prescriptions for hydrochlorothiazide and verapamil and tells him
to make an appointment with a physician who can treat him on a regular basis because he suspects that he may be suffering
from a serious condition. Taking his advice, the patient makes an appointment at a local free clinic. At this time, his sense of
muscle weakness has increased, as has his general feeling of fatigue. In addition to the symptoms above, he now also suffers
from muscle cramps and constipation. His new physician finds his blood pressure to be 190/95 mm Hg, and he also notes a mild
cardiac arrhythmia; blood serum analysis was normal except for his serum K+ level, which is 3.1 mm/L. The patient claims he
has been taking the pills prescribed by the doctor at the walk-in clinic on a regular basis. Which of the following conditions does
this patient most likely suffer from?
A) Hyperthyroidism
B) Conn syndrome
C) Addison disease
D) Cushing syndrome
E) Pheochromocytoma
Explanation
Answer Description
Hyperthyroidism can cause hypertension. Typical features include heat intolerance, difficulty concentrating, tremor, and weight
loss. This patient lacks clinical evidence of this disease.
Addison disease is caused by the underproduction of adrenal gland hormones, usually due to an autoimmune reaction.
Symptoms usually develop gradually and include low blood pressure, muscle weakness, bronzing of the skin, weight loss, and
fatigue.
Cushing’s syndrome is characterized by hypercortisolism, which can cause hypertension and hypokalemia. Typical features of
Cushing’s include central obesity, bruising, moon faces, and dorsal fat pad. This patient has none of these physical findings.
A pheochromocytoma should always be on the differential for patients with refractory hypertension. Typical features of
pheochromocytoma are paroxysmal headaches, flushing, and palpitations, none of which are experienced by this patient.
Summarized Points:
The presence of hypertension and hypokalemia is consistent with primary hyperaldosteronism (ie Conn’s syndrome) Primary
hyperaldosteronism is commonly caused by adrenal hyperplasia which is treated with an aldosterone blocking medication such
as spironolactone.
More Detail
Conn syndrome is an important example of a secondary hypertension, in this case caused by aldosteronism. Although some
studies suggest it is a rare condition, careful studies find it to be the cause of up to 15% of the cases of hypertension and should
be suspected in all cases resistant to treatment or accompanied by low serum K+ levels. It is caused by either an adenoma (a
nonmalignant aldosterone-producing tumor in one of the adrenals), or by bilateral adrenal hyperplasia. The former is readily
treated by surgery, while the latter generally may be treated with spironolactone, an aldosterone antagonist. The condition is
diagnosed by measuring serum aldosterone and renin levels.
References
Schirpenbach C, Reincke M (March 2007). "Primary aldosteronism: current knowledge and controversies in Conn's syndrome".
Nature Clinical Practice Endocrinology & Metabolism. 3 (3): 220–7. Objective number 9-1 (Hypertension) of Objectives for the
Qualifying Examination, 3rd edition.
Question #34
A 45-year-old man comes to the office for a follow-up visit. He was recently hospitalized for excessive urination and weight loss.
His blood glucose level in the emergency department was 32 mmol/L with a normal anion gap. His hemoglobin A1c was 12.1%.
The patient was not taking any medications before the hospitalization and had not seen a physician in more than 3 years. He
was initially treated with an insulin infusion and subsequently transitioned to a basal-bolus regimen of insulins glargine and
aspart. His blood glucose levels have been in the target range since discharge. The patient has a strong family history of type 2
diabetes. Examination is unremarkable except for BMI of 39 kg/m2 and neck acanthosis. The patient is very concerned about the
possibility of lifelong insulin therapy. Which of the following is the most appropriate advice for this patient?
A) Assay for anti-glutamic acid decarboxylase antibody will guide the need for continued insulin therapy
B) He cannot use oral anti-diabetic medication and most continue lifelong insulin treatment
C) He should consider going on an insulin pump
D) Once his glucose is well controlled, he may be able to manage his diabetes without insulin
E) Oral medications can be added, but he will likely always require insulin
Explanation
Answer Description
Autoantibodies such as anti-glutamic acid decarboxylase antibody can help differentiate type 1 from type 2 diabetes when the
clinical presentation is ambiguous. The patient clearly has type 2 diabetes mellitus as indicated by the absence of ketoacidosis
with high glucose levels, obesity, signs of insulin resistance, and family history.
Most patients can be switched to oral antidiabetic drugs once their insulin sensitivity and endogenous insulin production improve,
which usually takes a few weeks.
Insulin pumps are needed much less often in type 2 than in type 1 diabetes. The patient may be a candidate in the future if he
continues to require multiple insulin injections to control his diabetes.
Summarized Points:
Very high glucose levels can cause glucose toxicity, which is characterized by insulin resistance and impaired beta-cell function.
Short-term insulin therapy with tight glycemic control in patients with type 2 diabetes can reverse glucose toxicity and allow
transition to oral agents.
More Detail
Very high glucose levels have a detrimental effect on pancreatic beta-cell function and insulin sensitivity. This effect,
known as glucose toxicity, is common in patients with very high glucose levels (fasting >13.5 mmol/L, random >16.5mmol/L),
osmotic symptoms (e.g., polyuria, polydipsia), weight loss, ketosis, or hemoglobin A1c >9%. Glucose toxicity can be reversed by
maintaining tight glycemic control with insulin therapy. Although metformin is the agent of choice for most patients with type 2
diabetes, insulin is recommended for initial management in those with severe hyperglycemia. Once insulin sensitivity and
endogenous insulin production improve, usually within a few weeks, most patients can be transitioned to oral antidiabetic
medications (Choices B and E).
References
Objective number 37-2 (Diabetes Mellitus) of Objectives for the Qualifying Examination, 3rd edition. Campos C. Chronic
hyperglycemia and glucose toxicity: pathology and clinical sequelae. Postgrad Med. 2012 Nov;124(6):90-7.
Question #35
A 53-year-old woman is evaluated for abnormal thyroid function tests. Thyroid function testing was pursued because she has a
family history of Hashimoto thyroiditis. She has no symptoms of thyroid disease. She takes large doses of biotin, exceeding 300
mg/d, for its perceived health benefit, but no other drugs. On physical examination, vital signs are normal. The thyroid gland is
normal size without nodules. The remainder of the physical examination is normal. Laboratory studies show:
A) Start methimazole
B) Start propylthiouracil
C) Stop biotin; repeat thyroid tests
D) Refer for thyroidectomy
Explanation
Answer Description
Methimazole, propylthiouracil, and thyroidectomy are all treatments for Graves disease, which are inappropriate to consider until
abnormalities on a thyroid function test are confirmed. Thyroidectomy in Graves hyperthyroidism is most appropriate with large
goiter and compressive symptoms, moderate to severe Graves ophthalmopathy, and/or coexistent thyroid cancer or primary
hyperparathyroidism. Thyroidectomy is not the first-line therapy for most patients, and it is not indicated in this patient without a
confirmed diagnosis of Graves’ disease.
Summarized Points:
Patients taking 10 mg/d or more of biotin should discontinue ingestion 2 to 5 days before thyroid function testing.
More Detail
Biotin is a water-soluble vitamin commonly found in over-the-counter dietary supplements. High circulating levels of biotin
have been shown to interfere with laboratory assays that use streptavidin-biotin as an immobilizing system. Biotin interference
causes falsely high results with competitive immunoassays used to measure small molecules, including free thyroxine (T4),
free triiodothyronine (T3) , total To and total T3; it also causes falsely low results with sandwich assays used to measure large
molecules such as thyroid-stimulating hormone. This interference results in a testing profile that mimics thyrotoxicosis.
Biotin is a popular dietary supplement and is also used to treat multiple sclerosis and other neuromuscular disorders. Although
biotin is also found in plants, the richest sources are liver, egg yolk, soybeans, and yeast. An adequate intake for adults is
estimated to be 30 μg/d. Larger doses are commonly taken as an adjunctive medical treatment or for their perceived health
benefits. Most case reports of biotin interference with thyroid function assays document biotin doses of 300 mg/d or higher,
although interference with 10 mg/d has also been reported. Interference with thyroid assay results typically resolves within 48
hours, but expert opinion recommends testing after stopping biotin for several days. The most appropriate management for this
patient is to stop biotin and repeat thyroid function tests after 2 to 5 days of biotin discontinuation.
References
Objective number 63(Neck Mass, Goiter, Thyroid Disease) of Objectives for the Qualifying Examination, 3rd edition. Burch HB.
Drug Effects on the Thyroid. N Engl J Med. 2019 Aug 22;381(8):749-761. Ross DS. Laboratory assessment of thyroid function.
Post TW, ed. UpToDate. Waltham, MA: UpToDate Inc. Accessed June 14, 2022.
Question #36
A 39-year-old woman comes to the physician due to fatigue. She has a 6-month history of progressive fatigue associated with
weight gain, dry skin, and poor sleep. The patient's past medical history is unremarkable. She is married and has 2 children
under age 3. On examination, she has no thyroid enlargement or tenderness. Thyroid function tests are as follows:
In addition to initiating levothyroxine, which of the following is the most appropriate next step in the management of this patient?
Chronic central adrenal insufficiency may occur along with central hypothyroidism in patients with hypopituitarism. This patient
has an elevated TSH, suggesting normal pituitary function. Assessment of cortisol production is not necessary unless additional
clinical concerns arise.
Radioactive iodine uptake is used to evaluate patients with hyperthyroidism. Patients with increased synthesis of thyroid
hormones (e.g., Graves' disease) will have elevated uptake; those with the release of the preformed hormone will have
suppressed uptake. Radioactive iodine uptake is rarely useful in the evaluation of hypothyroidism.
Thyroid peroxidase antibody titers are increased in the vast majority of patients, although serologic testing has little impact on
management in patients with typical clinical features and is rarely required.
Thyroid ultrasound is indicated for evaluating patients with thyroid nodules. It is not useful for patients with hypothyroidism and
normal findings on examination.
Summarized Points:
Most patients with primary hypothyroidism have chronic autoimmune (Hashimoto) thyroiditis. Patients with typical hypothyroid
symptoms associated with an elevated TSH level and low free thyroxine can be treated with levothyroxine without additional
diagnostic evaluation.
More Detail
The correct answer is C.
This patient, with elevated TSH and low serum free thyroxine, has a typical presentation of primary hypothyroidism. Most
cases of primary hypothyroidism are autoimmune, and the incidence is significantly higher in women. Common syndromes
include chronic autoimmune (Hashimoto) thyroiditis, painless thyroiditis, and postpartum thyroiditis. Thyroid peroxidase antibody
titers are elevated in most patients, but serologic testing rarely affects management in patients with typical clinical features and is
not usually necessary (Choice 0). Some patients with hypothyroidism may have a transient hypothyroid phase followed by return
to a euthyroid state, but the majority will need lifelong supplementation with levothyroxine. Elderly patients or those with
comorbid heart disease should start treatment with low-dose levothyroxine (e.g., 25-50 µg/day) and gradually titrated;
younger healthy patients can start with higher doses (e.g., 75-100 µg/day). TSH should be rechecked at 6 weeks, and the
dose should be adjusted to keep serum TSH within the normal range. Some patients may continue to have nonspecific
symptoms (e.g., fatigue) despite adequate levothyroxine therapy. In such cases, the addition of liothyronine (T3) is not beneficial
and may lead to iatrogenic hyperthyroidism.
References
Objective number 33 (Fatigue) of Objectives for the Qualifying Examination, 3rd edition. Gaitonde DY, Rowley KD, Sweeney LB.
Hypothyroidism: an update. Am Fam Physician. 2012 Aug 1;86(3):244-51.
Question #37
Over the past 3 months, a 30-year-old woman has had intermittent episodes of headache, palpitations, sweating, and irritability.
Her blood pressure is 150/100 mm Hg while supine and 149/100 mm Hg while standing, and pulse is 90/min while supine and
110/min while standing. Examination shows no abnormalities except for pallor. Endocrine tests reveal results that are strongly
suggestive of a neuroendocrine tumor. Which of the following is the best next step in the management of this patient?
Answer Description
Choice B and E are not correct:
Removal of the tumor is performed only after localization, adequate preoperative control of blood pressure for 10-14 days with an
alpha-blocker, and intravascular fluid volume repletion with liberal fluid and salt. Intraoperative complications are much higher
with inadequate preoperative alpha blockade and intravascular fluid deficiency. Beta-blockers are given only to patients with
adequate and complete alpha blockade. Beta-blocking agents started before alpha blockade can lead to a paradoxical increase
in blood pressure.
Chromogranin A (CGA) is stored and released from dense-core secretory granules of neuroendocrine cells and is elevated in
nearly 80% of patients with pheochromocytoma. However, elevated CGA levels are not specific to pheochromocytoma and can
be elevated in other neuroendocrine tumors (eg, carcinoid) or in patients with mild renal insufficiency. CGA is sometimes used in
addition to measuring catecholamine and metanephrine levels in patients with possible drug interference with biochemical
testing.
If CT or MRI is negative in the presence of clinical and biochemical evidence of pheochromocytoma, one ought first to reconsider
the diagnosis. If it is still considered likely, then iodine-123 (123-I) metaiodobenzylguanidine (MIBG) scintigraphy may be done.
An MIBG scan can detect tumors not detected by CT or MRI, or multiple tumors when CT or MRI is positive.
Summarized Points:
Abdominal imaging (CT or MRI) is the next step for localizing a pheochromocytoma after biochemical confirmation of the tumor.
Patients with negative abdominal imaging usually require further testing, such as the metaiodobenzylguanidine scan. Removal of
the tumor is performed only after adequate preoperative control of blood pressure for 10-14 days with an alpha-blocker, and
intravascular fluid volume repletion with liberal fluid and salt. Beta-blockers are given only to patients with adequate and
complete alpha blockade.
More Detail
Abdominal imaging is the next step for localizing a pheochromocytoma after biochemical confirmation of the tumor. Both CT
and MRI are highly sensitive for localizing pheochromocytomas, which are usually located in the adrenal glands. Clinically
active pheochromocytomas are typically >4-4.5 cm in diameter and have a high tissue density on CT scan (generally 40-50
Hounsfield units). Larger tumors are more likely to have central necrosis and calcification. MRI usually shows high intensity on
T2-weighted images. Patients with negative abdominal imaging usually require further testing such as the
metaiodobenzylguanidine (MIBG) scan, which is functional scintigraphy with I123 labeled MIBG. MIBG resembles
norepinephrine, is taken up by adrenergic tissue, and can detect tumors not detected by CT or MRI. In addition, MIBG is also
performed in patients with large tumors (>5 cm diameter) as they have a higher risk of malignancy and possible extra-adrenal
disease. Younger patients and those with a familial disorder also require MIBG after initial CT or MRI as they have a higher risk
of extra-adrenal and multiple tumors.
References
Objective number 9-1(Hypertension) of Objectives for the Qualifying Examination, 3rd edition. Neumann HPH, Young WF Jr, Eng
C. Pheochromocytoma and Paraganglioma. N Engl J Med 2019; 381:552.
Question #38
A 37-year-old woman is evaluated for infertility. She and her male partner have been trying to get pregnant for the past 13
months without success, having had unprotected intercourse every 1 to 2 days. Her menses are irregular. Results of her
partner's semen analysis were normal. She has no previous pregnancies. Her partner fathered two children previously. Her only
medication is a prenatal vitamin. Physical examination findings are normal. Which of the following is the most appropriate next
step?
When evaluating infertility, male and female causes should be investigated concurrently. A semen analysis is the first step in the
evaluation of male infertility. Because semen analysis was normal for the patient's partner and he has fathered two children,
which points away from male causes of infertility, repeat semen analysis is not the correct choice. If an initial semen analysis is
abnormal, however, a repeat test is recommended to ensure collection after 2 to 3 days of sexual abstinence (longer timeframes
can result in decreased sperm motility). If repeat results are abnormal, referral to a male fertility specialist is recommended.
If anovulatory cycles are suspected, further evaluation should include measurement of prolactin, thyroid-stimulating hormone,
and follicle-stimulating hormone, and assessment for polycystic ovary syndrome. In this patient, a thyroid-stimulating hormone
level would be indicated if a midluteal phase serum progesterone is low.
The frequency of intercourse is important to discuss with all patients because the chance of pregnancy increases with the
frequency of intercourse and is highest with intercourse every 1 to 2 days. In this case, the couple has had unprotected
intercourse for more than 12 months with the appropriate frequency, so an evaluation is appropriate now. Recommending an
additional 12 months of intercourse should not be substituted for an evaluation of infertility.
Summarized Points:
Evaluation for causes of infertility is recommended for patients unable to achieve pregnancy after 12 months of consistent,
unprotected intercourse; in women older than 35 years, evaluation of fertility is reasonable after 6 months of infertility . The first
step in the workup of female infertility associated with normal menstrual cycles is to obtain a midluteal phase serum
progesterone level.
More Detail
References
Objective number 46(Infertility) of Objectives for the Qualifying Examination, 3rd edition. Fluker M. Ovulation. CFAS Consensus
Document for the Investigation of Infertility By First Line Physicians. CFAS Clinical Practice Guidelines. Canadian Fertility &
Andrology Society. August 2002 Kuohung W, Hornstein MD. Evaluation of female infertility. Post TW, ed. UpToDate. Waltham,
MA: UpToDate Inc. Accessed March 01, 2022.
Question #39
A 16-year-old boy comes to the physician for a routine health maintenance visit concerned about breast enlargement. Over the
past few months, he has noticed his left breast getting larger but has no pain. He smokes marijuana daily and occasionally
snorts amphetamines at weekend parties. The patient is a B student and on the wrestling team at school. He Is at the 50th
percentile for weight and height. Blood pressure is 115/68 mm Hg and pulse is 68/min. The patient has enlarged, tender,
palpable left breast tissue extending about 1.5 cm beyond the areola with no abnormalities of the right chest. He has Tanner
stage V male genitalia and copious axillary hair. No lymphadenopathy is palpated in the cervical, axillary, and groin regions.
Testicular volume is 22 mL. Which of the following is the most likely cause of this patient's breast enlargement?
A) Amphetamine abuse
B) Marijuana abuse
C) Breast malignancy
D) Chromosomal abnormality
E) Normal pubertal changes
Explanation
Answer Description
Although chronic amphetamine abuse may cause gynecomastia, it is less likely given this patient's history of infrequent use.
Despite this patient's relevant family history, breast malignancy is unlikely given that it is much less common in men than women.
It typically presents with axillary lymphadenopathy; non-tender, firm breast mass; and skin retraction or ulceration overlying the
cancerous lesion.
Although 47,XXY Klinefelter syndrome is associated with gynecomastia and breast cancer, it is not a likely diagnosis in this
patient given the absence of antisocial behaviors, autistic traits, tat stature, and atrophic testes.
Gynecomastia associated with normal pubertal changes typically occurs around age 14 during Tanner stages III and IV.
Therefore, this patient's gynecomastia is likely pathologic with his history of drug use and Tanner stage V on examination.
Summarized Points:
Adolescents who abuse marijuana, alcohol, amphetamines, heroin, or methadone should be aware that gynecomastia may
develop along with other serious psychosocial and medical problems. Treatment of drug-induced gynecomastia includes
discontinuation of the offending drug.
More Detail
References
Objective number 10-3 (gynecomastia) of Objectives for the Qualifying Examination, 3rd edition. Deepinder F, Braunstein GD.
Drug-induced gynecomastia: an evidence-based review. Expert Opin Drug Saf 2012; 11:779.
Question #40
A 61-year-old woman comes to the physician for a preventive visit. She inquires about the need for bone mineral density testing.
The patient is an active smoker and has smoked half a pack of cigarettes daily for the last 40 years. Her past medical history is
significant for high cholesterol, for which she is taking atorvastatin. Her current body weight is 57 kg. She lives alone, consumes
alcohol socially, and follows a healthy diet. Which of the following is the strongest additional risk factor for osteoporosis in this
patient?
High sodium intake is associated with increased urinary calcium excretion, but studies to date have not found a clear association
between sodium intake and osteoporosis.
Hyperlipidemia and osteoporosis have been linked in case reports of certain rare gene mutations. Hyperlipidemia is otherwise
not a significant risk factor for osteoporosis.
Heavy alcohol intake is associated with an increased risk of osteoporosis. The impact of lesser degrees of alcohol intake is
unclear, but some studies have reported a decreased risk of osteoporosis in patients with moderate alcohol consumption.
Summarized Points:
Bone mineral density (BMD) testing is recommended for individuals over the age of 65, and also for menopausal women and
men aged 50-64 with clinical risk factors for fracture, including fragility fracture, prolonged use of glucocorticoids or other high-
risk medications, parental hip fracture, vertebral fracture or osteopenia identified on X-ray, current smoking, high alcohol intake,
low body weight or major weight loss, rheumatoid arthritis, and other disorders strongly associated with osteoporosis.
More Detail
Bone mineral density (BMD) testing is recommended for individuals, including men and women over the age of 65. However,
for menopausal women and men aged between 50-64 years, BMD testing should be considered earlier if they have clinical
risk factors for fracture, such as:
• Fragility fracture after age 40
• Prolonged use of glucocorticoids, and use of other high-risk medications such as aromatase inhibitors or androgen
deprivation therapy.
• Parental hip fracture
• Vertebral fracture or osteopenia identified on x-ray
• Current smoking
• High alcohol intake
• Low body weight (<60 kg), major weight loss (>10% of weight at age 25 years)
• Rheumatoid arthritis, and other disorders strongly associated with osteoporosis such as primary hyperparathyroidism,
type 1 diabetes mellitus, osteogenesis imperfecta, uncontrolled hyperthyroidism, hypogonadism or premature
menopause (<45 years), cushing’s disease, chronic malnutrition or malabsorption, chronic liver disease, chronic
obstructive pulmonary disease (COPD), and chronic inflammatory conditions (e.g., Inflammatory bowel disease).
The risk of osteoporotic fracture may be estimated using the fracture risk assessment tool (FRAX) which categorizes patients
into low, medium, and high-risk groups. It is used to determine the appropriate treatment for each patient based on their
10-year fracture risk. This patient has several risk factors (e.g., current smoker, low body weight) that increase her risk for
osteoporotic fracture and make her an eligible candidate for an earlier bone mineral density test.
References
Objective number 67-1-2-4 (Local Pain, Spinal Compression/Osteoporosis) of Objectives for the Qualifying Examination, 3rd
edition. Papaioannou A, Morin S et al.; Scientific Advisory Council of Osteoporosis Canada. 2010 clinical practice guidelines for
the diagnosis and management of osteoporosis in Canada. Appendix 1: Background materials for 2010 clinical practice
guidelines for the diagnosis and management of osteoporosis in Canada. CMAJ. 2010;182(17):1864–1873. 213.
Question #1
A 65-year-old woman comes to the emergency department because of severe wrist pain after a fall. She says that she fell on an
outstretched hand 2 days ago, and has been experiencing severe pain ever since. She has had 3 episodes of nephrolithiasis,
chronic muscle weakness, and vague abdominal complaints over the past few months. Physical examination is normal. An x-ray
film of her wrist shows a fracture of the distal radius and osteopenia. Laboratory studies show:
A splint is placed on her wrist and a follow-up visit is scheduled. Which of the following is the most likely explanation for these
findings?
Decreased excretion of uric acid is the most common cause of hyperuricemia and gout. It can be caused by increased tubular
absorption, decreased tubular secretion, or decreased glomerular filtration. These individuals typically have a uric acid level
greater than 420 μmol/L. Complications include gouty arthritis, nephrolithiasis, renal insufficiency, and renal failure.
The development of thyroid-stimulating hormone receptor antibodies is the underlying mechanism of Graves’ disease, which is
one of the most common causes of hyperthyroidism. The signs and symptoms include a
diffuse, toxic goiter, tachycardia, tremor, nervousness, and exophthalmos. This patient does not fit this description.
An elevation of circulating aldosterone is a sign of Conn’s disease, which is caused by adrenal hyperplasia or an adenoma.
These patients typically have hypertension and hypokalemia. This patient has neither of these findings.
An elevation of circulating cortisol is a sign of Cushing syndrome, which may be caused by a pituitary adenoma (Cushing
disease), an adrenal cortical tumor, ectopic ACTH production (a lung tumor), or glucocorticoid therapy. Signs and symptoms
include central obesity, moon facies, muscle wasting, osteoporosis, striae, bruising, hypertension, menstrual irregularities,
hirsutism, and acne. This patient has osteopenia, but none of the other signs or symptoms.
Summarized Points:
Hypercalcemia due to primary hyperparathyroidism results in renal stones, osteopenia, and constipation, and may mimic
dementia. Remember the mnemonic stones, groans, moans, and bones.
More Detail
This patient has hyperparathyroidism, which is characterized by an elevation of circulating parathyroid hormone (PTH), most
Likely due to a benign adenoma. An elevation of PTH leads to high calcium and low phosphate levels. These patients often
have unbalanced bone resorption and formation leading to osteopenia and fractures, recurrent nephrolithiasis, vague
abdominal symptoms, weakness, and easy fatigability. “Bones, stones, abdominal groans, and moans” is a mnemonic used to
remember the symptoms. The treatment is typically surgical.
References
Sharata A, Kelly TL, Rozenfeld Y, et al. Management of Primary Hyperparathyroidism: Can We Do Better?. Am Surg. 2017 Jan
1. 83 (1):64-70. Objective number 12-2(Calcium Disorder) of Objectives for the Qualifying Examination, 3rd edition.
Question #2
A 67-year-old man comes to the office to establish care. His medications include rosuvastatin for hypercholesterolemia and
amlodipine for hypertension. He has gained 4.5 kg (10 lb) over the last 3 years since he quit smoking. His BMI is 31 kg/m2. The
patient's fasting blood glucose is 6.5 mmol/L (3.3-5.8) and hemoglobin A1c, is 6.3%. His fasting blood glucose was normal 3
years ago. Which of the following is the most appropriate intervention to prevent progression to overt type 2 diabetes mellitus in
this patient?
Answer Description
ACE inhibitors (e.g., ramipril) are recommended as first-line antihypertensive agents in patients with diabetes as they reduce the
risk of diabetic nephropathy. However, they do not reduce the incidence of diabetes.
Statins can modestly increase the risk of type 2 diabetes. Nonetheless, statins should not be stopped in prediabetes as these
patients have an increased risk of coronary heart disease, and the reduction in coronary events with statins significantly
outweighs their diabetogenic potential.
Intensive lifestyle intervention, including diet, exercise, and weight loss, reduces the risk of diabetes. The Mediterranean diet is
also associated with a reduction in this risk. A low-fat diet alone, however, has not been found to reduce the risk of diabetes.
High vitamin D levels are statistically associated with a lower incidence of diabetes. However, vitamin D supplementation has not
been shown to reduce diabetes risk, and vitamin D levels may simply reflect favorable lifestyle factors (i.e., balanced diet and
outdoor exercise).
Summarized Points:
Patients with elevated fasting glucose and hemoglobin A1c, levels are at increased risk for progression to overt diabetes mellitus.
Interventions that can reduce the risk of diabetes include intensive lifestyle modification (diet, exercise, and weight loss) and
metformin.
More Detail
This patient has fasting glucose and hemoglobin A1c levels that are elevated but do not meet diagnostic criteria for
diabetes. This condition, often termed prediabetes, is associated with an increased risk of progression to overt diabetes mellitus
and long-term diabetic complications. Several randomized, controlled trials have shown that intensive lifestyle intervention,
including a healthy diet with regular exercise to reduce body weight by 5%-10%, is effective in preventing progression to type 2
diabetes. In addition, metformin therapy is associated with a reduced risk of progression to diabetes. Metformin is safe and
inexpensive and is also associated with modest weight loss in many patients. Other oral agents have been considered for
diabetes prevention but are not currently recommended as they are ineffective, more expensive, and/or have more significant
side effects.
References
Objective number 37-2 (Diabetes Mellitus) of Objectives for the Qualifying Examination, 3rd edition. American Diabetes
Association. 3. Prevention or Delay of Type 2 Diabetes: Standards of Medical Care in Diabetes-2021. Diabetes Care 2021;
44:S34.
Question #6
A 58-year-old woman is evaluated for persistent fatigue. A year ago, she developed panhypopituitarism following resection of a
large nonfunctional pituitary tumor. Current treatment includes hydrocortisone and levothyroxine, with her most recent dose
adjustment a few months ago. The patient has had no associated headaches, excessive urination, visual disturbances, or weight
change. Vital signs are normal. BMI is 26 kg/m2 with no change in weight over the last 6 months. Physical examination is
unremarkable. Laboratory results are as follows:
An MRI of the pituitary 6 months ago showed no residual pituitary tumor. Which of the following is the most appropriate next step
in the management of this patient's disorder?
T3 is produced primarily by the deiodination of T4 in peripheral tissues. Serum T3 levels (total and free) correlate poorly with
overall thyroid physiologic status and are less reliable than T4, although they are occasionally used in the evaluation of
hyperthyroidism in patients with normal T4 levels, in those with alterations in thyroid-binding proteins, or in those with the
euthyroid sick syndrome. T3 has a shorter half-life than T4, and liothyronine (T3) therapy can lead to fluctuations in physiologic
status and increased risk of thyrotoxicosis. Studies have not shown the superiority of combined replacement over levothyroxine
(T4) alone.
Disruption of other pituitary hormones is common in central hypothyroidism. However, insulin-like growth factor 1 levels (e.g., as
used in the assessment of growth hormone deficiency) can be low in a hypothyroid state, and adequate levothyroxine dosing
should be confirmed before measuring insulin-like growth factor 1.
A low (i.e., suppressed) TSH in primary hypothyroidism indicates over-replacement and warrants a dose reduction. However, this
relationship does not hold in central hypothyroidism.
Summarized Points:
Levothyroxine therapy in central hypothyroidism should be based on serum free T4 levels, with the dose adjusted to maintain T4
in the high-normal range. TSH in central hypothyroidism demonstrates minimal responsiveness to levothyroxine and may not rise
in patients with inadequate replacement.
More Detail
References
Objective number 63(Neck Mass, Goiter, Thyroid Disease) of Objectives for the Qualifying Examination, 3rd edition. Persani L,
Brabant G, Dattani M, et al. 2018 European Thyroid Association (ETA) Guidelines on the Diagnosis and Management of Central
Hypothyroidism. Eur Thyroid J 2018; 7:225.
Question #8
A 52-year-old woman comes to the office due to a 3-day history of pain in the anterior lower neck radiating to the jaw. The pain is
exacerbated by turning her head. The patient has also had palpitations and a feeling of being "hot" She experienced an upper
respiratory infection 4 weeks ago, but otherwise her medical history is unremarkable. Temperature is 37.5 C (99.5 F), blood
pressure is 144/80 mm Hg, pulse is 125/min and regular, and respirations are 18/min. Due to neck pain, the patient does not
allow neck palpation. The oropharynx is not erythematous. Laboratory results are as follows:
A) Riedel thyroiditis
B) Thyroid abscess
C) Subacute thyroiditis
D) Thyroid hemorrhage
E) Thyroid lymphoma
https://afkebooks.com
Explanation
Answer Description
Riedel thyroiditis is characterized by chronic fibrosis of the thyroid and surrounding tissues. It presents with slowly progressive
thyroid enlargement and compressive symptoms (e.g., hoarseness, dysphagia). The goiter in Riedel thyroiditis is painless, hard,
and fixed to the surrounding tissue. Most patients are euthyroid.
Suppurative infection of the thyroid (infectious thyroiditis) is a rare condition that presents with high-grade fever and thyroid pain.
The thyroid may be enlarged due to abscess formation, but patients are usually euthyroid, as the involvement of the gland is
focal.
Bleeding into cystic lesions of the thyroid can cause acute pain and tenderness, but patients do not have fever or thyrotoxicosis
as in this patient.
Thyroid lymphoma presents with painless enlargement of the thyroid, often leading to compression of neighboring tissues. It is
usually seen in patients with long-standing Hashimoto thyroiditis (due to lymphocytic infiltration of the gland) and thus most
patients have a history of chronic hypothyroidism.
Summarized Points:
Subacute thyroiditis is characterized by fever, neck pain, and a tender goiter, typically following a viral illness. Patients have a
self-limited thyrotoxic phase followed by hypothyroidism and eventual recovery of thyroid function. Treatment is symptomatic with
beta-blockers and nonsteroidal anti-inflammatory drugs.
https://afkebooks.com
More Detail
This patient has subacute (de Quervain, subacute granulomatous) thyroiditis with fever, severe neck/thyroid pain,
thyrotoxicosis (elevated free thyroxine, suppressed TSH), and an elevated erythrocyte sedimentation rate. Subacute
thyroiditis is thought to be due to a postviral inflammatory process and often follows an upper respiratory illness.
Thyrotoxicosis in subacute thyroiditis is due to the release of the preformed thyroid hormone rather than increased thyroid
activity, and radioiodine uptake is low. Subacute thyroiditis is a self-limited condition and resolves within a few weeks. Patients
often have a transient hypothyroid phase followed by a return to a euthyroid state. Treatment includes beta-blockers to control
thyrotoxic symptoms and nonsteroidal anti-inflammatory drugs (NSAIDs) for pain relief. Glucocorticoids are used for severe
pain not responding to NSAIDs.
References
Objective number 63(Neck Mass, Goiter, Thyroid Disease) of Objectives for the Qualifying Examination, 3rd edition. Samuels
MH. Subacute, silent, and postpartum thyroiditis. Med Clin North Am. 2012 Mar;96(2):223-33.
https://afkebooks.com
Question #9
A 37-year-old-woman is evaluated for a 9-month history of oligomenorrhea, a deepening voice, and increased body hair. Her last
menses was 3 months ago. Medical history is otherwise unremarkable, and she takes no medications. On physical examination,
vital signs are normal. She has frontal hair loss and coarse dark hairs on her chin and chest. Larynx appears larger than normal.
Clitoromegaly is present on pelvic examination. Laboratory studies show:
Which of the following is the most appropriate diagnostic test to perform next?
A) Pituitary MRI
B) Abdominal CT
C) Adrenal vein sampling
D) Ovarian vein sampling
E) Pelvic ultrasonography
https://afkebooks.com
Explanation
Answer Description
Although virilization can be seen with Cushing's disease from pituitary overproduction of adrenocorticotropic hormone, it would
be very unlikely to see this level of DHEAS elevation and no other signs of Cushing's disease. Therefore, a pituitary MRI is not
indicated in this patient.
Adrenal vein sampling to localize the tumour is rarely required; regardless, it would not be the next diagnostic test.
Ovarian vein sampling is rarely indicated; it is reserved for hyperandrogenism in women who are pre-menopausal and wish to
preserve fertility and in whom an ovarian tumour is suspected but imaging results are normal. Women who are postmenopausal
with ovarian hyperandrogenism, whether presumed secondary to an ovarian tumour or ovarian hyperthecosis, can forego this
invasive procedure and proceed directly to bilateral oophorectomy.
Pelvic ultrasonography is recommended as the first imaging study if testosterone is greater than 5.2 nmol/L, which indicates that
an ovarian source of hyperandrogenism is likely. This patient's markedly elevated DHEAS level and mildly elevated testosterone
level make a testosterone-producing ovarian tumour less likely than an adrenal tumour. Pelvic MRI may be more sensitive at
detecting small ovarian tumours and is often considered as second-line imaging when pelvic ultrasound is negative.
Summarized Points:
For new-onset hyperandrogenism, ovarian and adrenal sources of androgen excess should be considered. A markedly elevated
dehydroepiandrosterone sulfate level in a patient with hyperandrogenism is indicative of an adrenal source.
https://afkebooks.com
More Detail
This patient is premenopausal and has new-onset hyperandrogenism; therefore, both ovarian and adrenal sources of
androgen excess should be considered. This combination of a markedly elevated serum dehydroepiandrosterone sulfate
(DHEAS) level and mildly elevated serum testosterone level suggests an adrenal source, given that the adrenal glands are
the major source of DHEAS. An abdominal CT is recommended when the serum DHEAS value is greater than 19.0 μmol/L).
Androgen-producing adrenal tumours are rare and lead to menstrual irregularities and virilization in women, including hirsutism,
voice-deepening, increased muscle mass, increased libido, and clitoromegaly. Marked elevations in DHEAS are specific to the
adrenal glands, and DHEAS-secreting tumours of the adrenal gland are usually readily visible on CT.
References
Objective number 43 (Hirsutism, Hypertrichosis) of Objectives for the Qualifying Examination, 3rd edition. Lizneva D, Gavrilova-
Jordan L, Walker W, Azziz R. Androgen excess: Investigations and management. Best Pract Res Clin Obstet Gynaecol. 2016
Nov;37:98-118. Barbieri RL, Ehrmann DA. Evaluation of premenopausal women with hirsutism. Post TW, ed. UpToDate.
Waltham, MA: UpToDate Inc. Accessed March 08, 2021.
https://afkebooks.com
Question #10
A 37-year-old woman comes to the physician because of muscle weakness and cramps for 2 weeks. She has been taking a ß-
adrenergic blocking agent for hypertension for 2 years. she also complains of headaches and having to urinate more often than
usual. She has past medial history of chronic lymphocytic thyroiditis (Hashimoto’s disease). Her blood pressure is 160/108 mm
Hg, and pulse is 60/min. Serum studies show:
Magnetic resonance angiography of the abdomen shows normal findings. Which of the following locations is affected in this
patient?
A) Adrenal cortex
B) Adrenal medulla
C) Aorta
D) Renal arteries
E) Thyroid gland
https://afkebooks.com
Explanation
Answer Description
Hypertension, hypokalemia and metabolic alkalosis all point to primary hyperaldosteronism. Pheochromocytoma is
catecholamine secreting tumor from the Adrenal medulla.
https://afkebooks.com
Choice D is not correct:
Hypertension, hypokalemia and metabolic alkalosis all point to primary hyperaldosteronism. Renal artery stenosis has high
Renin.
Summarized Points:
Primary hyperaldosteronism or Conn’s disease is Aldosterone secreting tumor from the adrenal cortex. It is manifested by
Hypertension, headache, polyuria, and muscle weakness. Triad of Hypertension, Hypokalemia and metabolic alkalosis always
point toward primary hyperaldosteronism.
More Detail
Excessive secretion of aldosterone from the adrenal cortex leads to Conn’s syndrome or primary hyperaldosteronism. It is
usually due to adrenocortical hyperplasia (70%) but can also result from unilateral adrenal adenoma (Conn syndrome).
Conn’s syndrome, or primary hyperaldosteronism manifested by Hypertension, headache, polyuria, and muscle weakness. Lab
in primary hyperaldosteronism shows hypokalemia, hypernatremia, low plasma renin, and increased plasma aldosterone.
• If the ratio is elevated, the next step is ordering a confirmatory test Saline loading test (Oral or IV).
◦ Saline loading test normal response is suppression of Aldosterone, if aldosterone failed to suppress diagnosis is
primary hyperaldosteronism.
Treatment
https://afkebooks.com
• Unilateral adenoma is resected by laparoscopy.
• Bilateral hyperplasia is treated with eplerenone or spironolactone.
References
Objective number 9-1 (Hypertension) of Objectives for the Qualifying Examination, 3rd edition. Kasper, D., et al (eds.), Harrison’s
Principles of Internal Medicine, 19th ed., McGraw-Hill, 2015.
https://afkebooks.com
Question #11
A 39-year-old woman comes to the office due to palpitations and fatigue over the last 2 months. She delivered a healthy child 9
months ago. The patient gained excessive weight during her pregnancy and has been trying to lose it. Blood pressure is 140/80
mm Hg and pulse is 100/min. Her BMI is 32 kg/m2. The thyroid gland is nontender and not enlarged. Laboratory results are as
follows:
https://afkebooks.com
Explanation
Answer Description
Chronic autoimmune (Hashimoto's) thyroiditis causes hypothyroidism with elevated TSH and low T4. A small percentage of
patients can have a transient hyperthyroid phase, but they have suppressed TSH with elevated T4. This patient has normal T4
and elevated T3.
https://afkebooks.com
Choice C is not correct:
Patients with Graves' disease may not have thyromegaly or extrathyroidal manifestations. However, Graves' disease can be
ruled out in this patient based on serum thyroglobulin and radioactive iodine uptake, which are elevated in those with Graves'
disease.
In most patients, postpartum painless thyroiditis occurs within 1-6 months after delivery. The radioactive iodine uptake is low, but
serum thyroglobulin is elevated.
Patients with subacute (de Quervain's) thyroiditis have fever, along with moderate to severe pain and tenderness in the thyroid
gland. Both T3 and T4 are typically elevated.
Summarized Points:
Factitious thyrotoxicosis causes hyperthyroid symptoms associated with elevated T3, suppressed TSH, and low radioactive
iodine uptake and thyroglobulin.
More Detail
This patient has thyrotoxicosis that is most likely due to exogenous thyroid supplements. Obese patients often attempt to
lose weight by taking dietary supplements that may contain high amounts of T3. Patients have typical hyperthyroid symptoms
(e.g., weight loss, palpitations), but there is usually no goiter as the exogenous thyroid hormones inhibit TSH secretion, leading
to thyroid atrophy. Patients may have lid lag but do not have exophthalmos as seen in Graves' disease.
Laboratory studies in factitious thyrotoxicosis usually show high T3, low serum thyroglobulin levels, and suppressed TSH.
Radioactive iodine uptake scan shows <1% uptake (due to suppressed TSH). Serum thyroglobulin is low in exogenous
thyroid hormone-induced thyrotoxicosis but is normal to elevate all other causes of thyrotoxicosis.
https://afkebooks.com
References
Objective number 63(Neck Mass, Goiter, Thyroid Disease) of Objectives for the Qualifying Examination, 3rd edition. Ross DS.
Exogenous hyperthyroidism. Post TW, ed. UpToDate. UpToDate Inc. Accessed August 13, 2021.
https://afkebooks.com
Question #12
A 46-year-old woman is diagnosed with hyperparathyroidism and is scheduled for surgery. While awaiting surgery she rapidly
develops muscular weakness, nausea, vomiting, and confusion and is brought to the emergency department. Which of the
following is the most appropriate immediate management of her current condition?
A) Intravenous calcitonin
B) Intravenous hydration with normal saline
C) Intravenous mithramycin
D) Intravenous potassium
E) Surgical consultation for immediate parathyroid exploration
https://afkebooks.com
Explanation
Answer Description
When the calcium level remains elevated, hypocalcemic agents should be used, such as calcitonin. Calcitonin given
intravenously or subcutaneously has rapid effect in reversing hypercalcemia.
Intravenous mithramycin inhibits bone resorption and can effectively lower the serum calcium concentration within 24 hours of
administration. It has potential hematologic, renal, and hepatic toxicity and hence, it is used only in palliation of malignancy or in
cases in which conventional therapy for hypercalcemia is ineffective or contraindicated.
Intravenous potassium is not indicated except in hypokalemia that presents with muscle weakness.
Parathyroid exploration with resection of the offending gland represents the definitive treatment of hypercalcemia, but it should
be delayed until the serum calcium level is reduced to a safer level.
Summarized Points:
Isotonic saline (4-5 L) over 24 h ± loop diuretic (e.g. furosemide) but only if hypervolemic (urine output >200mL/h).
More Detail
Hypercalcemic crisis occurs when the serum calcium level rises to 3.5 mmol/L or greater. Symptoms of hypercalcemic crisis
https://afkebooks.com
include weakness, nausea, vomiting, and confusion, which, if untreated, may lead to coma. Hypercalcemic crisis is a medical
emergency. Initial management should be designed to lower the serum calcium level acutely. Lowering of the serum calcium
level is best accomplished by vigorous intravenous saline infusion to expand the intravascular volume, followed by
administration of loop diuretic, such as furosemide, to increase the urinary calcium excretion. Hypercalcemia can cause cardiac
arrhythmias and hence, cardiac monitoring is required, especially when the calcium level is higher than 4 mmol/L.
References
Agus AZ. Etiology of hypercalcemia. Rose BD (editor). Waltham: Up To Date. 2010. Objective number 12(Calcium/Phosphate
Concentration Abnormal, Serum) of Objectives for the Qualifying Examination, 3rd edition.
https://afkebooks.com
Question #13
A 43-year-old woman comes to the office due to irregular menstrual periods and intermittent milky discharge from both breasts
for the past 6 months. During this time, she has also had vaginal dryness and dyspareunia. The patient has had no headaches,
visual loss, weight changes, or hirsutism. She has had no prior medical problems and takes no medications except for over-the-
counter vitamin supplements. Blood pressure is 112/70 mm Hg and pulse is 66/min. BMI is 21 kg/m2. Physical examination
shows no abnormalities except mild expressive galactorrhea from both breasts. Laboratory results are as follows:
Serum chemistry is within normal limits. MRI of the brain shows a 7-mm pituitary mass. The patient is very reluctant to start any
treatment. Which of the following is the patient at an increased risk of developing if her current condition is left untreated?
A) Tricuspid regurgitation
B) Vision loss
C) Osteoporosis
D) Endometrial cancer
E) Panhypopituitarism
https://afkebooks.com
Explanation
Answer Description
Patients treated with higher doses of cabergoline (≥2 mg/week: e.g., as used for Parkinson's disease) have an increased risk for
valvular heart disease and should be monitored with an echocardiogram every 2 years. This risk is not seen with the lower doses
used for prolactinoma and is not an effect of untreated hyperprolactinemia.
Most microprolactinomas do not grow substantially over time and do not usually cause compression of surrounding tissues.
Compressive effects (eg, panhypopituitarism, visual field defects) are more likely with macroprolactinomas, which are a more
common presentation of hyperprolactinemia in men and postmenopausal women.
Endometrial cancer is usually caused by unopposed high estrogen rather than the low estrogen seen in this patient.
Summarized Points:
Hyperprolactinemia suppresses gonadotropin secretion and leads to hypogonadism, which leads to low estrogen levels with
bone loss and osteoporosis in women. Microprolactinomas are generally slow growing with low risk of rapid enlargement.
More Detail
Treatment of lactotroph pituitary adenomas (prolactinomas) is dependent on tumor size and metabolic complications. A
macroprolactinoma (>10 mm) typically requires treatment, but patients with a microprolactinoma (<10 mm) are usually
treated only if they have specific indications. Hyperprolactinemia suppresses gonadotropin secretion and causes
hypogonadism in premenopausal women. The resultant low estrogen level can lead to infertility, amenorrhea or
https://afkebooks.com
oligomenorrhea, vaginal dryness with dyspareunia, and hot flashes. Low estrogen levels are also a risk factor for rapid bone
loss and osteoporosis.
This patient has a microprolactinoma with significant hyperprolactinemia. Her vaginal symptoms suggest low estrogen levels.
She should therefore receive treatment with a dopamine agonist (e.g., cabergoline), which can lower prolactin production
and mitigate hypoestrogenic complications.
References
Objective number 67-1-2-4 (Local Pain, Spinal Compression/Osteoporosis) of Objectives for the Qualifying Examination, 3rd
edition. Lake MG, Krook LS, Cruz SV. Pituitary adenomas: an overview. Am Fam Physician. 2013 Sep 1;88(5):319-27. Snyder
PJ. Clinical manifestations and evaluation of hyperprolactinemia. Post TW, ed. UpToDate. Waltham, MA: UpToDate Inc.
Accessed February 09, 2022.
https://afkebooks.com
Question #14
A 36-year-old female presents to your office complaining of weakness, especially with activities that require muscular force, such
as climbing stairs. Her symptoms have developed gradually over the past year and she has largely ignored them. She reports a
recent weight gain of 11Kg (25 lb) over the past year and has been feeling melancholy for the past 5 months. She has also had
back pain for the past several months. Her medical history is significant for hypertension, and Diabetes. She takes metoprolol
and insulin. Physical examination reveals mild obesity, with fat deposition mainly around the trunk and the posterior neck. You
note some facial hair and scattered purple striae on the abdomen. Radiographs reveal a compressed fracture at the level of T11.
Her blood pressure is 140/85 mm Hg, pulse is 70/min. What would be the best initial test in this patient?
A) Serum ACTH
B) Magnetic resonance imaging of the brain
C) Computed tomography scan abdomen
D) 24-hour urine-free cortisol
E) Corticotropin-releasing hormone stimulation test
https://afkebooks.com
Explanation
https://afkebooks.com
https://afkebooks.com
Answer Description
Serum ACTH is not used as best initial test and is only helpful in determining the etiology location once the diagnosis of Cushing
syndrome has been made.
MRI brain and CT abdomen may help to localized pituitary adenoma and adrenal adenoma respectively but this test is premature
at this point as the diagnosis of Cushing has not yet been confirmed.
Corticotrophin releasing Hormone (CRH) stimulation test is only used once hypercortisolism has been established and ACTH is
elevated
Summarized Points:
Cushing syndrome presented with bone pain (osteoporosis), bruising, hirsutism, buffalo hump, central obesity, acne. The best
initial test to diagnose Cushing syndrome is either Low-dose dexamethasone or 24-hour urinary cortisol level. Both of them are
same, but 24-hour urinary cortisol level is the gold standard.
More Detail
Cushing’s syndrome is a disorder caused by prolonged exposure to high levels of cortisol. Typical signs and symptoms of
Cushing’s syndrome include upper body obesity, a rounded face, striae on the skin of the abdomen, thighs, and breasts, thin skin
with easy bruising, weakened bones (Osteoporosis), excess body hair growth(Hirsutism) and menstrual irregularities in women,
and decreased fertility in men.
Base on above table, the best initial test would be either low-dose dexamethasone or 24-hour urinary cortisol no different.
https://afkebooks.com
Among choice, only 24-hour urinary cortisol level available.
References
Nieman LK, Biller BM, Findling JW, Newell-Price J, Savage MO, Stewart PM, et al. The diagnosis of Cushing's syndrome: an
Endocrine Society Clinical Practice Guideline. J Clin Endocrinol Metab. 2008 May. 93 (5):1526-40. Objective number 118-1
(weight gain, obesity) of Objectives for the Qualifying Examination, 3rd edition.
https://afkebooks.com
Question #15
Our patient results ready, His lab shows High 24-hour urinary cortisol level. What would be the most appropriate next test in this
patient?
A) Serum ACTH
B) Magnetic resonance imaging of the brain
C) Computed tomography scan abdomen
D) Low-dose dexamethasone
E) Corticotropin-releasing hormone stimulation test
https://afkebooks.com
Explanation
https://afkebooks.com
https://afkebooks.com
Answer Description
In order to understand what imaging modality CT or MRI to use we need to order 2nd test either High-dose dexamethasone or
CRH-stimulating test.
Low-dose dexamethasone test is the best initial test to confirm Cushing syndrome. This have been done, it is not necessary to
repeat it.
First we need to measure ACTH level, if ACTH level is high then. CRH-stimulating test is necessary.
Summarized Points:
After confirming Cushing syndrome diagnosis, next step is to order ACTH level.
More Detail
High 24-hour urinary cortisol level confirms Cushing syndrome diagnosis. Next step is locating tumor or disease for that we
need to order ACTH level.
If ACTH level is High, 2nd test need to be ordered to further localized tumors. 2nd test is either High-dose dexamethasone or
CRH-stimulating test, both of them are same.
https://afkebooks.com
References
Aymes S. Endocrine Society releases guidelines on treatment of Cushing’s Syndrome. Endocrinology Advisor. Aug 26, 2015.
https://afkebooks.com
Question #17
A 63-year-old woman is evaluated after an abdominal CT scan for diverticulitis revealed an incidental adrenal mass. She has no
other medical problems and takes no medications. On physical examination, vital signs are normal. The remainder of the
examination is unremarkable. Abdominal CT scan shows a 3.5-cm left adrenal mass with a density of 7 Hounsfield units and
absolute contrast washout of 80% at 10 minutes. The remainder of the scan is normal. Serum creatinine and electrolytes are
normal. Testing for mild autonomous cortisol excess is negative. Which of the following is the most appropriate next step in
management?
A) Adrenal biopsy
B) Adrenalectomy
C) Repeat abdominal CT at 12 months
D) Screening for primary aldosteronism
Explanation
Answer Description
Adrenal biopsy has a limited role in the evaluation of incidentalomas and is reserved for lesions suspicious for metastases or an
infiltrative process such as lymphoma or infection. This patient has no indication for an adrenal biopsy.
Adrenalectomy should be considered for patients with a functioning pheochromocytoma, aldosterone-producing tumour, or
hypercortisolism or a suspicious imaging phenotype for adrenal carcinoma. Elements of a suspicious imaging phenotype include
size larger than 4 cm, Hounsfield units 10 or higher (indicative of low lipid content), and absolute contrast washout of 60% or less
at 10 minutes. Adrenalectomy would also be appropriate for masses that demonstrate growth of more than 1 cm/year. This
patient meets none of these criteria.
The patient does not require screening for primary aldosteronism because she does not have hypertension or hypokalemia. Only
patients with an incidental adrenal mass and these features require screening for primary hyperaldosteronism.
Summarized Points:
Asymptomatic patients with incidentally discovered adrenal mass with no evidence of mild autonomous cortisol excess and
benign imaging phenotype can undergo repeat imaging at 12 months; clinical observation can also be considered. In patients
with an adrenal mass, screening for primary hyperaldosteronism is indicated in the presence of hypertension or hypokalemia.
More Detail
Biochemical testing for hypercortisolism should be undertaken in all patients with an incidentally discovered adrenal mass,
even in the absence of typical symptoms; screening for pheochromocytoma is indicated if the unenhanced CT attenuation is
greater than 10 Hounsfield units, even in the absence of hypertension. Most benign adrenal adenomas are smaller than 4 cm
and have high lipid content, which corresponds to a density of less than 10 Hounsfield units, and contrast washout is greater
than 60% in 10 minutes. This asymptomatic patient with an incidentally discovered adrenal mass has no evidence of hormone
excess associated with mild autonomous cortisol excess and has a benign imaging phenotype. Clinical observation can
also be considered. A large study of patients with small nonfunctioning adrenal tumours showed significant growth in only
2.5% of tumours at 50 months, and no cases of adrenal carcinoma were diagnosed. Clinically overt hormone excess developed
in less than 0.1% of patients.
References
Objective number 2 (Abdominal / Pelvic Mass) of Objectives for the Qualifying Examination, 3rd edition. Elhassan YS, Alahdab F,
Prete A, et al. Natural History of Adrenal Incidentalomas With and Without Mild Autonomous Cortisol Excess: A Systematic
Review and Meta-analysis. Ann Intern Med. 2019 Jul 16;171(2):107-116. Young WF, Kebebew E. Evaluation and management of
the adrenal incidentaloma. Post TW, ed. UpToDate. Waltham, MA: UpToDate Inc. Accessed September 01, 2020.
Question #18
A 67-year-old female presents to your office with a 1-month history of intermittent neck and shoulder pain with shortness of
breath that normally occurs when she does chores around the house or climbs stairs. She is previously healthy and denies any
medical conditions. She does not smoke or drink alcohol. The patient is up to date on vaccinations and had a mammogram done
at her previous appointment 1 year ago that was normal. Her Blood pressure is 148/86, pulse is 88/min, respiratory rate is 15/
min, temperature is 37 C. She had a colonoscopy performed 6 years ago that was normal. You order an ECG, chest x-ray,
complete blood count, and electrolytes. What should be recommended to the patient at her visit today?
A) Mammogram
B) Colonoscopy
C) Calcium and phosphorus levels
D) Dual-energy x-ray absorptiometry (DEXA)
E) CT scan of the chest
Explanation
Answer Description
The patient had a normal mammogram the year before. Mammograms should be performed every 2 years in her age group.
The patient had a colonoscopy 6 years ago that was normal. She is due for another colonoscopy in 4 years (reaching the
10-year mark after her previous one).
Calcium and phosphorus levels are normal in patients with osteoporosis and have no value in screening for the condition.
There is no history or clinical suspicion here for lung cancer, so a CT scan of the chest is not warranted.
Summarized Points:
Recommend a one-time screening for osteoporosis in all women aged 65 years or older with DEXA scan of the spine and
hips.
More Detail
Risk factors for osteoporosis include smoking, family history, low body weight, excessive alcohol use, and secondary organic
causes such as premature menopause, among others. Regardless of symptoms, recommend a one-time screening for
osteoporosis in all women aged 65 years or older with Dual-energy x-ray absorptiometry (DEXA) scan of the spine and hips. A
bone density with T-score < 2.5 standard deviations below the mean is associated with osteoporosis and a T-score between 1
and 2.5 standard deviations below the mean is associated with osteopenia.
References
Dawson-Hughes B, Gold DT, Rodbard HW, et al. Physician’s guide to prevention and treatment of osteoporosis. Washington:
National Osteoporosis Foundation, 2003. Objective number 67-1-2-4 (Local Pain, Spinal Compression/Osteoporosis) of
Objectives for the Qualifying Examination, 3rd edition.
Question #19
A 62-year-old man with advanced renal disease due to diabetic nephropathy has an estimated glomerular filtration rate of 15 mL/
min, serum calcium of 2.3 mmol/L (2.18-2.58), and phosphate of 1.6 mmol/L (0.8-1.5). Which of the following is most likely to be
elevated in this patient?
A) Parathyroid hormone
B) Calcitriol
C) Ionized calcium
D) Serum iron
E) Thyroid-stimulating hormone
Explanation
Answer Description
Calcitriol (1,25 dihydroxy vitamin D) synthesis is decreased in CKD, resulting in lower calcium absorption through the intestinal
tract.
In chronic kidney disease patients, increased serum phosphate binds to circulating calcium, forming insoluble calcium phosphate
and further lowering serum calcium.
CKD causes anemia due to decreased renal erythropoietin production. Serum iron may be normal or low (if patients also develop
iron deficiency).
Patients with CKD can have slightly lower T4 and T3 levels due to impaired peripheral conversion (T4 to T3) and decreased
binding to carder proteins. However, TSH is usually not elevated.
Summarized Points:
Secondary hyperparathyroidism in chronic renal failure is characterized by hypo- or normocalcemia, hyperphosphatemia, and
increased parathyroid hormone levels.
More Detail
This patient, with hyperphosphatemia in the setting of chronic kidney disease (CKD), has a typical presentation of secondary
hyperparathyroidism (SHPT). In CKD, there is decreased production of calcitriol (1,25 dihydroxy vitamin D), leading to
decreased intestinal absorption of calcium. Concurrently, as the glomerular filtration rate (GFR) decreases, the kidney cannot
adequately excrete phosphate, leading to hyperphosphatemia. The increased serum phosphate binds to circulating calcium,
forming insoluble calcium phosphate and further lowering serum calcium. Hypocalcemia and hyperphosphatemia stimulate
the release of parathyroid hormone (PTH) to maintain normal calcium and phosphorus levels. The magnitude of PTH elevation
usually correlates with the severity of the renal failure. In mild to moderate CKD (GFR 20-60 mL/min/1.73 m2), higher PTH
levels may be adequate to normalize serum calcium and phosphorus levels. However, in advanced CKD (GFR <20 mL/min/1.73
m2), more significant phosphate retention and hypocalcemia may develop. Management includes a low-phosphate diet and/or
phosphate binders (e.g., calcium carbonate, calcium acetate, sevelamer) and supplementation of calcium and vitamin D.
References
Objective number 89-2 (Chronic kidney injury) of Objectives for the Qualifying Examination, 3rd edition. Jamal SA, Miller PD.
Secondary and tertiary hyperparathyroidism. J Clin Densitom. 2013 Jan-Mar;16(1):64-8.
Question #20
A 45-year-old man is evaluated for hypertension. He has no symptoms except for mild fatigue. He does not use tobacco, alcohol,
or illicit drugs. The patient's blood pressure is 160/94 mm Hg. His laboratory results were as following:
CT of the abdomen shows a 2-cm, round right adrenal mass. He declines surgery for removal of the mass. Which of the
following is the best initial therapy for this patient?
A) Chlorthalidone
B) Clonidine
C) Hydralazine
D) Ramipril
E) Spironolactone
Explanation
Answer Description
Some patients may have persistent hypertension despite aldosterone antagonist therapy. Additional antihypertensives such as
thiazide diuretics (e.g., chlorthalidone) or angiotensin-converting-enzyme inhibitors (e.g., ramipril) are usually recommended for
blood pressure control in these patients. However, they are not the best initial therapy.
Clonidine and hydralazine, as well as calcium channel blockers such as verapamil, are not generally recommended for medical
therapy in primary hyperaldosteronism. They may occasionally be considered for patients with persistent hypertension despite
appropriate first-line treatment.
Summarized Points:
Primary hyperaldosteronism is usually due to an adrenal adenoma or bilateral adrenal hyperplasia. Surgery is preferred for
unilateral adrenal adenoma. Medical therapy with aldosterone antagonists (e.g., spironolactone, eplerenone) is recommended
for patients with bilateral adrenal hyperplasia or with unilateral adrenal adenoma who either refuse surgery or are poor surgical
candidates.
More Detail
This patient's presentation of hypertension, hypokalemia and CT scan showing an adrenal mass is consistent with primary
hyperaldosteronism. The condition is usually due to an adrenal adenoma or bilateral adrenal hyperplasia. Diagnosis is
suggested by an elevated plasma aldosterone level, with plasma aldosterone to plasma renin activity ratio >20 (i.e.,
aldosterone is elevated independent of renin, and the elevated aldosterone suppresses renin secretion through feedback
inhibition). Measurement of urine aldosterone after an oral saline load can confirm the diagnosis. In normal patients, a
sodium load will suppress aldosterone secretion, but in primary aldosteronism, the aldosterone will remain elevated.
Adrenal imaging with CT scan and adrenal venous sampling can usually distinguish between unilateral adrenal adenoma and
bilateral adrenal hyperplasia. Surgery is preferred for unilateral adrenal adenoma. Medical therapy is recommended for
patients with bilateral adrenal hyperplasia or those with unilateral adrenal adenoma who refuse surgery or are poor
surgical candidates. Spironolactone is an aldosterone antagonist and is the preferred initial medical therapy in most
patients. However, spironolactone is also a progesterone and androgen receptor antagonist that can cause significant side
effects in both men (e.g., decreased libido, gynecomastia) and women (e.g., breast tenderness, menstrual irregularities).
Eplerenone is a selective mineralocorticoid antagonist with a very low affinity for progesterone and androgen receptors and is an
alternate therapy for patients who develop side effects on spironolactone.
References
Objective number 9-1(Hypertension) of Objectives for the Qualifying Examination, 3rd edition. Parthasarathy HK, Ménard J. A
double-blind, randomized study comparing the antihypertensive effect of eplerenone and spironolactone in patients with
hypertension and evidence of primary aldosteronism. J Hypertens. 2011 May;29(5):980-90.
Question #21
A 26-year-old woman visits her physician because she has been experiencing acute attacks of headache, profuse sweating, and
racing heartbeat during the past 2 months. She also has bouts of nausea, abdominal pain, and dyspnea. She is being treated
with clonidine for hypertension without any response. Her blood pressure is 160/80 mm Hg and her heart rate is normal.
Laboratory studies show:
Blood Chemistry
Sodium 140 mmol/L (135-145)
Potassium 4.4 mmol/L (3.5-5.0)
Chloride 99 mmol/L (98-106)
Bicarbonate 30 mmol/L (24-30)
Blood urea nitrogen 3.57 mmol/L (2.5-8.0)
Creatinine 35.5 µmol/L (50-90)
Serum glucose 9.5 mmol/L (3.3-5.08)
Thyroid-stimulating
3 µU/mL (0.4-5.0)
hormone
A) Anxiety disorder
B) Clonidine withdrawal
C) Hyperthyroidism
D) Pheochromocytoma
E) Primary hyperaldosteronism
Explanation
Answer Description
Pheochromocytoma may be mistaken for an anxiety disorder such as a panic attack given common physical symptoms of racing
heart, sweating and nervousness, headache, muscle tension, chest pain, and abdominal distress. However, patients with
pheochromocytoma more often than not present with hypertension (90%) which may be sustained or paroxysmal in nature.
Clonidine withdrawal syndrome can result from the abrupt withdrawal or tapering of clonidine causing a hyperadrenergic state
that mimics pheochromocytoma. The syndrome consists of nausea, palpitation, anxiety, sweating, and headache, along with an
elevation in blood pressure. In this clinical scenario, the patient has remained on clonidine despite a poor response.
Tachycardia, heat intolerance, weight loss, and anxiety are also features of hyperthyroidism. However, in this clinical scenario,
serum thyroxine and thyroid-stimulating hormone levels are within the normal range and therefore rule out the diagnosis of
hyperthyroidism.
Patients with primary hyperaldosteronism typically have other physical and biochemical findings such as hypokalemia (the
patient in this clinical scenario has normal potassium) and alkalosis.
Summarized Points:
Pheochromocytoma should be suspected in patients with episodes of paroxysmal headache, diaphoresis, and palpitations.
More Detail
The correct answer is D.
References
Thompson LD. Pheochromocytoma of the Adrenal gland Scaled Score (PASS) to separate benign from malignant neoplasms: a
clinicopathologic and immunophenotypic study of 100 cases. Am J Surg Pathol. 2002 May. 26(5):551-66. Objective number
9-1(Hypertension) of Objectives for the Qualifying Examination, 3rd edition
Question #23
A 57-year-old man with chronic obstructive pulmonary disease undergoes a total hip arthroplasty for avascular necrosis of the
femoral head. On the second postoperative day, he has diffuse, profound weakness and vomiting. His blood pressure is 85/50
mmHg, and pulse is 100/min. The operative site is clean and dry, with minimal output from the drains. Hemoglobin level is 138 g/
L, serum sodium level is 132 mmol/L, serum potassium level is 5.8 mmol/L, creatinine 76 µmol/L. Which of the following is most
likely to confirm the diagnosis?
TSH is the best initial test to diagnose thyroid disease not Addison disease.
Direct Coombs test is a screening test used to detect IgG antibodies bound to the surface of RBCs, it has nothing to do with
Addison crisis.
Ventilation/perfusion scanning is a useful tool in diagnosing pulmonary thromboembolism. Mismatched perfusion defect is
characteristic.
Echocardiography is useful diagnostic tool to assess left ventricular ejection fraction (LVEF), left ventricular size. ventricular
compliance, wall thickness, and valve function.
Summarized Points:
Developing weakness, nausea, vomiting, abdominal pain and hypotension after precipitating factors like major surgery, infection,
trauma point to Addison disease. First treat patient with IV normal saline and hydrocortisone and order for electrolyte and Arterial
blood gas. To confirm diagnosis order ACTH stimulating test (AKA Cosyntropin).
More Detail
Diagnosis:
References
Objective number 37-1 (Glucose Abnormalities) of Objectives for the Qualifying Examination, 3rd edition. Husebye ES, Allolio B,
Arlt W, et al. Consensus statement on the diagnosis, treatment, and follow-up of patients with primary adrenal insufficiency. J
Intern Med. 2014;275(2):104-115.
Question #24
A 35-year-old man comes to your office complaining of weight loss, anxiety, hot flashes, and diarrhea. He also reports that he is
unable to fall asleep at night and wakes early in the morning. He works as a nurse at the local community hospital. Her
temperature is 37.0°C (98.6°F), blood pressure is 110/70 mm Hg, pulse is 110/min, and respirations are 15/min. He appears
anxious and jittery throughout the interview. Physical examination shows warm, moist skin; a diffusely enlarged thyroid gland;
and a slight decrease in his peripheral vision bilaterally, His reflexes are hyperactive but symmetric. Suspecting hyperthyroidism,
you order the following labs, with the following results:
Factitious disorders are an important cause of illness, and the Medical Council of Canada Qualifying Examination (MCCQE) Part
1 likes to ask questions about them. A case with multiple laboratory inconsistencies, especially if the patient is a health care
worker, might make you suspect a factitious disorder. In this case, ingestion of levothyroxine would lead to elevated thyroid
hormone levels but would suppress TSH. Remember that this is a diagnosis of exclusion.
Antithyroglobulin and antithyroid peroxidase antibodies are helpful in confirming a diagnosis of Hashimoto thyroiditis
(hypothyroidism). They can also be elevated in Graves disease, and are elevated in a small percentage of euthyroid patients as
well.
TSH receptor antibodies are positive in approximately 80% of patients with Graves disease. They are a reasonable test to order
if you suspect Graves disease, but a negative test certainly would not exclude Graves as your diagnosis.
Radioactive uptake scans can be part of the workup for Graves disease because they not only help confirm the diagnosis by
showing a diffuse increase in uptake but later are useful to determine the dose of radioactive iodine that would be necessary if
the patient were to opt for radioactive ablation as the form of therapy. In the present patient, you would likely see a diffuse uptake
of radioactive iodine because the gland is actively producing excess thyroid hormone. Radioactive uptake scans would not help
in ruling out a pituitary etiology.
Summarized Points:
The scenario of a thyroid-stimulating hormone level unaffected by negative feedback by elevated triiodothyronine and thyroxine
that is in a hyperthyroid patient and accompanied by visual problems is most likely caused by a pituitary adenoma. The best next
step is to do an MRI of the brain.
More Detail
MRI of the pituitary gland will likely reveal an adenoma that is likely secreting thyroid-stimulating hormone (TSH). This patient is
obviously hyperthyroid by history. She reports anxiety, weight loss, difficulty sleeping, hot flashes, and diarrhea. Her exam
reveals a nervous woman with a goiter, moist skin, tachycardia, and hyperreflexia. She lacks the exophthalmos seen in Graves
disease. The "mild decrease in peripheral vision" suggests a pituitary mass. Her thyroid function tests reveal inappropriate
secretion of TSH by the pituitary despite elevated triiodothyronine (T3) and thyroxine (T4). This suggests that the pituitary gland
is functioning autonomously without the proper feedback from T3 and T4. Normally, high levels of thyroid hormone would be
expected to suppress TSH. It is important to recognize that the TSH in this patient could have been within normal range but in
the presence of elevated thyroid hormone this would be inappropriate as well. An MRI of the pituitary gland will probably reveal a
macroadenoma.
References
Objective number 63 (Neck Mass, Goiter, Thyroid Disease) of Objectives for the Qualifying Examination, 3rd edition. Ónnestam
L, Berinder K, Burman P, et al. National incidence and prevalence of TSH-secreting pituitary adenomas in Sweden. J Clin
Endocrinol Metab 2013; 98:626.
Question #25
The patient has a serum calcium level of 3 mmol/L and an elevated PTH level. The surgeon suspected hyperparathyroidism
before surgery. Which imaging modality is helpful in the differentiation of hyperplasia from adenoma?
Answer Description
Painless, noninvasive, inexpensive, and does not expose the patient to radiation;
Summarized Points:
99m
TC Sestamibi scan able to localize a solitary adenoma from parathyroid hyperplasia.
More Detail
The most common cause of primary hyperparathyroidism is parathyroid adenoma (80%), hyperplasia (20%), and carcinoma
(<1%).
In order to differentiate, order 99mTC Sestamibi scan which localizes a solitary adenoma. Definitive therapy for
hyperparathyroidism is surgery (parathyroidectomy).
References
Bliss RD, Gauger PG, Delbridge LW. Surgeon's approach to the thyroid gland: surgical anatomy and the importance of
technique. World J Surg. 2000;24(8):891-897. Fraser WD. Hyperparathyroidism. Lancet. 2009;374(9684):145-158.
Question #26
A 59-year-old man is evaluated for headaches, pallor, and panic attacks for the past year. His long-term hypertension has also
become harder to control. Medications are chlorthalidone, lisinopril, metoprolol, and diltiazem. On physical examination, blood
pressure is 164/98 mm Hg. The remainder of the examination is normal. Which of the following is the most appropriate
diagnostic test?
A) Adrenal CT
B) Adrenal venous catecholamine sampling
C) Plasma free metanephrine measurement
D) Iodine 123-metaiodobenzylguanidine scan
Explanation
Answer Description
The search for a tumour should begin when a biochemical diagnosis of pheochromocytoma/paraganglioma is supported by
laboratory results, to avoid misdiagnosing an incidental nonfunctioning adrenal mass as a pheochromocytoma. If biochemical
testing supports the diagnosis of pheochromocytoma, an adrenal CT scan or MRI of the abdomen should be performed. Imaging
is not an initial diagnostic test.
Adrenal venous catecholamine sampling should not be performed because it may result in inappropriate adrenalectomy. Healthy
individuals show a significant difference in catecholamine concentration in the right versus left adrenal vein; therefore, adrenal
vein sampling for catecholamines has no diagnostic value.
The average size of a symptomatic pheochromocytoma at diagnosis is 4 cm. If the CT is negative, reconsidering the diagnosis is
the first step; however, if suspicion of a catecholamine-secreting tumour is high, the next step is an iodine
123-metaiodobenzylguanidine scan. This test may also be indicated in patients with very large pheochromocytomas (>10 cm) to
detect metastatic disease or paragangliomas to detect multiple tumours. This scan is not an initial diagnostic test.
Summarized Points:
In the context of a high index of suspicion for pheochromocytoma, plasma free metanephrine is an appropriate screening test,
whereas urine fractionated metanephrine and catecholamines may be a better option for cases with low suspicion. Imaging for
pheochromocytoma should be performed only after documentation of elevated catecholamine levels.
More Detail
Elevation in catecholamines can occur in patients under psychological or physical stress. Certain medications can affect results
and should be discontinued at least 2 weeks before testing. Mild elevations may require repeat testing. Levels more than four
times the upper limit of normal, in absence of acute stress or illness, are consistent with a catecholamine-secreting tumor.
The plasma free metanephrine test is highly sensitive (96%-100%). The specificity is 85% to 89%. Urine fractionated
metanephrine and catecholamines have higher specificity (98%) and high sensitivity (up to 97%). Neither test is superior, so
clinicians can use an estimate of pretest probability to select the initial test. In the context of a high index of suspicion, plasma
free metanephrine is chosen whereas urine fractionated metanephrine and catecholamines may be a better option for
cases with low suspicion.
References
Objective number 9-1(Hypertension) of Objectives for the Qualifying Examination, 3rd edition. Neumann HPH, Young WF Jr, Eng
C. Pheochromocytoma and Paraganglioma. N Engl J Med. 2019 Aug 8;381(6):552-565. Young WF Jr. Clinical presentation and
diagnosis of pheochromocytoma. Post TW, ed. UpToDate. Waltham, MA: UpToDate Inc. Accessed May 02, 2022.
Question #27
A 45-year-old man is concerned about developing diabetes. He has gained 6.8 kg (15 lb) over the last 5 years. The patient's
medical history is unremarkable and he takes no medications. His father suffers from type 2 diabetes mellitus. BMI is 32 kg/m2.
Laboratory evaluation shows a fasting plasma glucose of 7 mmol/L (3.3-5.8) and a hemoglobin A1c of 6%. Which of the following
is the best management for this patient?
Answer Description
A home glucose monitor is useful for assessing possible hypoglycemic symptoms and adjusting the dose of insulin therapy.
However, the diagnosis of diabetes should not be made by finger-stick glucose testing.
An oral glucose tolerance test is very sensitive for diagnosing diabetes and is frequently ordered to screen for gestational
diabetes. However, it is time-consuming and expensive, and simply repeating the FPG should confirm the diagnosis in this
patient.
If there are no clear signs or symptoms of hyperglycemia, diabetes should not be diagnosed just on a single abnormal reading,
but rather on the results of two separate tests or a repeat measurement of the same test on a different day.
Summarized Points:
If the results from 2 different tests for diabetes mellitus are discordant, the test with a value above the diagnostic threshold
should be repeated
More Detail
Diabetes mellitus is most commonly diagnosed based on either fasting plasma glucose (FPG) ≥7 mmol/L or hemoglobin A1c
(6.5%); nonfasting glucose of 11.1 mmol/L is also diagnostic if it is associated with symptoms of hyperglycemia (e.g., polydipsia,
polyuria) or occurs during an oral glucose tolerance test. If there are no unequivocal symptoms of hyperglycemia, the
diagnosis of diabetes should not be made on the basis of a single abnormal reading but rather should be confirmed on 2
different tests or on repeat measurement of the same test on a subsequent day.
However, test results can be discordant. If results from 2 different tests are discordant, the test with the value over the
diagnostic threshold should be repeated. This patient has an FPG over the threshold for the diagnosis of diabetes but an A1c
below; his FPG should be repeated. FPG has higher sensitivity than A1c but has a high coefficient of variation and can change
rapidly, depending on diet and activity. A single abnormal FPG often normalizes on repeat testing. Conversely, A1c is more stable
but is affected by conditions that increase red blood cell turnover (e.g., hemolytic anemia).
References
Objective number 37-2(Diabetes Mellitus) of Objectives for the Qualifying Examination, 3rd edition. Diabetes Canada Clinical
Practice Guidelines Expert Committee. Diabetes Canada 2018 Clinical Practice Guidelines. Can J Diabetes
2018;42(S1):S1-S296.
Question #28
A 35-year-old female presents to your office with the complaint of insomnia and difficulty concentrating at work for several weeks.
She reports a 20-lb weight loss over the past 2 to 3 months, despite eating more. When questioned, she reports that she
frequently feels “hot and sweaty” at work and at home. She denies chest pain or palpitations but does have diarrhea frequently.
Her blood pressure is 130/80 mmHg, pulse is 98/min, respiratory rate is 15/min, temperature is 37 C (98.9 F). She appears
worried. On physical examination, she has warm and moist skin. She has a slight hand tremor. On palpation of her thyroid, you
note a diffusely enlarged thyroid gland that is nontender. Which of the following is the best next step in management?
A) Free T4 levels
B) Thyroid-stimulating hormone (TSH) levels
C) Thyroglobulin levels
D) Radioactive iodine uptake (RAIU) scan
E) A fine-needle aspiration (FNA) biopsy
Explanation
Answer Description
Free T4 is important in the diagnostic workup of thyroid disorders and should be the next test ordered if TSH comes back
decreased. If free T4 is increased with a decreased TSH level, this is diagnostic of primary hyperthyroidism. If free T4 is
decreased with a decreased TSH level, then central hypothyroidism is the diagnosis and the etiology involves the pituitary gland
or the hypothalamus. Finally, if free T4 is normal with a decreased TSH, then this is subclinical hypothyroidism.
Thyroglobulin is often increased in goiter and hyperthyroidism and is also a tumor marker for thyroid cancer. It is not the best first
test in working up thyroid disorders.
Radioactive iodine uptake (RAIU) scan is the next best step once primary hyperthyroidism is diagnosed (increased free T4 with
decreased TSH) as it can help differentiate causes of hyperthyroidism (Graves vs. multinodular goiter vs. silent thyroiditis).
A fine-needle aspiration (FNA) biopsy is far too invasive of a test without having basic laboratory values.
Summarized Points:
In approaching the diagnosis of thyroid disorders, the first step is to order a TSH level. TSH is the most sensitive test to detect
primary hypothyroidism and hyperthyroidism. Based on the TSH level, additional tests can be performed.
More Detail
References
Blick C, Jialal I. Thyroid, Thyrotoxicosis. 2018 Jan. Objective number 63 (Neck Mass, Goiter, Thyroid Disease) of Objectives for
the Qualifying Examination, 3rd edition.
Question #30
A 56-year-old man comes to the office because of a neck mass that he first noticed 2 weeks ago while shaving. He denies any
pain in his neck, dysphagia, odynophagia, or fever. He has no significant medical problems and does not take any medications.
On physical examination, you palpate an approximately 2-cm round firm nodule in the right lobe of the thyroid. He does not have
significant cervical lymphadenopathy. Which of the following physical examination or imaging finding is most suggestive of
malignancy?
The finding of multiple 1-cm cervical lymph nodes on a CT of the neck is not suggestive of malignancy. Lymph nodes are not
considered pathologic unless they are greater than 1 cm in their short axis. Finding small lymph nodes anywhere in the lymphatic
chain is normal or may represent a recent inflammatory or infectious process that is resolving. If the CT demonstrated cervical
lymph nodes greater than 1 cm in size in this patient, this would be suggestive of a malignant process.
A "hot" lesion on a radioactive iodine scan of the thyroid is suggestive of a benign process. A "hot" lesion is indicative of
increased radioactive tracer uptake in that particular region and may be secondary to a functioning thyroid adenoma. Most
malignant tumors of the thyroid are seen as "cold" lesions on the scan since the malignant cells do not function like normal
thyroid cells and thus do not uptake iodine. Although "cold" lesions are more suggestive of malignant tumors, only 10% of "cold"
lesions are found to be malignant at pathology.
The finding of multiple solid nodules in both lobes of the thyroid is suggestive of a benign process. The finding of multiplicity is
usually associated with a benign process. This sonographic finding is suggestive of a multinodular goiter. Most malignant tumors
of the thyroid are single solid lesions on ultrasonography.
If ultrasonography of the nodule demonstrates that it is a cyst, this is consistent with a benign lesion. The thyroid gland is
normally homogeneous in echo-texture on ultrasound. There are, however, many other sonographic appearances that are still
considered benign. Some thyroid glands may appear diffusely heterogeneous and some may have multiple solid and cystic
areas. The cystic areas may represent colloid cysts or degenerating thyroid tissue in a multinodular goiter.
Summarized Points:
Thyroid carcinoma most commonly manifests as a painless, palpable, solitary thyroid nodule. Patients or clinicians discover most
of these nodules during routine palpation of the neck. Signs and symptoms associated with malignancy in thyroid nodules
include: solitary nodules in patients older than 60 years and in patients younger than 30 years, male gender, nodular growth,
rapid growth, hard and fixed nodules.
More Detail
The finding that is most suggestive of a thyroid cancer is a nodule that is fixed in place and does not move with swallowing.
Other physical examination features suggestive of malignancy include a firm or irregular nodule and a rapidly growing
lesion. Malignant lesions in the thyroid or other organ systems tend to infiltrate the adjacent tissues and often seem fixed in
place on physical exam.
References
Haugen BR, Alexander EK, Bible KC, et al. 2015 American Thyroid Association Management Guidelines for Adult Patients with
Thyroid Nodules and Differentiated Thyroid Cancer: The American Thyroid Association Guidelines Task Force on Thyroid
Nodules and Differentiated Thyroid Cancer. Thyroid. 2016 Jan. 26 (1):1-133. Objective number 63(Neck Mass, Goiter, Thyroid
Disease) of Objectives for the Qualifying Examination, 3rd edition.
Question #31
A 65-year-old woman comes to the office to discuss the results of a screening DEXA test. The patient experienced menopause
at age 52; she was treated with combined estrogen/progestin menopausal hormone therapy due to hot flashes but discontinued
the treatment after a few months due to concerns regarding adverse effects. She has experienced a 1 cm (0.4 in) loss of height
since that time. The patient quit smoking 30 years ago and drinks wine only on social occasions. Her BMI is 22 kg/m2. Physical
examination is unremarkable.
Femoral neck
DEXA Scan T -2.1
score Lumbar spine
-2.6
Complete blood
Normal
count
Serum Phosphate Normal
Renal function Normal
Which of the following additional investigations should be performed for this patient at this time?
Many experts advise measurement of 24-hour urine calcium excretion, especially in women who are likely to have inadequate
intestinal absorption of calcium (e.g., malabsorptive syndromes) or signs of excess calcium excretion (e.g., kidney stones).
However, this should be performed only after correction of vitamin D deficiency, and patients should have an adequate intake of
calcium (1,000-1200 mg/day) for 2 weeks prior to the test.
Spinal imaging (e.g., lateral spine x-ray) is indicated for patients with low bone density who are most likely to have a vertebral
fracture, including women age ≥70 (men age ≥80), loss of height >4 cm, self-reported vertebral fracture, and systemic
glucocorticoid therapy >3 months. This patient's minimal loss of height does not require imaging.
Patients with a new diagnosis of osteoporosis warrant a basic laboratory evaluation for reversible secondary causes.
Serum protein electrophoresis is indicated in patients with evidence of a possible plasma cell dyscrasia (e.g., multiple myeloma),
such as anemia, renal insufficiency, hypercalcemia, or proteinuria.
Summarized Points:
Patients with osteoporosis warrant laboratory testing for reversible secondary causes. The evaluation should include serum
chemistries (e.g., calcium, phosphorus, albumin, total protein, and renal and hepatic function markers) and complete blood
count. In addition, vitamin D stores should be assessed with a 25-hydroxyvitamin D level.
More Detail
The correct answer is B.
Osteoporosis is a common disorder in postmenopausal women. It has a long asymptomatic phase, and treatment can
reduce the risk of fracture in patients with osteoporosis. It is therefore an ideal subject for a routine screening program.
Screening for osteoporosis by DEXA is recommended for women age ≥65, and for younger women with significant risk
factors (e.g., prior low-trauma fracture, long-term glucocorticoid use, low body weight). Screening DEXA is typically performed at
the lumbar spine and hips; a T score ≤-2.5 at any location is consistent with osteoporosis. Osteoporosis is most commonly due
to age-related bone loss, which accelerates following menopause in most women. However, patients with a new diagnosis of
osteoporosis warrant a basic laboratory evaluation for reversible secondary causes. Typical studies include serum
chemistries (e.g., calcium, phosphorus, albumin, total protein, and renal and hepatic function markers) and complete blood
count. In addition, vitamin D stores should be assessed with a 25-hydroxyvitamin D level. Intake of adequate amounts of
vitamin D and calcium helps reduce bone loss.
References
Objective number 67-1-2-4 (Local Pain, Spinal Compression/Osteoporosis) of Objectives for the Qualifying Examination, 3rd
edition. Cosman F, de Beur SJ, LeBoff MS, et al. Clinician's Guide to Prevention and Treatment of Osteoporosis. Osteoporos Int
2014; 25:2359.
Question #34
A 13-year-old boy is brought to the office for follow-up of newly diagnosed type 1 diabetes mellitus. He was diagnosed a week
ago when he was admitted to the hospital with diabetic ketoacidosis. The patient was discharged on a treatment regimen
including insulin glargine and insulin lispro. Since returning home, his finger-stick blood glucose levels have ranged from 7-9.5
mmol/L (3.3-5.8). The patient is referred for diabetes education. Which of the following is also advised for the patient at this time?
Answer Description
Choice A is not correct:
Continuous subcutaneous insulin infusion (insulin pump) therapy can provide superior glycemic control compared with multidose
insulin injections. However, home management of an insulin pump requires a thorough understanding of diabetes, and initiation
of insulin pump therapy is typically made after the patient has received comprehensive diabetes education. The doses of this
patient's insulin regimen should be adjusted now, and an insulin pump can be considered in the near future.
A complete blood count is not indicated for patients with T1 DM in the absence of additional indications (e.g., hypertension, celiac
disease).
Microvascular complications of diabetes typically evolve slowly over years. In patients with T1DM, screening for microvascular
complications (e.g., spot urine albumin-creatinine ratio for nephropathy, dilated eye examination for retinopathy) should be
performed beginning 5 years after the diagnosis. In contrast, patients with type 2 diabetes commonly have elevated glucose for
years before diagnosis and therefore should be screened beginning at the time of diagnosis.
Summarized Points:
Patients with type 1 diabetes mellitus are at increased risk for additional autoimmune disorders, especially autoimmune thyroid
disease and celiac disease. Patients should be screened for autoimmune thyroid disease with a serum TSH level and for celiac
disease with assay for tissue transglutaminase (IgA) antibodies.
More Detail
Type 1 diabetes mellitus (T1DM) is caused by an autoimmune response against pancreatic beta cells that leads to
progressive loss of beta-cell mass and net secretory capacity. Patients with T1 DM are at increased risk for additional
autoimmune disorders, especially autoimmune thyroid disease and celiac disease. Patients with T1 DM should be screened
with a serum TSH level, and many experts also recommend an assay for thyroid peroxidase antibodies, especially in those with
mild elevations in TSH. This is especially important in children and adolescents, as subclinical hypothyroidism is associated with
decreased growth velocity and increased risk of hypoglycemia. Screening for celiac disease can be accomplished with an assay
for tissue transglutaminase (IgA) antibodies, with a referral for endoscopy if positive.
References
Objective number 37-2 (Diabetes Mellitus) of Objectives for the Qualifying Examination, 3rd edition. American Diabetes
Association. 4. Comprehensive Medical Evaluation and Assessment of Comorbidities: Standards of Medical Care in
Diabetes-2019. Diabetes Care. 2019 Jan;42(Suppl 1):S34-S45.
Question #38
A 48-year-old woman presents to the primary care clinic for the first time. He states that he has been in good health throughout
his life and takes no medications. She was once athletic but has noted a dramatic decrease in her muscle strength and exercise
tolerance over the past year. On examination the patient is moderately hypertensive, with a tanned, round, plethoric face; large
supraclavicular fat pads; and significant truncal obesity. He has no focal cardiovascular, pulmonary, or neurologic findings. Her
fasting blood sugar is 11 mmol/L. Which of the following is the most common etiology of this condition?
Although adrenal tumors can potentially lead to over-secretion of cortisol, they are not as common as ACTH secreting pituitary
adenomas.
An ectopic ACTH secreting tumor will present in a manner similar to an ACTH-secreting pituitary adenoma because it is
secreting ACTH out of the control of the hypothalamus-pituitary-adrenal axis. It is not, however, as common as the ACTH-
secreting pituitary adenoma.
Adrenal hyperplasia, which is most commonly bilateral, can lead to over-secretion of cortisol and Cushing’s syndrome. Primary
disease (in the absence of elevated ACTH) is extremely rare and is most appropriately treated by bilateral adrenalectomy.
Small cell lung cancers can be ACTH secreting and are a possible cause of Cushing’s syndrome, although not the most
common.
Summarized Points:
Cushing ’s syndrome presents with weakness, insomnia, mood disorders, impaired cognition, easy bruising, oligo-/amenorrhea,
hirsutism, and acne (at the mandible level).
Physical signs: central obesity, round face, supraclavicular and dorsal fat pads, facial plethora, proximal muscle wasting,
purple abdominal striae, skin atrophy, acanthosis nigricans, Hypertension, hyperglycemia, osteoporosis, pathologic fractures,
hyperpigmentation (Cushing disease).
The most common cause of Cushing syndrome
More Detail
This patient has the classic symptoms of Cushing’s syndrome, which most directly results from an excess of cortisol. ACTH is
secreted by the anterior pituitary and stimulates the adrenal cortex to secrete cortisol. The most common cause of Cushing
syndrome is exogenously administered corticosteroids; however, the patient was not on any medication, and the most
common non-iatrogenic etiology is Cushing’s disease, an ACTH-secreting pituitary adenoma. In addition, hyperpigmentation
with hypercortisolism is only seen in cushing's disease, due to increased production of melanocyte stimulating hormone (α-
MSH) induced by the tumor.
References
Graversen D, Vestergaard P, Stochholm K, Gravholt CH, Jorgensen JO. Mortality in Cushing's syndrome: a systematic review
and meta-analysis. Eur J Intern Med. 2012 Apr. 23 (3):278-82. Objective number 43 (Hirsutism, Hypertrichosis) of Objectives for
the Qualifying Examination, 3rd edition.
Question #39
A 60-year-old postmenopausal woman sustains a fragility fracture of her left wrist. Her bone density scan shows T-scores as low
as -2.8. The patient is an active smoker and has a thin body habitus. She elects to receive bisphosphonate treatment but would
like to avoid oral agents due to a history of gastroesophageal reflux disease. Which of the following side effects is most likely to
occur with intravenous zoledronic acid treatment?
Acute kidney injury (AKI) is a potential adverse effect of ZA, especially in elderly patients or patients with preexisting chronic
kidney disease. AKI is more frequent in patients receiving repeated infusions for skeletal metastases than in those receiving only
yearly infusions for osteoporosis.
Atrial fibrillation has been inconsistently reported as an adverse effect of ZA. However, the risk, if it is existent, would be low.
Bisphosphonates increase the risk of subtrochanteric and femoral shaft fractures (atypical femoral fractures), but this is a very
rare adverse event and usually occurs after long-term use (5-7 years).
Osteonecrosis of the jaw (OJN) has been reported in oncology patients receiving ZA for the treatment of skeletal metastases.
The risk of OJN increases with poor dentition or concurrent dentoalveolar surgery.
Summarized Points:
Flu-like symptoms including fever, fatigue, arthralgias, and myalgias are very common in patients receiving intravenous
bisphosphonates. Atypical femoral fractures, osteonecrosis of the jaw, and acute kidney injury are possible but uncommon
adverse events in postmenopausal women receiving treatment for osteoporosis.
More Detail
References
Objective number 67-1-2-4 (Local Pain, Spinal Compression/Osteoporosis) of Objectives for the Qualifying Examination, 3rd
edition. Powell D, Bowler C, Roberts T, Garton M, Matthews C, McCall I, Davie M. Incidence of serious side effects with
intravenous bisphosphonate: a clinical audit. QJM. 2012 Oct;105(10):965-71.
Question #40
A 67-year-old woman comes to the office to review the results of her recent DEXA scan of the hips and spine. She has a history
of essential hypertension, osteoarthritis, and acid reflux disease. Current medications include lisinopril, omeprazole, and
acetaminophen as needed. She has never smoked and has no history of fractures or falls. Height is 165 cm (5 ft 5 in) and weight
is 70 kg (154.3 lb). Laboratory testing, including renal function, electrolytes, TSH, and parathyroid hormone level, is normal.
Results of the DEXA scan are as follows:
Younge Age-
Bone
adult matched
density
(T (Z
(g/cm2)
score) score)
Lumbar spine
L1-4 (average) 0.916 -2.3 -0.7
Total proximal
femur
Left 0.641 -2.9 -1.8
Right 0.729 -2.2 -1.1
Mean 0.685 -2.6 -1.4
Which of the following is the most appropriate next step in the management of this patient?
Osteopenia is defined by the World Health Organization and the International Society for Clinical Densitometry as a T score of
-1.1 to -2.4, and osteoporosis is defined as a T score ≤ -2.5. The T score of this patient's left hip is -2.9, indicating osteoporosis
rather than osteopenia.
Peripheral bone density (e.g., calcaneus, forearm) is not recommended for routine use due to lack of standardization but can be
considered in certain circumstances, such as morbid obesity (exceeding the weight limit of the DXA machine) or metal implants
in the spine or hips (e.g., joint replacements). In addition, forearm measurement is recommended in primary
hyperparathyroidism, in which bone loss is more pronounced in cortical rather than cancellous bone.
Z scores are used to compare an individual's bone density with others the same age and can identify abnormal bone loss that is
disproportionate to that expected from aging. Z scores are useful for fracture risk estimation in young premenopausal women or
in children but are less helpful in postmenopausal women.
Summarized Points:
Osteopenia is defined as a T score of -1.1 to -2.4 and osteoporosis as a T score Diagnostic interpretation is based on the lowest
T score at the lumbar spine (average bone density at L1-L4) and either hip.
More Detail
This patient is at risk for osteoporosis based on age and female sex. In addition, she takes omeprazole, which can increase
bone loss. Osteoporosis screening is recommended for women age ≥65 and younger women who have additional risk factors
for osteoporosis. The World Health Organization and the International Society for Clinical Densitometry define osteopenia as a
T score of -1.1 to -2.4 and osteoporosis as a T score ≤-2.5. In the lumbar spine, the average bone density at L1-L4 should be
used rather than bone density at individual vertebrae. The final diagnostic interpretation is based on the lowest T score at the
lumbar spine and either hip (total proximal femur or femoral neck). This patient has a lowest T score of -2.9 (at the left hip) and
therefore meets diagnostic criteria for osteoporosis rather than osteopenia.
After an osteoporosis diagnosis, the next crucial step is to evaluate the patient's 10-year risk of fracture. This can be
accomplished by using either the WHO Fracture Risk Assessment Tool (FRAX) or the Canadian Association of
Radiologists and Osteoporosis Canada Risk Assessment Tool (CAROC). Both of these assessment tools take into account
the patient's bone mineral density and risk factors based on their medical history, and then classify the patient into one of
three categories: low, medium, or high risk. This information is vital for healthcare professionals to create a personalized
treatment plan that effectively manages the patient's condition and reduces the risk of future fractures.
References
Objective number 67-1-2-4 (Local Pain, Spinal Compression/Osteoporosis) of Objectives for the Qualifying Examination, 3rd
edition. Cosman F, de Beur SJ, LeBoff MS, et al. Clinician's Guide to Prevention and Treatment of Osteoporosis. Osteoporos Int
2014; 25:2359.
Question #1
A 51-year-old woman without significant past medical history comes to the office for a periodic routine physical examination. She
has no complaints at this time. She does not smoke cigarettes and has an “occasional” glass of wine with dinner. She has had
normal annual screening mammograms since she turned 50. Routine laboratory studies are normal except for a calcium level of
2.8 mmol/L (2.18‐2.58) and an elevated PTH level. Which of the following is the most likely diagnosis?
A) Multiple myeloma
B) Osteoporosis
C) Parathyroid adenoma
D) Parathyroid carcinoma
E) Squamous cell carcinoma
Explanation
Answer Description
Choice A is not correct:
Multiple myeloma would be suggested by anemia, fatigue, back pain, renal failure, etc. These findings, along with a low PTH
level, warrant evaluation
with an Serum Protein Electrophoresis (SPEP) and Urine Protein Electrophoresis (UPEP). High PTH levels are not consistent
with malignancy.
Osteoporosis is the result of hypercalcemia, not a cause. Osteoporosis is an indication for surgery in primary
hyperparathyroidism. Other indications for
surgery include urolithiasis, creatinine clearance <70%, serum calcium levels 1 mmol/L above upper limit, age <50 years, or
marked hypercalciuria.
In the patients with primary hyperparathyroidism, adenoma accounts for 80%, while parathyroid carcinoma accounts for <1%.
The remaining patients have parathyroid hyperplasia.
Squamous cell cancers can cause hypercalcemia via the release of parathyroid hormone-related protein. Patients classically
have abrupt, symptomatic onset of their hypercalcemia. Their serum PTH level is low.
Summarized Points:
In Primary hyperparathyroidism, patient is asymptomatic or has mild complaints of fatigue or constipation. Finding elevated
calcium with an elevated PTH (or high normal) is diagnostic of primary hyperparathyroidism. In addition, 80% of cases
caused by a parathyroid adenoma.
More Detail
The correct answer is C.
Primary hyperparathyroidism and malignancy account for most cases of hypercalcemia. Finding elevated calcium with an
elevated PTH (or high normal) is diagnostic of primary hyperparathyroidism.
References
Sharata A, Kelly TL, Rozenfeld Y, et al. Management of Primary Hyperparathyroidism: Can We Do Better? Am Surg. 2017 Jan 1.
83 (1):64-70. Objective number 12-2(Calcium Disorder) of Objectives for the Qualifying Examination, 3rd edition.
Question #2
A 24-year-old man was admitted to the hospital with a history of fatigue and confusion. A few days previously, he developed an
infection in his foot, which has become worse. The patient is a diabetic who takes insulin. His temperature is 38.5 C (101.3 F),
pulse is 96/min regular, respiration is 20/min, and blood pressure is 90/60 mm Hg. His tongue is dry, and he has poor skin turgor.
Apart from confusion, his neurologic examination is nonfocal. Cardiovascular examination is normal except for sinus tachycardia.
Respiratory system examination is unremarkable with the exception of a fruity odor in his breath. He has diffuse abdominal
tenderness, but no masses or organomegaly are noted. Bowel sounds are present. He has cellulitis in his right leg, as a result of
the infection in his foot. Serum glucose level is 39 mmol/L. The appropriate diagnosis was made, and he received intravenous
fluids, antibiotics, and human insulin. During therapy, the patient develops respiratory paralysis requiring intubation and assisted
ventilation. Which one of the following is the cause of the patient’s respiratory failure?
Bacteremia is a feature of septic shock, in which the cardiovascular system is the target. Endotoxins, especially those due to
gram-negative sepsis, lead to hypotension, tachycardia, and tachypnea but not to paralysis of the respiratory muscles.
Although hyperkalemia is commonly seen in the setting of DKA, it is not due to an excess of potassium stores, but is the result of
a transcellular shift of potassium out of cells as excess hydrogen ions in ketoacidosis are buffered intracellularly. This
transcellular shift often disguises the marked deficits in total body potassium that these patients have because of urinary
potassium loss due to the osmotic effect of glucosuria. Severe hypokalemia can also cause muscle paralysis by preventing
muscle repolarization. Therefore, potassium supplementation is extremely important in the treatment of DKA and can be given
as potassium phosphate rather than potassium chloride.
Summarized Points:
Depletion of phosphate as a result of the transcellular shift effect of insulin therapy will cause a decrease in adenosine
triphosphate (ATP) within the muscle, leading to paralysis of the respiratory muscles and respiratory failure in the patient. This
sequence of events is the rationale for providing phosphate supplementation in the treatment of DKA when phosphate levels
begin to decline during insulin therapy.
More Detail
This patient has diabetic ketoacidosis (DKA), most likely due to the increased insulin requirement generated by the
infection, a common problem. In DKA, glucosuria results in the loss of significant amounts of sodium, potassium, and
phosphorus in the urine. When insulin is used in the treatment of DKA, phosphate is normally transported along with glucose
into muscle and adipose cells; this permits phosphorylation of glucose, allowing further metabolism.
Insulin treatment also enhances glycolysis, which further depletes the already decreased concentration of phosphate in the
blood, leading to hypophosphatemia. Depletion of phosphate results in a corresponding decrease in adenosine triphosphate
(ATP) within the muscle, leading to paralysis of the respiratory muscles and respiratory failure in the patient. Phosphate
replacement should be strongly considered if severe hypophosphatemia occurs (serum phosphate concentration below 0.32
mmol/L), especially if cardiac dysfunction, hemolytic anemia, and/or respiratory depression develop. When needed, potassium
or sodium phosphate 20 to 30 mEq can be added to 1 L of intravenous fluid.
References
Braunwald E, Fauci AS, Kasper DL, et al. Harrison’s principles of internal medicine, vol 2. New York: McGraw Hill, 2001.
Chapter: DM; p2109-2135. Objective number 37 (Glucose Abnormal, Serum/Diabetes Mellitus/Polydipsia) of Objectives for the
Qualifying Examination, 3rd edition.
Question #4
A 65-year-old man comes to the office a month after sustaining a nondisplaced right femoral neck fracture in a ground-level fall.
He underwent internal fixation of the right hip and was discharged to a skilled nursing facility. The patient is now able to ambulate
with a cane. Medical history is significant for hypertension treated with hydrochlorothiazide. The patient does not use tobacco or
alcohol. Since hospital discharge, the patient has taken adequate amounts of calcium and vitamin D via diet and oral
supplementation. His physical examination is unremarkable. Laboratory tests show normal complete blood count, basic
metabolic panel, hepatic function tests, TSH, 24-hour urinary calcium, and 25-hydroxyvitamin D.
Femoral neck
Bone mineral density -T -2.0
score Lumbar spine
-2.5
Plain radiograph of the
Normal
spine
Which of the following is the most appropriate next step in the evaluation of this patient?
Hyperprolactinemia in men typically presents with central hypogonadism due to suppression of GnRH (and subsequently LH)
secretion. Common symptoms include erectile dysfunction and loss of libido. Osteoporosis is a possible manifestation, due to
low testosterone production. However, serum testosterone levels should be obtained first because, if serum testosterone is
normal, hyperprolactinemia is unlikely and there is no need to measure prolactin.
A common cause of osteoporosis is excess glucocorticoids. However, screening for hypercortisolism with a 24-hour urine cortisol
assay is necessary only for those with clinical features of Cushing syndrome.
In addition to localized osteolytic lesions, multiple myeloma can cause generalized bone loss. However, this is a less common
cause of osteoporosis, and testing with serum and urine protein electrophoresis is needed only for those with additional features
suggestive of myeloma (e.g., anemia, parathyroid hormone-independent hypercalcemia, vertebral fracture).
Bone turnover markers (e.g., urine N-telopeptide) can assess ongoing bone loss and may be useful if the need for antiresorptive
therapy is uncertain. However, antiresorptive therapy is indicated for this patient due to established osteoporosis with a fragility
fracture; measurement of bone turnover markers would not further improve his care.
Summarized Points:
Osteoporosis in men is often due to secondary causes. Elderly men with hip fractures frequently have hypogonadism and should
be screened with an early-morning serum testosterone assay.
More Detail
In contrast to women, in whom osteoporosis is usually due to physiologic bone loss following menopause, osteoporosis
in men is more likely due to secondary causes (e.g., glucocorticoid use, excessive alcohol intake). As many as two-thirds of
elderly men with hip fractures have hypogonadism, which is often not associated with other specific symptoms of testosterone
deficiency (e.g., changes in body hair, loss of libido). This patient with osteoporosis (low bone mineral density [BMD] on DEXA)
and a fragility fracture has no clear etiology for his low BMD; therefore, an early-morning serum testosterone assay is
indicated. Some men with hypogonadism who have osteoporosis may benefit from testosterone replacement therapy, which
increases BMD. Additional tests for men with osteoporosis should include blood cell counts, serum chemistries (e.g.,
electrolytes, calcium, phosphorus), liver function tests, albumin and total serum protein, alkaline phosphatase, TSH, and
25-hydroxyvitamin D. A 24-hour urine calcium assay can also be considered during initial testing, but vitamin D deficiency must
be corrected before this test is ordered. Some experts also recommend a parathyroid hormone level (especially in patients with
hypercalcemia) and a tissue transglutaminase antibody assay (especially in patients with evidence of malabsorption).
References
Objective number 67-1-2-4 (Local Pain, Spinal Compression/Osteoporosis) of Objectives for the Qualifying Examination, 3rd
edition. Finkelstein JS, Lee H, Leder BZ, et al. Gonadal steroid-dependent effects on bone turnover and bone mineral density in
men. J Clin Invest 2016; 126:1114.
Question #5
A 72-year-old woman is hospitalized for a severe urinary tract infection complicated by vomiting, altered mentation, and
hypotension and is admitted to the intensive care unit for intravenous fluids and antibiotics. On the second hospital day, the
patient develops atrial fibrillation with rapid ventricular response. Medical history is significant for hypertension. Results of thyroid
function studies are as follows:
Which of the following is the most likely cause of this patient's abnormal thyroid studies?
A) Graves’ disease
B) Pituitary apoplexy
C) Euthyroid sick syndrome
D) Silent thyroiditis
E) Toxic multinodular goiter
Explanation
Answer Description
Graves’ disease and toxic multinodular goiter cause hyperthyroidism and can lead to atrial fibrillation. However, TSH is usually
very low (<0.1 µU/mL) or undetectable, T3 and T4 are high-normal or high, and patients typically have chronic hyperthyroid
symptoms. The hyperthyroid phase of silent thyroiditis would also cause elevated T4 and T3 and usually present in younger (e.g.,
postpartum) women.
Pituitary apoplexy is caused by hemorrhage into the pituitary, typically in patients with a pituitary adenoma. This can cause acute
central hypothyroidism with low TSH and low T3, but the level of the T4 and rT3 would likely also be low and most patients have
prominent mass effect symptoms (e.g., severe headache, diplopia, visual field defects).
Summarized Points:
Euthyroid sick syndrome (ESS) is a constellation of abnormal thyroid function tests seen in severe illness. Serum T3 is low, TSH
and T4 may be normal or low, and reverse T3 is elevated. ESS usually resolves spontaneously and does not require treatment
unless the abnormalities persist after the patient has returned to baseline health.
More Detail
Euthyroid sick syndrome (ESS) is a constellation of abnormal thyroid function tests seen in patients with severe illness. T4 is
produced in the thyroid gland, whereas T3 is produced mainly by the peripheral conversion of T4. Elevated cortisol production,
cytokine release, starvation, and certain medications (e.g., glucocorticoids, amiodarone) can cause reduced conversion of T4
to T3, leading to low serum T3; reverse T3 (rT3 ); the inactive metabolite of T4) levels are typically elevated as rT3 metabolism is
inhibited in the setting of nonthyroidal illness. Severe or prolonged disease may cause decreased T4 as well; serum TSH is
variable but often normal. Some experts believe ESS to be a form of mild, transient central hypothyroidism that occurs as an
adaptive response to starvation or illness, although in true central hypothyroidism both T3 and rT3 are low.
ESS usually resolves spontaneously and does not require treatment unless the abnormalities persist after the patient has
returned to baseline health. In general, thyroid function testing should be avoided in acutely ill patients, although certain
conditions (e.g., acute atrial fibrillation, as in this patient) may necessitate testing to rule out hyperthyroidism.
References
Objective number 63(Neck Mass, Goiter, Thyroid Disease) of Objectives for the Qualifying Examination, 3rd edition. Moura Neto
A, Zantut-Wittmann DE. Abnormalities of Thyroid Hormone Metabolism during Systemic Illness: The Low T3 Syndrome in
Different Clinical Settings. Int J Endocrinol 2016; 2016:2157583.
Question #6
A 55-year-old-man comes to the office due to erectile dysfunction. He has normal libido but has been unable to maintain a
satisfactory erection to have intercourse. The patient often wakes up in the morning with an erection. He has no history of
testicular injury or peripheral vision loss. He has end-stage renal disease and is on peritoneal dialysis every other day; he is also
being evaluated for a kidney transplant. The patient does not use tobacco, alcohol, or illicit drugs. Temperature is 37.1 C (98.7
F), blood pressure is 138/82 mm Hg, pulse is 76/min, and respirations are 12/min. BMI is 25 kg/m2. Visual fields are normal on
confrontation. Testes are normal in size and consistency, and cremasteric reflex is present. There is trace edema at the ankles
and feet. Laboratory results are as follows:
Which of the following is the most appropriate next step in the management of this patient's erectile dysfunction?
Impaired gonadal function in ESRD can lead to low serum testosterone levels, decreased libido, and erectile dysfunction.
However, this patient has normal libido and a normal serum testosterone level; testosterone supplementation is unlikely to be
beneficial.
This patient has no clinical features (e.g., visual field defects) to suggest a pituitary mass lesion. Men with ESRD often have
elevated prolactin levels due to increased production and decreased clearance. The levels can be corrected with dopamine
agonists (e.g., bromocriptine), but this has not been shown to improve sexual function.
Alprostadil (prostaglandin E1) can be administered intraurethrally or injected into the penile shaft and causes vasodilation of the
cavernosal arteries. It may be used as second-line therapy in men who fail phosphodiesterase inhibitors but is associated with a
risk of local bleeding in uremic men who often have platelet dysfunction.
Sexual dysfunction in men with ESRD often improves following renal transplant and may also improve with correction of
concurrent anemia and zinc deficiency. However, dialysis does not usually improve sexual function, regardless of modality.
Summarized Points:
Most men with end-stage renal disease experience sexual dysfunction (e.g., low libido, erectile dysfunction, decreased
frequency of intercourse, infertility). Contributing factors include vascular impairment, peripheral and autonomic neuropathy,
gonadal dysfunction, psychological stress, concurrent medications, and medical comorbidities. Most patients respond to
phosphodiesterase inhibitors (e.g., sildenafil).
More Detail
Most men with end-stage renal disease (ESRD) experience sexual dysfunction. Manifestations can include low libido, erectile
dysfunction, decreased frequency of intercourse, and infertility. Sexual dysfunction in ESRD is usually multifactorial. Common
contributing factors include vascular/vasoactive impairment, peripheral and autonomic neuropathy, gonadal dysfunction,
psychological stress, concurrent medications, and medical comorbidities.
The initial evaluation of sexual dysfunction in patients with ESRD should include a clinical review for possible contributing
factors (e.g., medications, vascular disease) and limited laboratory testing (e.g., serum testosterone, prolactin levels). Most
patients with erectile dysfunction respond to phosphodiesterase inhibitors (e.g., sildenafil). This patient's normal nonsexual
morning erections suggest a possible psychogenic component, but this is unlikely to be the only factor in a patient with ESRD,
and the effectiveness of psychotherapy for sexual dysfunction in men with ESRD is unknown.
References
Objective number 48(Erectile Dysfunction) of Objectives for the Qualifying Examination, 3rd edition. Lasaponara F, Sedigh O.
Phosphodiesterase type 5 inhibitor treatment for erectile dysfunction in patients with end-stage renal disease receiving dialysis or
after renal transplantation. J Sex Med. 2013 Nov;10(11):2798-814.
Question #8
A 28-year-old woman with type 1 diabetes mellitus has 6 months of increasing fatigue and occasional lightheadedness. The
patient takes 2-3 naps during the day due to feeling exhausted. Her blood pressure is 92/60 mm Hg, and pulse is 95/min. The
patient's body mass index is 18 kg/m2. Physical examination shows hyperpigmentation around her lips and in the palmar
creases. Her serum creatinine and liver function tests are normal. Which of the following is the most likely cause of this patient's
symptoms?
A) Adrenal insufficiency
B) Dilated cardiomyopathy
C) Hemochromatosis
D) Hypothyroidism
E) Insulin resistance
Explanation
Answer Description
Dilated cardiomyopathy can cause congestive heart failure with fatigue, dyspnea on exertion, and hypotension. However, it is
usually associated with weight gain due to peripheral edema. In addition, dilated cardiomyopathy does not typically cause
hyperpigmentation.
Hemochromatosis is characterized by abnormal liver function studies, weakness, hyperpigmented skin, and diabetes mellitus.
This patient's normal liver function tests make this less likely.
Hypothyroidism can cause fatigue, weakness, periorbital edema, and shortness of breath. However, it is not usually associated
with significant hypotension or hyperpigmentation.
Insulin resistance is associated with type 2 diabetes and can present with acanthosis nigricans, hyperglycemia, hirsutism, and
obesity. This patient's anorexia and type 1 diabetes make this unlikely.
Summarized Points:
Chronic primary adrenal insufficiency can present with chronic fatigue, generalized weakness, weight less, gastrointestinal
symptoms, postural hypotension, and hyperpigmentation or vitiligo. Testing should include early morning measurement of serum
cortisol and ACTH and a high-dose ACTH stimulation test
More Detail
The correct answer is A.
This patient's presentation suggests chronic primary adrenal insufficiency (PAI) (Addison's disease). PAI is most commonly
due to autoimmune adrenalitis but can also be caused by infection (e.g., HIV, tuberculosis), hemorrhage (e.g., meningococcus,
oral anticoagulants), or metastatic malignancy. Undiagnosed chronic PAI may lead to acute adrenal crisis, presenting with shock,
fever, abdominal pain, and confusion. Chronic PAI can have findings due to both mineralocorticoid and glucocorticoid
deficiency. Patients usually develop chronic malaise, fatigue, generalized weakness, and weight loss. Other findings can
include nonspecific abdominal symptoms (e.g., nausea, diarrhea, abdominal pain) and postural hypotension with dizziness or
syncope. Patients can have hyperpigmentation (increased melanocyte-stimulating hormone and melanin synthesis) or vitiligo
(autoimmune destruction of melanocytes). Common metabolic abnormalities include hyponatremia, hyperkalemia, and metabolic
acidosis. Initial evaluation should include early morning measurement of serum cortisol and ACTH and a high-dose ACTH
(cosyntropin) stimulation test.
References
Objective number 33(Fatigue) of Objectives for the Qualifying Examination, 3rd edition. Bornstein SR, Allolio B, Arlt W, et al.
Diagnosis and Treatment of Primary Adrenal Insufficiency: An Endocrine Society Clinical Practice Guideline. J Clin Endocrinol
Metab. 2016 Feb;101(2):364-89. Nieman LK. Diagnosis of adrenal insufficiency in adults. Post TW, ed. UpToDate. Waltham, MA:
UpToDate Inc. Accessed October 16, 2019.
Question #11
A 42-year-old man undergoes computed tomography (CT) of his cervical spine for chronic neck pain and radiculopathy. Results
reveal a 3-millimetre incidental pituitary mass. A physical examination shows normal visual fields and no neurologic
abnormalities. Which additional step should be taken?
All pituitary incidentalomas should be assessed for hormonal hypersecretion with at least a prolactin level.
The size of the mass (3 millimetres) is consistent with a microadenoma, which is defined as <10 millimetres. Some experts
recommend repeating imaging in 12 months for all microadenomas, whereas other experts recommend repeating imaging only
for microadenomas that are larger than 5 millimetres and/or those that secrete hormones. Repeat imaging in 6 months is only
recommended for macroadenomas (>10 millimetres).
Checking for hypopituitarism is more strongly and universally recommended for macroadenomas or larger microadenomas, as
compression due to the lesion's size creates hypopituitarism. To check for hypopituitarism, assessing free T4, morning cortisol,
and testosterone levels are more consistently recommended, although some experts encourage measuring TSH, LH, FSH, and
IGF-1 (insulin growth factor 1) levels as well.
Summarized Points:
All patients with an incidental finding of pituitary adenoma should be assessed for hormonal hypersecretion with a prolactin level
at a minimum. Experts consistently recommend that functional adenomas, larger microadenomas, and macroadenomas should
be followed with imaging, whereas some experts do not recommend follow-up imaging for nonfunctioning microadenomas <5
millimetres in size.
More Detail
References
Lake M, Krook L, Cruz S. Pituitary adenomas: an overview. Am Fam Physician. 2013;88(5):319-327. Snyder PJ. Incidentally
discovered sellar masses (pituitary incidentalomas). Post TW, ed. UpToDate. UpToDate Inc. Accessed March 31, 2021.
Question #13
A 2-year-old boy with Down syndrome is brought to the clinic for a routine health maintenance visit. His parents say that he has
been constipated and has hard, pellet-like stools twice a week. His last bowel movement was 3 days ago, but he has had no
abdominal pain or vomiting. The boy takes no medications and has a history of duodenal atresia successfully repaired shortly
after birth. His vaccinations are up to date. Complete blood count and thyroid function testing were normal at his 1-year checkup.
His growth chart shows declining height velocity from the 15th percentile to <5th percentile since birth. Weight is at the 50th
percentile. Examination shows typical Down facies with a single palmar crease and protruding tongue. What is the best next step
in the management of this patient?
A) Abdominal x-ray
B) Complete blood count
C) Fasting serum glucose
D) Serum thyroid-stimulating hormone
E) Reassurance, follow-up in 1 month
Explanation
Answer Description
Choice A is not correct:
Abdominal x-rays are unnecessary in the absence of vomiting, abdominal pain, or other signs or symptoms of obstruction.
Although Hirschsprung disease occurs in approximately 1% of patients with DS, it usually presents in the neonatal period with
bilious emesis, abdominal distension, and failure to pass meconium within the first 2 days of life.
The risk of developing acute lymphoblastic leukemia is 10-20 times higher in DS, but this patient does not have typical acute
leukemic symptoms of fever, bleeding, bone pain, or lymphadenopathy.
Although the risk of type I diabetes mellitus may be up to 8 times greater in DS and diabetes mellitus can stunt growth, this
patient does not have polyuria, polydipsia, or polyphagia. Constipation is not a feature of diabetes mellitus.
Delay in evaluation is inappropriate and may lead to further harmful declines in growth and development. Untreated
hypothyroidism can lead to complications such as fatigue, weakness, cognitive dysfunction, depression, arthralgia, and pubertal
delay.
Summarized Points:
Annual screening for thyroid disorders should be performed in all patients with Down syndrome. Hypothyroidism in children
commonly presents with declining growth velocity and should be evaluated by serum thyroid-stimulating hormone.
More Detail
Down syndrome (DS) is the most common chromosomal abnormality among liveborn infants. It is characterized by a variety of
dysmorphic features, congenital malformations, and other comorbidities (Table). Thyroid disorders are common in DS, with
hypothyroidism most commonly presenting with declining growth velocity. Other manifestations of hypothyroidism include
constipation, poor appetite, cold intolerance, sluggishness, dry skin, brittle hair, facial puffiness, and muscle aches and pains. In
addition to newborn screening, thyroid function testing should be performed at ages 6 and 12 months and repeated
annually thereafter, even if the child is asymptomatic. This child's growth pattern and symptoms are concerning for
hypothyroidism, and serum thyroid-stimulating hormone should be checked. Levothyroxine is the treatment of choice for
hypothyroidism, and it should be started if laboratory results are abnormal to restore normal growth and development.
References
Objective number 36-2 (Congenital Anomalies, Dysmorphic Features) of Objectives for the Qualifying Examination, 3rd edition.
Purdy IB, Singh N, Brown WL, Vangala S, Devaskar UP. Revisiting early hypothyroidism screening in infants with Down
syndrome. J Perinatol. 2014 Dec;34(12):936-40. Bunt CW, Bunt SK. Role of the family physician in the care of children with
Down syndrome. Am Fam Physician. 2014 Dec 15;90(12):851-8. Ostermaier KK. Down syndrome: Management. Post TW, ed.
UpToDate. Waltham, MA: UpToDate Inc. Accessed Nov 09, 2020.
Question #15
A 36-year-old woman comes to the office for follow-up of hypothyroidism. She has no current symptoms. The patient was
diagnosed with hypothyroidism secondary to Hashimoto's thyroiditis 4 years ago. She takes 150 µg levothyroxine daily before
breakfast and her dose has been stable for the last year. Physical examination, including thyroid examination, is normal. Thyroid
function tests are as follows:
Which of the following is the best next step in the management of this patient?
Answer Description
In most patients on LT4 therapy who have a TSH within the reference range, circulating T3 levels will also be within the reference
range. In some cases, treatment with LT4 alone may fail to restore plasma T3 to a level within the reference range despite a
normal TSH. However, the significance of this low T3 is unknown, and the American Thyroid Association recommends against
increasing LT4 dose, treating with combination LT4 and liothyronine (T3), or switching to liothyronine alone in these patients.
Choice C is not correct:
Levels of free T4 above the reference range are often observed during replacement therapy with LT4, especially if the blood
sample is drawn 1-3 hours after LT4 administration. This condition is not associated with adverse events, and the dose of LT4
should not be changed.
Thyroid peroxidase (TPO) antibodies are useful for diagnosis in cases of subclinical hypothyroidism, recurrent miscarriages, and
euthyroid diffuse goiters. However, there is no benefit to measuring TPO antibodies once the diagnosis of Hashimoto's thyroiditis
has been established (as in this patient).
Summarized Points:
The therapeutic goal in primary hypothyroidism is to maintain TSH levels within the normal reference range by replacement with
levothyroxine. Levothyroxine dose adjustment should be based on TSH levels alone rather than direct measurement of thyroid
hormone levels.
More Detail
Synthetic levothyroxine (LT3) is the preferred replacement therapy for hypothyroidism. Most actions of the thyroid hormone in
the body are mediated through triiodothyronine (T3), which is secreted in active form by the thyroid gland (15%-20%) and also
converted from thyroxine (T4) in extrathyroidal tissues (80%-85%). Because a large percentage of T3 is formed from T4
conversion in the peripheral tissues, patients with hypothyroidism are usually able to achieve adequate T3 levels with LT4
treatment alone.
The recommended therapeutic goal in patients with primary hypothyroidism is to maintain TSH within the normal reference
range. Serum TSH is the most reliable marker of levothyroxine replacement therapy in hypothyroid patients as it reflects the
biologic activity of thyroid hormones in the tissues (i.e., hypothalamus). For this reason, TSH alone is usually sufficient to monitor
LT4 treatment in patients with primary hypothyroidism. However, despite this recommendation, thyroid hormone levels are
frequently ordered by practitioners along with TSH to monitor LT4 therapy.
References
Objective number 33 (Fatigue) of Objectives for the Qualifying Examination, 3rd edition. Jonklaas J; American Thyroid
Association Task Force on Thyroid Hormone Replacement. Guidelines for the treatment of hypothyroidism: prepared by the
american thyroid association task force on thyroid hormone replacement. Thyroid. 2014 Dec;24(12):1670-751.
Question #16
A 42-year-old female presents to your office with chief complaint of fatigue and weight gain for the past 5 to 6 months. Her
fatigue has also affected her performance at work, where she has difficulty concentrating on tasks. She recently began losing
hair. She feels more tired than usual after work and has difficulty playing with her children in the evening. She did not have any of
these symptoms until 6 months ago. She denies hot/cold intolerance. Weight has been relatively stable but she has gained <10
pounds in the past 6 months despite attempting to eat healthy and exercise a few times weekly. She is generally happy and
denies any recent mood changes. Past medical history is insignificant. The patient smokes five to six cigarettes a day and drinks
alcohol socially. She takes birth control pills. Her blood pressure is 120/80 mm Hg, pulse is 55/min, respiratory rate is 16/min,
BMI 31Kg/m2. Physical examination reveals a pleasant female, oriented and appropriate. Ear, nose and throat examination
reveals normal pharynx, no neck fullness, and no palpable thyroid nodules. Abdomen is soft, nontender. Skin examination
reveals no lesions/rashes and there is no lower extremity edema. She donated blood last week and was told that her hemoglobin
was normal. What is the appropriate next step in managing this patient?
A) Ultrasound of thyroid
B) Thyroid peroxidase antibodies (TPO)
C) Thyroid uptake scan
D) Fine needle aspiration (FNA)
E) Thyroid stimulating hormone (TSH)
Explanation
Answer Description
Thyroid peroxidase antibodies (TPO) would help confirm Hashimoto’s as a potential cause of hypothyroidism but would not help
confirm hypothyroidism itself as patients may have elevated antibodies but normal thyroid function.
Thyroid uptake scan would be more helpful in evaluating hyperthyroidism, such as thyroiditis or Grave's disease.
Fine needle aspiration (FNA) is only indicated if a patient has a nodule with size or features worrisome for malignancy and in
need of tissue diagnosis.
Summarized Points:
Primary hypothyroidism is the failure of the thyroid to produce sufficient thyroid hormone. This accounts for about 95% of all
cases. The best initial screening test is thyroid stimulating hormone (TSH).
More Detail
The patient has symptoms suggestive of hypothyroidism (bradycardia, fatigue, weight gain, hair loss) and thyroid stimulating
hormone (TSH) is the best initial screening test. If TSH levels are above the reference range, the next step is measure free
https://afkebooks.com
thyroxine (T4).
References
Garber JR, Cobin RH, Gharib H, et al. Clinical practice guidelines for hypothyroidism in adults: cosponsored by the American
Association of Clinical Endocrinologists and the American Thyroid Association. Thyroid. 2012 Dec. 22(12):1200-35. Objective
number 63(Neck Mass, Goiter, Thyroid Disease) of Objectives for the Qualifying Examination, 3rd edition.
https://afkebooks.com
Question #21
A 27-year-old woman comes to the office due to excess facial hair that she finds embarrassing. She has had this problem for
years and has tried different methods of hair removal, with unsatisfactory results. The patient has no chronic medical conditions
and has regular menstrual periods. She is not sexually active and takes no medications. BMI is 29 kg/m2. On physical
examination, there is coarse, dark hair on her upper lip, chin, midsternum, and buttocks. The rest of the examination is
unremarkable. Which of the following is the best next step in the management of this patient?
https://afkebooks.com
Explanation
Answer Description
Patients who do not wish to become pregnant are treated with a combination of estrogen/progestin oral contraceptive pills, which
can reduce androgenic symptoms (e.g., hirsutism, acne). Serum testosterone levels are measured first to rule out a more serious
cause.
https://afkebooks.com
Very high testosterone levels (>5.2nmol/L) suggest a possible androgen-secreting ovarian tumor and require pelvic ultrasound.
Spironolactone is used as second-line therapy for hyperandrogenic symptoms that do not improve with oral contraceptives.
Thyroid function tests can diagnose hypo- or hyperthyroidism. This patient has no other findings (e.g., fatigue, weight changes)
to suggest thyroid disease.
Summarized Points:
Hirsutism is excess terminal hair growth in androgen-dependent areas (e.g., chin, upper lip, upper abdomen, chest, back). All
women with hirsutism require a serum total testosterone level to evaluate for possible underlying androgen disorders.
More Detail
Nearly 5%-10% of women will develop hirsutism, defined as excess terminal hair growth (dark, coarse hair) in androgen-
dependent areas (e.g., chin, upper lip, upper abdomen, chest, back). Hirsutism should be differentiated from hypertrichosis,
which is increased growth of light unpigmented hair in nonsexual areas that can be due to systemic conditions (e.g.,
hypothyroidism) or medications (e.g., phenytoin). Mildly increased facial hair is also seen in certain ethnicities (e.g., Hispanic)
and can be considered a normal variant.
Common causes of hirsutism include polycystic ovary syndrome (PCOS), idiopathic hirsutism, and nonclassic
21-hydroxylase deficiency. PCOS is the most common and is associated with oligomenorrhea, obesity, type 2 diabetes
mellitus, and dyslipidemia. All women with hirsutism require a serum total testosterone level to evaluate for an underlying
androgen disorder. Additional testing (e.g., 17-hydroxyprogesterone) may be required depending on additional clinical features
(e.g., irregular menses, virilization).
References
https://afkebooks.com
Objective number 43 (Hirsutism, Hypertrichosis) of Objectives for the Qualifying Examination, 3rd edition. Martin KA, Anderson
RR, Chang RJ, et al. Evaluation and Treatment of Hirsutism in Premenopausal Women: An Endocrine Society Clinical Practice
Guideline. J Clin Endocrinol Metab 2018; 103:1233.
https://afkebooks.com
Question #26
A 34-year-old woman is evaluated in the first trimester of pregnancy for a 2-week history of heat intolerance, palpitations, and
tremulousness. Her only medication is folic acid. On physical examination, pulse rate is 110/min; remaining vital signs are
normal. The thyroid is nontender and symmetrically and diffusely enlarged. A fine tremor on the patient's outstretched hands is
noted. Laboratory studies show:
Thyroid-stimulating
0.01 mIU/L (0.4‐5.0)
hormone
Thyroxine, free 68 pmol/L (13-27 )
https://afkebooks.com
Explanation
Answer Description
Thyroid scintigraphy with RAIU is contraindicated in pregnancy. In this patient, measurement of TSI or TRAb is preferred to
evaluate the possibility of Graves disease.
Thyroid ultrasonography may help identify thyroid nodules not detected on physical examination but would not provide
information about whether they are the cause of thyroid dysfunction. Doppler studies with thyroid ultrasonography may show
increased vascularity indicative of thyroid hyperfunction or decreased vascularity indicative of thyroiditis or exogenous thyroid
use. This imaging technique alone is insufficiently specific to guide management.
Total T3 measurement may be useful in identifying clinical thyrotoxicosis in the setting of a normal free T4 level, although total T3
levels are increased in pregnancy because of changes in thyroid-binding globulin. In this patient, however, the free T4 level is
undisputedly elevated and the total T3 would not add any useful information.
Summarized Points:
Possible causes of thyrotoxicosis in pregnant persons include human chorionic gonadotropin-mediated hyperthyroidism, Graves’
disease, and thyroiditis. In the diagnosis of Graves’ disease, measurement of thyroid-stimulating immunoglobulin or thyrotropin
receptor antibodies can be helpful if thyroid scintigraphy with radioactive iodine uptake is unavailable, unreliable, or
contraindicated.
More Detail
https://afkebooks.com
The diagnosis of hyperthyroidism is based on biochemical testing demonstrating a low serum thyroid-stimulating hormone
(TSH) level and elevated concentrations of free thyroxine (T4) and/or total triiodothyronine (T3). Thyroid scintigraphy with
radioactive iodine uptake (RAIU) can verify the cause. Additional testing can be done when the clinical diagnosis is unclear;
when RAIU is unavailable or unreliable, such as in patients taking amiodarone or lithium or those recently exposed to iodinated
contrast material; or when scintigraphy is contraindicated, such as in pregnancy and lactation. In the absence of RAIU, additional
tests include the measurement of TSI or thyrotropin receptor antibodies (TRAb). In this patient, TSI measurement is a
reasonable first test because an abnormal result has prognostic and treatment implications. Other possible causes of the
patient's hyperthyroidism include human chorionic gonadotropin (hCG)-mediated hyperthyroidism and thyroiditis. Because
hCG stimulates thyroid hormone secretion, TSH may be mildly suppressed as a result. Serum TSH gradually returns to the
nonpregnant reference range in the second and third trimester. Determining the cause of hyperthyroidism is important because
transient causes of pregnancy-related hyperthyroidism, such as hCG-mediated hyperthyroidism and thyroiditis, may not require
intervention other than laboratory monitoring.
References
Objective number 63(Neck Mass, Goiter, Thyroid Disease) of Objectives for the Qualifying Examination, 3rd edition. Alexander
EK, Pearce EN, Brent GA, et al. 2017 Guidelines of the American Thyroid Association for the Diagnosis and Management of
Thyroid Disease During Pregnancy and the Postpartum. Thyroid. 2017 Mar;27(3):315-389. Ross DS. Hyperthyroidism during
pregnancy: Clinical manifestations, diagnosis, and causes. Post TW, ed. UpToDate. Waltham, MA: UpToDate Inc. Accessed
October 15, 2021.
https://afkebooks.com
Question #27
A 27-year-old woman comes to the physician due to poor sleep, hand tremors, and heat intolerance for the last 2-3 weeks. She
has no fever or neck pain. Examination shows mild, diffuse thyroid enlargement. Her hands are warm and sweaty with a fine
tremor. The remainder of the examination is unremarkable. Laboratory results show:
Which of the following is the best next step in the management of this patient?
A) Beta blacker
B) High-dose glucocorticoid
C) Methimazole
D) Nonsteroidal anti-inflammatory drug
E) Radioactive iodine ablation
https://afkebooks.com
Explanation
Answer Description
Nonsteroidal anti-inflammatory drugs (NSAIDs) are used to treat subacute (de Quervain's) thyroiditis, which is characterized by
systemic symptoms and a tender goiter. High-dose glucocorticoids are generally used in amiodarone-induced destructive
thyroiditis (type II), subacute thyroiditis unresponsive to NSAIDs, and severe PT.
Methimazole decreases thyroid hormone formation and is effective for Graves' disease. However, it is less effective for painless
thyroiditis as the hyperthyroid state is due to the release of preformed thyroid hormone from the thyroid gland.
Radioactive iodine ablation is effective for Graves' disease and toxic nodular goiter. It can also be considered for patients with
recurrent PT. However, most patients with PT will recover normal thyroid function and ablative therapies are not needed.
Summarized Points:
Painless thyroiditis is characterized by a brief period of hyperthyroid symptoms followed by a hypothyroid phase and subsequent
recovery. Patients with mild symptoms usually require no therapy; beta-blockers are used for symptomatic thyrotoxic patients.
More Detail
This patient, with hyperthyroidism and a diffuse painless goiter, has typical features of subacute painless thyroiditis (PT).
PT is likely a variant of Hashimoto's thyroiditis with autoimmune thyroid inflammation and is characterized by transient
hyperthyroidism followed by possible hypothyroidism and then recovery within 3-6 months. It is most common in women
following pregnancy, although by convention painless thyroiditis within 1 year of delivery is termed post-partum thyroiditis. PT
https://afkebooks.com
can be distinguished from Graves' disease by radioactive iodine uptake (RAIU), which is decreased in PT and increased in
Graves' disease. Patients with mild symptoms generally require no treatment and are monitored with repeat thyroid function
studies in 6-8 weeks. However, those with significant symptoms may benefit from beta-blockers for symptom relief. Thyroxine
replacement therapy is also sometimes required during the hypothyroid phase; most patients eventually recover, but up to 20%
can develop permanent hypothyroidism.
References
Objective number 63(Neck Mass, Goiter, Thyroid Disease) of Objectives for the Qualifying Examination, 3rd edition. Bahn Chair
RS, Burch HB; American Thyroid Association; American Association of Clinical Endocrinologists. Hyperthyroidism and other
causes of thyrotoxicosis: management guidelines of the American Thyroid Association and American Association of Clinical
Endocrinologists. Thyroid. 2011 Jun;21(6):593-646.
https://afkebooks.com
Question #29
A 63-year-old woman with a 10-year-history of type 2 diabetes mellitus comes to the physician because of unsteadiness. She
stumbles occasionally and has had several near-fall episodes over the last 6 months. Her medications include aspirin, lisinopril,
metformin, sitagliptin, and statin. Her blood pressure is 133/70 mm Hg supine and 135/76 mm Hg while standing, and her pulse
is 82/min. Her body mass index is 32 kg/mV. Pinprick and vibration sensations are decreased in both feet Ankle reflexes are
absent bilaterally. Laboratory results are as follows:
Which of the following most likely contributed to this patient's current condition?
A) Autonomic neuropathy
B) Jejunal villous atrophy
C) Metformin therapy
D) Pancreatic atrophy
E) Proteinuria
https://afkebooks.com
Explanation
Answer Description
Autonomic neuropathy can result in recurrent falls and presyncopal episodes due to orthostatic hypotension. However, this
patient's blood pressure does not drop upon standing.
Vitamin B12 deficiency is not generally seen with jejunal villous atrophy because the absorption of vitamin B12 occurs in the
terminal ileum. In patients with jejunal villous atrophy (e.g., celiac disease), iron and vitamin D deficiencies are common.
Pancreatic atrophy usually presents with steatorrhea. Although deficiency of fat-soluble vitamins and vitamin B12 may occur with
pancreatic atrophy, clinically significant deficiency is not very common except for vitamin D deficiency.
Proteinuria does not usually cause vitamin B12 deficiency. Heavy proteinuria can cause iron and vitamin D deficiency due to
urinary losses of transferrin and vitamin D binding protein.
Summarized Points:
The risk of developing vitamin B12 deficiency is increased in patients treated with metformin. The risk increases with older
patients, higher metformin doses, and longer durations of treatment. Metformin is thought to impair the terminal ileal absorption
of the vitamin by inhibiting the calcium-dependent binding of the intrinsic factor- B12 complex to its receptor.
More Detail
https://afkebooks.com
This patient's presentation (low vitamin B12 level and associated neurologic symptoms) is consistent with B12 deficiency.
Although this patient may have diabetic peripheral neuropathy, a, deficiency is most likely contributing to her neurologic
dysfunction. Metformin reduces intestinal absorption of vitamin B12 by up to 30%. The mechanism is thought to involve
impaired calcium-dependent binding of the intrinsic factor- B12 complex to its receptor, thus decreasing ileal absorption. The risk
of developing vitamin B12 deficiency increases with older patients, higher metformin doses, and longer durations of
treatment. Neurologic symptoms may appear before anemia develops. Small amounts of vitamin B12 present in daily
multivitamin supplements do not protect against the development of vitamin B12 deficiency in metformin-treated patients. As a
result, patients taking metformin should be routinely monitored for B12 deficiency. Increased calcium intake may improve vitamin
B12 absorption.
References
Objective number 37-2 (Diabetes Mellitus) of Objectives for the Qualifying Examination, 3rd edition. Ahmed MA, Muntingh G,
Rheeder P. Vitamin B12 deficiency in metformin-treated type-2 diabetes patients, prevalence and association with peripheral
neuropathy. BMC Pharmacol Toxicol 2016; 17:44.
https://afkebooks.com
Question #31
A woman at 7 months’ gestation presents for a prenatal examination. She has a history of mild hyperthyroidism, and despite her
advanced pregnancy, she is found to have lost 4 lb since her last visit a month ago. She also has developed a fine tremor of her
fingers. Her resting heart rate is 120 beats/min, and her thyroid gland is noticeably larger than it was at the time of her last office
visit. Management of this patient’s condition would be best achieved by treatment with which of the following?
A) Lugol’s solution
B) Radioactive iodine (131I)
C) Iodide followed by surgical removal of the gland
D) Propylthiouracil
E) Propranolol
https://afkebooks.com
Explanation
Answer Description
Lugol’s solution (iodine and potassium iodide) exerts only a temporary antithyroid effect.
Antithyroid agents carry less risk to the patient and the pregnancy than does surgery; in any event, surgery is contraindicated
during the third (and first) trimester of pregnancy.
Propranolol should not ordinarily be given as an adjuvant because it may cause fetal growth retardation and neonatal respiratory
depression.
Summarized Points:
Propylthiouracil is usually preferred over methimazole, in part because the latter drug enters the fetal circulation more readily and
has been reported to cause aplasia cutis.
More Detail
Treatment of hyperthyroidism during pregnancy depends in part on the severity of the disorder. Hyperthyroid females can often
tolerate the mild exacerbation of the state that accompanies pregnancy without therapy, and there appears to be no increase in
https://afkebooks.com
the incidence of fetal loss. If treatment is judged to be required, propylthiouracil is usually preferred over methimazole, in part
because the latter drug enters the fetal circulation more readily and has been reported to cause aplasia cutis.
Propylthiouracil also inhibits 5'-deiodinase, decreasing formation of triiodothyronine (T3). The primary objective of drug therapy is
to maintain free thyroxine (FT4) and thyroid-stimulating hormone (TSH) at high-to-normal levels using the lowest possible dose
of propylthiouracil.
References
Blick C, Jialal I. Thyroid, Thyrotoxicosis. 2018 Jan. Objective number 63 (Neck Mass, Goiter, Thyroid Disease) of Objectives for
the Qualifying Examination, 3rd edition.
https://afkebooks.com
Question #33
A 35-year-old woman is evaluated for persistent mild hypercalcemia that was incidentally discovered several years ago. Her
father also has persistent, asymptomatic hypercalcemia. The patient has no medical concerns and takes no medications. She
takes vitamin D, 5000 IU, daily. Physical examination is normal. Laboratory studies:
https://afkebooks.com
Explanation
Answer Description
An ectopic PTH-secreting tumour is a relatively rare cause of hypercalcemia and is described in a limited number of case
reports. Although ectopic PTH secretion can cause elevated PTH and calcium levels, it is not associated with hypocalciuria.
Primary hyperparathyroidism in adolescents and young adults may be the first sign of multiple endocrine neoplasia (MEN)
syndrome. Primary hyperparathyroidism is associated with MEN1 and MEN2A syndromes. In contrast to sporadic primary
hyperparathyroidism, patients with MEN syndromes have hyperplasia of multiple parathyroid glands. However, patients with
MEN-associated hypercalcemia have elevated urine calcium, often occurring before hypercalcemia occurs. This patient's urine
calcium excretion is low.
Although this patient is taking a high dose of vitamin D, in the absence of other causes of hypercalcemia and/or without
concomitant generous calcium intake, a supplement of 5000 IU/d of vitamin D will not cause toxicity or hypercalcemia. If excess
vitamin D action were responsible for this patient's hypercalcemia, PTH would be suppressed and urine calcium excretion
increase. This patient has elevated PTH in relation to serum calcium and low urine calcium excretion; neither is compatible with
the diagnosis of vitamin D toxicity.
Summarized Points:
Signs suggestive of familial hypocalciuric hypercalcemia include mild hypercalcemia since childhood; low 24-hour urine calcium
excretion, especially if calcium-creatinine clearance ratio is less than 0.01; and/or family history of parathyroidectomy without
resolution of hypercalcemia.
https://afkebooks.com
More Detail
The parathyroid glands and kidney detect serum calcium concentrations through the calcium-sensing receptor (CaSR). In
Familial hypocalciuric hypercalcemia (FHH), inactivating mutation of the CaSR gene causes the parathyroid gland to perceive
serum calcium levels as low, resulting in increased parathyroid hormone (PTH) secretion and a higher serum calcium level.
Simultaneously, the mutated CaSR in the kidney increases kidney reabsorption of calcium, leading to paradoxical
hypocalciuria in the setting of hypercalcemia. Although these patients appear to have primary hyperparathyroidism, FHH is a
benign condition that will not resolve with parathyroidectomy. Patients do not have sequelae of hypercalcemia, such as stones
or osteoporosis. Signs suggestive of FHH include mild hypercalcemia since childhood; low 24-hour urine calcium excretion,
especially if the calcium-creatinine clearance ratio is less than 0.01; and/or family history of parathyroidectomy without
resolution of hypercalcemia. Although testing for mutations in the CaSR gene is a more direct approach, not all affected families
have mutations in this gene, yet all will have a low calcium-creatinine clearance ratio. PTH values in FHH may be slightly above
or within the reference range and, as such, do not help distinguish primary hyperparathyroidism from FHH.
References
Objective number 12-2(Calcium Disorders) of Objectives for the Qualifying Examination, 3rd edition. Lee JY, Shoback DM.
Familial hypocalciuric hypercalcemia and related disorders. Best Pract Res Clin Endocrinol Metab 2018; 32:609. Mannstadt M.
Disorders of the calcium-sensing receptor: Familial hypocalciuric hypercalcemia and autosomal dominant hypocalcemia. Post
TW, ed. UpToDate. Waltham, MA: UpToDate Inc. Accessed November 11, 2020.
https://afkebooks.com
Question #34
A previously healthy 18-year-old man is brought to the emergency department because of abdominal pain and nausea for 6
hours. He has had decreased appetite for the past week. He takes no medications. He drinks one to two beers daily and
occasionally more on weekends. He does not use illicit drugs. His temperature is 37.8°C (100°F), pulse is 120/min, respirations
are 24/min, and blood pressure is 105/60 mm Hg. Abdominal examination shows diffuse tenderness with no guarding or
rebound. Bowel sounds are normal. Finger stick in the way of emergency department shows a glucose level of 35 mmol/L. In the
emergency department, fluids and insulin are started, and laboratory tests show:
About 10 hours later, the ECG tracing shows U waves and a prolonged QT interval. Which of the following is the most likely
etiology of this patient ECG changes?
A) Fluid overload
B) Hyperglycemic hyperkalemia
C) Hypomagnesemia
D) Ischemic cardiomyopathy
E) Whole body potassium depletion
https://afkebooks.com
Explanation
Answer Description
Fluid overload is unlikely in patients being treated for DKA, who have severe dehydration and must be treated aggressively with
fluids. Fluid overload might cause signs of heart failure in patients with underlying heart disease but would not cause these ECG
changes.
Summarized Points:
Patients with diabetic ketoacidosis suffer from potassium depletion; however, levels may be normal or even elevated on initial
presentation due to potassium movement out of cells secondary to acidosis. As insulin is infused, potassium returns to cells, and
levels can drop acutely, causing hypokalemia.
More Detail
https://afkebooks.com
This patient is suffering from diabetic ketoacidosis (DKA).Patients with DKA suffer from potassium depletion; paradoxically,
their potassium levels may be normal or even elevated on initial presentation due to potassium movement out of cells secondary
to acidosis. As insulin is infused, however, potassium returns to cells, and levels can drop acutely. Potassium must be carefully
monitored and repleted in these patients. In this case, potassium was not repleted, and the patient exhibits ECG changes
characteristic of hypokalemia: U waves and a prolonged QT interval.
References
Maletkovic, J; Drexler, A (December 2013). "Diabetic ketoacidosis and hyperglycemic hyperosmolar state". Endocrinology and
metabolism clinics of North America. 42 (4): 677–95. Objective number 37-2 (Diabetes Mellitus) of Objectives for the Qualifying
Examination, 3rd edition.
https://afkebooks.com
Question #36
A 51-year-old woman comes to the physician due to a persistent dry cough after an upper respiratory infection 2 months ago.
She has no hemoptysis, chest pain, or weight loss. Chest examination is unremarkable. Laboratory results are as follows:
Chest x-ray reveals right paratracheal and bilateral hilar adenopathy. CT reveals adenopathy and a 1.5- cm left lower lung
nodule. Which of the following is the most likely cause of this patient's hypercalcemia?
https://afkebooks.com
A) Multiple myeloma
B) Parathyroid adenoma
C) Squamous cell carcinoma
D) Sarcoidosis
E) Vitamin D intoxication
https://afkebooks.com
Explanation
https://afkebooks.com
https://afkebooks.com
Answer Description
Multiple myeloma can cause PTH-independent hypercalcemia due to osteolysis and increased bone turnover. However,
1,25-hydroxyvitamin D is not elevated in multiple myeloma.
Parathyroid adenoma is the most common cause of primary hyperparathyroidism. Patients will have PTH-dependent
hypercalcemia, with elevated or high-normal PTH out of proportion to the hypercalcemia. However, this patient's low-normal PTH
makes this unlikely.
Squamous cell carcinoma can cause PTH-independent hypercalcemia due to the production of PTHrP. However, it does not
usually cause elevated 1,25-hydroxyvitamin D.
Vitamin D intoxication with 25-hydroxyvitamin D levels > 374 nmol/L increases calcium absorption and bone turnover, causing
hypercalcemia. Vitamin D intoxication does not cause elevated 1,25-hydroxyvitamin D, and this patient's hilar adenopathy makes
sarcoidosis or lymphoma more likely.
Summarized Points:
Patients with hypercalcemia and elevated 1,25-hydroxyvitamin D should have a chest x-ray to evaluate for granulomatous
diseases (sarcoidosis) and lymphoma.
More Detail
https://afkebooks.com
Hypercalcemia can be divided into 2 broad categories depending on the parathyroid hormone (PTH) level: parathyroid-
dependent (elevated PTH) and parathyroid-independent (suppressed PTH). Examples of parathyroid-dependent
hypercalcemia include primary hyperparathyroidism, lithium-induced hypercalcemia, and familial hypocalciuric hypercalcemia.
Parathyroid-independent causes include most malignancies, vitamin D toxicity, granulomatous disease, and milk-alkali
syndrome. Patients with suppressed PTH should have measurement of PTH-related peptide (PTHrP) and both
1,25-hydroxyvitamin D and 25- hydroxyvitamin D. Elevated PTHrP usually indicates solid tumor malignancy; elevated
25-hydroxyvitamin D indicates vitamin D toxicity. This patient has elevated 1,25-hydroxyvitamin D, which is usually seen in
granulomatous diseases and lymphoma. Sarcoidosis is a multisystem granulomatous disease with noncaseating tissue
granulomas. Activated pulmonary macrophages convert 25-hydroxyvitamin D to 1,25-hydroxyvitamin D, which is responsible for
hypercalcemia and hypercalciuria.
References
Objective number 12-1(Hypercalcemia) of Objectives for the Qualifying Examination, 3rd edition. Renaghan AD, Rosner MH.
Hypercalcemia: etiology and management. Nephrology Dialysis Transplantation. 2018; 33 (4):549-551. Gwadera Ł, Białas AJ,
Iwański MA, Górski P, Piotrowski WJ. Sarcoidosis and calcium homeostasis disturbances-Do we know where we stand? Chron
Respir Dis. 2019 Jan-Dec;16:1479973119878713. Shane E. Diagnostic approach to hypercalcemia. Post TW, ed. UpToDate.
Waltham, MA: UpToDate Inc. Accessed June 07, 2022
https://afkebooks.com
Question #39
A 47-year-old man comes to the office after his blood pressure was found to be 146/92 mm Hg at a local health fair. Since then,
his blood pressure at home has consistently been >130/80 mm Hg. The patient has had no unusual headaches, blurry vision,
polyuria, chest pain, or shortness of breath. His brother underwent thyroidectomy for differentiated papillary thyroid cancer at age
34. The patient has a current 15-pack-year smoking history and does not drink alcohol or use illicit drugs. Blood pressure is 140/
90 mm Hg, and pulse is 68/min and regular. BMI is 27 kg/m2. Repeat blood pressure measurement after 5 minutes is 140/92 mm
Hg. Physical examination is unremarkable. Nonfasting laboratory results are as follows:
Serum chemistry
Urine microalbumin-to-creatinine ratio, TSH, and ECG are normal. Fasting lipid profile is pending. Which of the following
additional tests is recommended at this time?
Screening for lung cancer with an annual low-dose chest CT scan is indicated for patients age 55-80 who have a ≥30-pack-year
smoking history and who currently smoke or quit within the last 15 years. In light of this patient's younger age and <30-pack-year
smoking history, screening is not recommended.
Testing for primary hyperaldosteronism with plasma aldosterone to plasma renin activity ratio is appropriate for patients with
resistant hypertension (i.e., uncontrolled despite an appropriate multi-drug regimen) or hypertension with significant
hypokalemia. Screening for pheochromocytoma with plasma fractionated metanephrines is appropriate for patients with severe,
labile, or resistant hypertension. These tests are not needed for routine evaluation of uncomplicated hypertension.
Screening for thyroid malignancy with thyroid ultrasound is indicated for patients at high risk for thyroid cancer (e.g., multiple
endocrine neoplasia type 2, childhood neck irradiation). This patient's brother had papillary thyroid cancer, which is usually
sporadic and does not warrant screening of family members.
Summarized Points:
The prevalence of type 2 diabetes mellitus is increased in patients with hypertension, and patients with both hypertension and
diabetes are at increased risk for cardiovascular complications. Patients with hypertension should be screened for diabetes with
a fasting glucose level, hemoglobin A1 c, or oral glucose tolerance test.
More Detail
The prevalence of type 2 diabetes mellitus is increased in patients with hypertension, at least in part due to shared risk
factors (e.g., obesity, sedentary lifestyle). Furthermore, hypertensive patients who also have diabetes are at elevated risk for
cardiovascular complications (e.g., myocardial infarction, stroke). For these reasons, screening for diabetes is
recommended for patients with a new diagnosis of hypertension. Identifying comorbid diabetes is also important because it
may influence the choice of antihypertensive therapy (eg, ACE inhibitor). Screening tests for diabetes include fasting blood
glucose, hemoglobin A1c, and 2-hour oral glucose tolerance test (primarily used in pregnant women). Nonfasting glucose levels
can be helpful, especially when very high (≥11.1 mmol/L ), but are less specific. Screening is also appropriate for patients who
are overweight or obese, and those who have a sedentary lifestyle, first-degree family history of diabetes, or history of
gestational diabetes.
References
Objective number 37-2 (Diabetes Mellitus) of Objectives for the Qualifying Examination, 3rd edition. Diabetes Canada Clinical
Practice Guidelines Expert Committee. Diabetes Canada 2018 Clinical Practice Guidelines. Can J Diabetes
2018;42(S1):S1-S296.
Question #2
A 61-year-old man with ischemic cardiomyopathy comes to the physician with painful enlargement of both breasts for the last 2-3
weeks. He also has significant fatigue, loss of libido, and erectile dysfunction. The patient's medications include candesartan,
carvedilol, spironolactone, furosemide, atorvastatin, and aspirin. Bilateral tender gynecomastia is seen on examination. Lungs
are clear to auscultation. Testicular examination is unremarkable. Which of the following is the most appropriate next step in the
management of this patient?
Certain beta-blockers, including carvedilol, extended-release metoprolol, and bisoprolol, as well as ACE inhibitors and
angiotensin receptor blockers have been shown to decrease hospitalization and mortality in SCHF. Gynecomastia is not a
common adverse effect of these medications.
This patient's symptoms are likely due to the antiandrogenic effects of spironolactone. If spironolactone discontinuation does not
resolve his symptoms, evaluation for other causes (e.g., hypogonadism, hyperprolactinemia, hypothyroidism) would then be
appropriate.
Summarized Points:
Aldosterone receptor blockers - spironolactone and eplerenone - improve survival in systolic heart failure with low left ventricular
ejection fraction. Both medications may cause hyperkalemia. Spironolactone has significant anti-androgenic effects. Eplerenone
has fewer endocrine side effects.
More Detail
This patient most likely has gynecomastia from spironolactone. Aldosterone receptor antagonists (spironolactone and
eplerenone) are widely used in clinical practice as they have been shown to reduce mortality in patients with systolic
congestive heart failure (SCHF). However, aldosterone receptor antagonists are associated with significant potential adverse
effects, especially hyperkalemia. In addition, spironolactone blocks testosterone receptors, which may lead to low libido and
breast enlargement and tenderness in men. It also increases clearance of testosterone, increases conversion of testosterone
to estrogen, and decreases overall testosterone production. In general, spironolactone is preferred over eplerenone due to its
lower cost and well-established clinical experience. However, both aldosterone receptor blockers appear equally efficacious;
eplerenone may be substituted in patients who develop anti-androgenic side effects on spironolactone. Mild gynecomastia is
usually reversible after discontinuation of spironolactone.
References
Objective number 10-3 (gynecomastia) of Objectives for the Qualifying Examination, 3rd edition. Sica DA. Mineralocorticoid
Receptor Antagonists for Treatment of Hypertension and Heart Failure. Methodist Debakey Cardiovasc J. 2015 Oct-
Dec;11(4):235-9.
Question #6
A 55-year-old woman presents to the clinic complaining of a 4-week history of urinary frequency. She also complains of
excessive thirst during the day and awakening several times during the night to urinate. About 4 months ago, she sustained a
head injury after falling from a 10-foot ladder while cleaning windows at her home. Her 24-hour urine output is 200 mL/kg per
day. Which of the following is the most likely diagnosis?
A) Acute cystitis
B) Acute pyelonephritis
C) Central diabetes insipidus
D) Nephrogenic diabetes insipidus
E) Primary polydipsia
Explanation
Answer Description
Acute cystitis usually presents with irritative voiding symptoms (e.g., dysuria), frequency, urgency, and suprapubic pain and
discomfort. Laboratory studies typically show pyuria, hematuria, and bacteriuria.
Acute pyelonephritis typically presents with fever, chills, flank pain, costovertebral angle tenderness, and nausea and vomiting.
Nephrogenic diabetes insipidus can present similarly to central diabetes insipidus, however, this is due to impaired (rather than
insufficient production of) antidiuretic hormone action in the kidney. Causes include drugs (lithium), metabolic conditions
(hypokalemia), or intrinsic renal damage.
Primary polydipsia, also referred to as psychogenic polydipsia, is characterized by a primary increase in water intake. This
disorder is most often seen in middle-aged women and in patients with psychiatric illnesses, including those taking
phenothiazine, which can lead to the sensation of a dry mouth.
Summarized Points:
Diabetes insipidus (DI) results from insufficient antidiuretic hormone (ADH) production by the hypothalamus (central diabetes
insipidus) or from impaired ADH action in the kidney (nephrogenic diabetes insipidus). ADH deficiency results in large amounts
of dilute urine. In central diabetes insipidus, an insufficient amount of ADH is released in response to physiologic stimuli. Causes
of central diabetes insipidus include head trauma (as in this case), congenital, and genetic disorders. This patient most likely
suffered traumatic damage to the pituitary stalk resulting in central diabetes insipidus.
More Detail
Polyuria, nocturia, and polydipsia following head trauma is strongly suggestive of central diabetes insipidus. Central
diabetes insipidus results from insufficient antidiuretic hormone production by the hypothalamus. The most likely cause for
central diabetes insipidus in this patient is head trauma that likely resulted in damage to the hypothalamic-pituitary stalk.
Other causes include congenital (e.g., congenital hypopituitarism) and genetic disorders. Treatment is geared towards
decreasing urine output and can be achieved by a low-sodium diet, desmopressin, and thiazide diuretics.
References
Objective number 110-2 (Polyuria and/or Polydipsia) of Objectives for the Qualifying Examination, 3rd edition. Bichet DG. Clinical
manifestations and causes of central diabetes insipidus. Post TW, ed. UpToDate. Waltham, MA: UpToDate Inc. Accessed
November 11, 2020.
Question #8
A 66-year-old woman with rheumatoid arthritis presents to the clinic to go over her recent bone density testing results. She is
taking methotrexate. Her T-score is -2.6 in her spine and hip. After discussing treatment options, she elects to start alendronate.
What is the next best step in management?
Although zoledronic acid is bisphosphate. it is given intravenously and is not recommended as first-line therapy for most patients
with osteoporosis. It may be used in patients who cannot tolerate oral bisphosphonates, and it is administered annually.
Routine monitoring of patients on antiresorptive therapy with bone turnover markers (fasting urinary N-telopeptide or serum
carboxy-terminal collagen crosslinks) is not necessary. However, for patients who are reluctant to take medications regularly or
have disorders that can interfere with drug absorption (e.g., small bowel resection), monitoring bone turnover markers at
baseline then at three and six months is an option for monitoring therapy.
Summarized Points:
For patients starting on therapy, obtain a follow-up dual-energy x-ray absorptiometry (DXA) of the hip and spine after two years,
and if BMD is stable or improved, less frequent monitoring thereafter.
More Detail
Osteoporosis is characterized by low bone mass, bone fragility, disruption of bony tissue microarchitecture, and consequent
increased risk of fracture. The risk of osteoporosis increases with age, and dual-energy X-ray absorptiometry (DXA)
screening is recommended for all women 65 years of age and for postmenopausal women < 65 years of age with risk
factors, such as rheumatoid arthritis, long-term glucocorticoid therapy, previous fragility fracture, risk of falls, low body weight,
cigarette smoking, excessive alcohol consumption, parental history of osteoporotic fracture, white or Asian race, and factors
leading to secondary osteoporosis (inflammatory bowel disease, hypogonadism or premature menopause. chronic liver disease,
and malabsorption).
Diagnosis is made if DXA T-score is 2.5 standard deviations or more below the young-adult mean bone density or in the
presence of a fragility fracture (due to a fall from a standing height or less), particularly at the hip, spine, rib, pelvis, or
humerus. The mainstay of treatment includes adequate intake of calcium and vitamin D, fall prevention counselling,
smoking cessation, weight-bearing exercise, and avoidance of heavy alcohol use. Oral bisphosphonates (e.g., alendronate)
are recommended as first-line agents for the majority of postmenopausal women with osteoporosis. After initiation of an oral
bisphosphonate, a follow-up DXA should be performed in two years.
References
Objective number 67-1-2-4 (Local Pain, Spinal Compression/Osteoporosis) of Objectives for the Qualifying Examination, 3rd
edition. Rosen HN, Drezner MK. Overview of the management of osteoporosis in postmenopausal women. Post TW, ed.
UpToDate. Waltham, MA: UpToDate Inc. Accessed July 19, 2021.
Question #15
A 46-year-old woman presents to your office with a chief complaint of polyuria that has become progressively worse over the
past several days. In-office urinalysis is negative for glucose and reveals a low urine osmolality. A basic metabolic panel reveals
a serum sodium concentration of 145 mmol/L (135-145). What additional test is the most appropriate to establish the diagnosis
at this time?
A hemoglobin A1C concentration would be beneficial to obtain if the patient was thought to have a diagnosis of diabetes mellitus.
This particular patient had no glucose in her urine, making diabetes mellitus less likely.
Plasma renin levels are often useful when evaluating sodium disorders but are not routinely indicated in the evaluation of
diabetes insipidus. Renin levels may be low in mineralocorticoid deficiency or high in glucocorticoid deficiency.
A renal ultrasound may reveal anatomic abnormalities and should be considered, particularly if other testing is inconclusive. The
water restriction test is a more appropriate initial diagnostic step in this particular patient.
24-hour urine output collection used to be a primary evaluation test in polyuric disease. It has fallen out of favor, however, due to
potential collection issue problems, especially given the need for multiple containers due to high volumes.
Summarized Points:
The water restriction test is diagnostic for diabetes insipidus. This test determines whether a patient can concentrate urine when
fluid is withheld. Normally, this would cause increased production of antidiuretic hormone and small volumes of very
concentrated urine. In diabetes insipidus patients, they are unable to concentrate their urine. The second part of the test involves
administering desmopressin. This will help the central diabetes insipidus patient concentrate their urine, but not the nephrogenic
or psychogenic cases.
More Detail
The correct answer is A.
Polyuria and dilute urine are commonly due to uncontrolled diabetes mellitus, primary polydipsia, central diabetes insipidus, or
nephrogenic diabetes insipidus. This patient is likely suffering from diabetes insipidus. Central diabetes insipidus is due to a
lack of antidiuretic hormone secretion. Most cases are idiopathic, but some are triggered by trauma, hypoxic or ischemic
encephalopathy, or pituitary surgery. In contrast, nephrogenic diabetes insipidus is defined by normal antidiuretic hormone
secretion but impaired renal response to the hormone's effects. In adults, nephrogenic diabetes insipidus is most commonly
acquired secondary to lithium use and hypercalcemia. The patient's history and serum sodium concentration can help to
discriminate between the various causes of polyuria, however, specific testing is often necessary to establish a diagnosis of
diabetes insipidus. With central diabetes insipidus, the onset is abrupt, and with nephrogenic, the onset is more gradual.
A clue to diagnosis is the finding of new-onset nocturia since the body's impaired ability to concentrate urine with diabetes
insipidus often leads to nocturia. An elevated serum sodium concentration, in conjunction with dilute urine, typically supports
diabetes insipidus. If unclear, the diagnosis of diabetes insipidus can be confirmed with a water restriction test. This test can
also help to differentiate between primary polydipsia and central diabetes insipidus as both conditions will respond to exogenous
antidiuretic hormone administration. Patients should be instructed to stop drinking fluids at least two to three hours prior to
the test. Then, urine volume and urine osmolality are measured every two hours. If the urine osmolality reaches a normal
value, it indicates that antidiuretic hormone release and its effects are appropriate (as in the case of primary polydipsia). If the
urine osmolality remains stable despite an increase in plasma osmolality, this is considered an abnormal result (indicating
diabetes insipidus). Desmopressin can then be administered, and the urine volume and osmolality should be measured. Urine
osmolality will not change upon administration of desmopressin if the patient has nephrogenic diabetes insipidus but will adjust
significantly if central diabetes insipidus is present.
References
Objective number 110-2 (Polyuria and/or Polydipsia) of Objectives for the Qualifying Examination, 3rd edition. Bichet DG.
Evaluation of patients with polyuria. Post TW, ed. UpToDate. UpToDate Inc. Accessed June 25, 2021.
Question #24
A 30-year-old woman, gravida 1 para 1, comes to the office due to ongoing nipple discharge despite stopping breastfeeding 2
months ago. She had an uncomplicated vaginal delivery 13 months ago and her child is healthy. Her last menstrual period was
prior to her pregnancy. She is monogamous and sexually active with her husband and does not use contraception. Milky
discharge is easily expressed from both breasts. No other abnormalities are seen. Urine β-hCG is negative. Laboratory results
are as follows:
61 µmol
Serum creatinine
(50-90)
4 mIU/
FSH
mL(2-12)
3 mIU/mL
LH
(2-9)
209 ng/
Prolactin
mL(3-27)
4.8 µU/mL
TSH
(0.4-5.0)
Which of the following is the most appropriate next step in the management of this patient?
A) Breast ultrasound
B) Medroxyprogesterone acetate challenge
C) Reassurance and follow-up in 1 month
D) Total T4, free T4, and free T3 levels
E) Brain MRI
Explanation
Answer Description
Although patients with nipple discharge may require breast imaging (e.g., ultrasound, mammogram) to evaluate for malignancy,
breast cancer is associated with unilateral serous or serosanguinous discharge. This patient’s markedly elevated prolactin level
and bilateral nipple discharge make prolactinoma a more likely diagnosis.
A medroxyprogesterone acetate challenge induces menstrual bleeding in patients with amenorrhea if there is sufficient estrogen
priming of the endometrium. This patient may or may not bleed following medroxyprogesterone depending on her estrogen
status, but this would not address the hyperprolactinemia.
This patient is no longer breastfeeding and is not taking any medications (e.g., antipsychotics) that would explain increased
prolactin levels. As a result, brain MRI is the next best step.
Hypothyroidism leads to high thyrotropin-releasing hormone, which can stimulate prolactin release and cause mild
hyperprolactinemia. However, markedly elevated prolactin levels are not caused by hypothyroidism, and this patient's TSH is
normal.
Summarized Points:
Pituitary prolactinomas present with galactorrhea, amenorrhea, and marked hyperprolactinemia (>200 ng/mL). Diagnosis is
confirmed via brain MRI. First-line treatment is with dopamine agonists (e.g., cabergoline, bromocriptine).
More Detail
Prolactin levels naturally increase during pregnancy due to stimulation from high estrogen levels, peak at delivery, and then
normalize within 1-2 months after delivery, even with continued breastfeeding. Although prolactin levels increase transiently
above basal levels with response to suckling, this response significantly diminishes a few weeks after delivery. This patient is no
longer breastfeeding and is not taking any medications (e.g., antipsychotics) that would explain increased prolactin levels.
Therefore, the most likely cause of this patient's elevated prolactin level is a prolactinoma (i.e., lactotroph adenoma).
Prolactinomas can present with secondary amenorrhea, bilateral milky nipple discharge (i.e., galactorrhea), and
hypogonadism. Patients with significantly elevated prolactin (>200 ng/mL) require brain MRI to screen for a pituitary mass.
Additional evaluation of prolactinomas includes formal visual field testing due to the potential for the pituitary mass to
compress the optic chiasm. Prolactinomas that cause hypogonadism (e.g., amenorrhea) or neurologic symptoms (e.g.,
headaches, visual disturbances) secondary to mass effect are treated first-line with dopamine agonists (e.g., cabergoline,
bromocriptine) and followed with serial MRIs. Surgery (e.g., transsphenoidal resection) is reserved for patients who fail medical
therapy.
References
Objective number 10-2 (Breast Discharge) of Objectives for the Qualifying Examination, 3rd edition. Romijn JA.
Hyperprolactinemia and prolactinoma. Handb Clin Neurol. 2014;124:185-95.
Question #27
A 48-year-old woman with type 2 diabetes mellitus comes to the office for follow-up. She is currently taking the maximum daily
dose of metformin. The patient has been counseled about her diet and exercise but has had a 3.2-kg (7-Ib) weight gain over the
last year. Her BMI is 33 kg/m2. Serum creatinine is 79 µmol/L (50-90), and hemoglobin A1c is 7.8%. Which of the following is the
most appropriate second medication for improving this patient's glycemic status?
A) Glimepiride
B) Insulin glargine
C) Exenatide
D) Pioglitazone
E) Sitagliptin
Explanation
Answer Description
Patients who do not reach their target A1c or who cannot tolerate these medications may benefit from the addition of a
sulfonylurea (e.g., glimepiride) or thiazolidinedione (e.g., pioglitazone) or from stopping the medications and starting basal insulin
(e.g., insulin glargine). However, these options are associated with significant weight gain.
Dipeptidyl peptidase-4 (DDP-4) inhibitors (e.g., sitagliptin) increase endogenous GLP-1 and gastric inhibitory peptide levels by
decreasing their breakdown by DPP-4 enzymes. Compared with GLP-1 analogues, DDP-4 inhibitors are less effective for
achieving both glycemic control and weight loss.
Summarized Points:
Injectable glucagon-like peptide-1 (GLP-1) analogues (e.g., exenatide, liraglutide) are useful in patients with type 2 diabetes who
fail metformin monotherapy. GLP-1 analogues are associated with weight loss and low hypoglycemic risk.
More Detail
This patient has suboptimal control of her type 2 diabetes on maximum-dose metformin and requires a second agent. Given
her recent weight gain, a glucagon-like peptide-1 (GLP-1) receptor agonist (e.g., exenatide, liraglutide) is the best choice.
GLP-1 is released by the small intestine in response to food intake and induces insulin secretion by pancreatic beta cells. Other
effects include suppression of glucagon secretion, delayed gastric emptying, and enhanced satiety. Injectable GLP-1 analogues
that mimic the effects of native GLP-1 are effective in treating type 2 diabetes.
GLP-1 receptor agonists are usually added to metformin and reduce hemoglobin A1c by 0.5%-1%, induce weight loss, and have
a low risk of hypoglycemia (alone or with metformin). Current guidelines from Diabetes Canada recommend GLP-1 analogues
for patients with A1c <8% who desire weight loss and low hypoglycemic risk.
References
Objective number 37-2 (Diabetes Mellitus) of Objectives for the Qualifying Examination, 3rd edition. Davies M, Færch L,
Jeppesen OK, et al. Semaglutide 2·4 mg once a week in adults with overweight or obesity, and type 2 diabetes (STEP 2): a
randomised, double-blind, double-dummy, placebo-controlled, phase 3 trial. Lancet 2021; 397:971.
Question #28
A 49-year-old patient underwent thyroidectomy and sustained damage to the parathyroid glands. Which of the following
symptoms/findings would be expected?
A) Polyuria
B) Hyperreflexia
C) Constipation
D) Headache
E) Low blood pressure
Explanation
Answer Description
Polyuria is more indicative of hypercalcemia or other electrolyte imbalances but is not associated with hypocalcemia.
Low blood pressure is not typical of hypocalcemia though it may be associated with other electrolyte disturbances, such as
hyponatremia.
Summarized Points:
Acute hypocalcemia causes neuromuscular irritability. This can manifest as cramps, paresthesias, seizures, tetany,
laryngospasm, and arrhythmias.
More Detail
Hyperreflexia is typically seen in patients with hypocalcemia. Acute hypocalcemia leads to neuromuscular irritability. This
can lead to cramps, paresthesias (especially peri-oral and in the peripheral extremities), tetany, seizures, laryngospasm, and
QTc interval prolongation. Physical manifestations of hypocalcemia include the Chvostek sign and Trousseau sign. The Chvostek
sign is a unilateral contraction of the facial muscles when the facial nerve is tapped in front of the ear. Trousseau sign is carpal
spasm after occluding the brachial artery with an inflated blood pressure cuff.
References
Objective number 12-2 (Calcium Disorders) of Objectives for the Qualifying Examination, 3rd edition. Wang TS, Lyden ML, Sosa
JA. Thyroidectomy. Post TW, ed. UpToDate. Waltham, MA: UpToDate Inc. Accessed August 25, 2021.
Question #36
A 29-year-old woman is evaluated for a 3-month history of galactorrhea, fatigue, constipation, and weight gain of 3.6 kg (8.0 lb).
With the onset of galactorrhea, her menstrual periods have become irregular and associated with excessive bleeding. Her most
recent menstrual period was 5 weeks ago. She has no other medical concerns and takes no medications. On physical
examination, vital signs are normal. BMI is 28 kg/m2. Spontaneous galactorrhea is present. Visual fields are intact. Deep tendon
reflexes are delayed. The remainder of her physical examination is normal.
A) Pituitary MRI
B) Cabergoline therapy
C) Estrogen and progesterone therapy
D) Thyroid-stimulating hormone measurement
Explanation
Answer Description
Pituitary MRI is not indicated at this time because this patient's hyperprolactinemia may be explained by hypothyroidism and
there is no evidence of mass effect. If hypothyroidism is diagnosed, it will be necessary to ensure that the hyperprolactinemia
normalizes after treatment with levothyroxine. If the prolactin level does not normalize, pituitary MRI is indicated.
In patients with macroadenomas, dopamine agonist therapy is recommended to lower prolactin, reduce tumour size, and restore
gonadal function. Cabergoline, a dopamine agonist, is the preferred agent because of its superior efficacy in lowering prolactin
and tumour shrinkage compared with bromocriptine. For this patient, a pituitary prolactinoma has not yet been diagnosed;
therefore, consideration of cabergoline is premature. Although cabergoline is an appropriate therapy for a patient with
hyperprolactinemia caused by a prolactinoma, this patient's hyperprolactinemia may be explained by hypothyroidism.
Further evaluation is needed before initiating estrogen/ progesterone therapy to treat this patient's amenorrhea and to prevent
bone loss. If hypothyroidism is the cause of this patient's hyperprolactinemia and menstrual irregularities, and if the patient does
not currently want to become pregnant, estrogen/progesterone replacement therapy is reasonable.
Summarized Points:
For a patient presenting with hyperprolactinemia and hypothyroidism, the hypothyroidism should be treated first and then the
patient's prolactin level should be reevaluated to ensure that the hyperprolactinemia has resolved.
More Detail
References
Objective number 10-2 (Breast Discharge) of Objectives for the Qualifying Examination, 3rd edition. Petersenn S. Biochemical
diagnosis in prolactinomas: some caveats. Pituitary. 2020 Feb;23(1):9-15. Snyder PJ. Management of hyperprolactinemia. Post
TW, ed. UpToDate. Waltham, MA: UpToDate Inc. Accessed June 30, 2022.
Question #38
A 26-year-old woman comes to the office due to persistent fatigue and inability to lose weight. She has also experienced hair
loss and dry skin. The patient was diagnosed with primary hypothyroidism 2 years ago and was started on levothyroxine therapy,
which she takes on an empty stomach every morning. She exercises 5 times per week, follows very strict caloric restrictions, and
has eliminated gluten-containing foods from her diet. Medical history is unremarkable, and the patient has never been pregnant.
In addition to levothyroxine, she also takes a cyclic estrogen-progestin oral contraceptive. Blood pressure is 124/80 mm Hg and
pulse is 70/min. BMI is 26 kg/m2. The thyroid gland is diffusely enlarged, firm, and nontender, and the remainder of the physical
examination is normal. Laboratory results are as follows:
Which of the following is the most appropriate next step in the management of this patient's condition?
Selenium supplements can lower thyroid peroxidase antibody levels in Hashimoto thyroiditis and may decrease levothyroxine
dose requirements in some patients. However, this patient is biochemically euthyroid, and the evidence does not show clinically
significant enhancements in thyroid function in such patients.
Estrogen-containing oral contraceptives stimulate the synthesis of thyroxine-binding globulin, necessitating a higher dose of
levothyroxine to maintain free hormone levels. However, this patient's normal TSH indicates that this effect has been accounted
for. In addition, although weight gain is commonly attributed to oral contraceptives, this has not been confirmed in controlled
trials.
Because many hypothyroidism symptoms are nonspecific, patients frequently believe that their T4 dose is insufficient when they
feel tired or gain weight. Before increasing the dose, serum TSH should be measured to rule out the possibility of an inadequate
current T4 dose. If their serum TSH level is normal, the T4 dose does not need to be changed.
Clinicians sometimes add liothyronine (T3) to levothyroxine (T4) in an attempt to alleviate residual symptoms. However
randomized studies have shown no benefit of this approach over levothyroxine monotherapy, and the short half-life of
liothyronine can lead to nonphysiologic fluctuations in thyroid hormone levels.
Summarized Points:
Primary hypothyroidism is treated with levothyroxine, with the dose adjusted to maintain serum TSH in the normal (or low-
normal) range. Further dose increases or the addition of liothyronine (T3) are not recommended. Hypothyroid symptoms are
nonspecific, and patients with residual symptoms should be evaluated for alternate etiologies.
More Detail
This patient has primary hypothyroidism, most likely due to chronic lymphocytic (Hashimoto) thyroiditis (e.g., diffuse, non-
tender goiter). She is on appropriate levothyroxine replacement therapy, and although she has persistent symptoms, her normal
TSH indicates that she is in a biochemically euthyroid state (some experts suggest checking free T4 for confirmation in such
patients, but this is not routinely needed).
However, the symptoms of hypothyroidism are nonspecific and overlap extensively with other common conditions (e.g., affective
disorders, sleep disorders, anemia). Some patients report a greater sense of well-being when their levothyroxine dose is titrated
to maintain TSH in the lower half of the normal range, but the dose should not be increased further in an attempt to improve
symptoms as doing so may increase the risk of thyrotoxic complications (e.g., hypertension, osteoporosis). This patient is on
an appropriate dose of levothyroxine; other causes for her symptoms should be sought.
References
Objective number 63(Neck Mass, Goiter, Thyroid Disease) of Objectives for the Qualifying Examination, 3rd edition. Jonklaas J,
Bianco AC, Bauer AJ, et al. Guidelines for the treatment of hypothyroidism: prepared by the american thyroid association task
force on thyroid hormone replacement. Thyroid 2014; 24:1670.
Question #40
A 33-year-old man presents complaining of 3 weeks of palpitations, anxiety, and tremors. He feels his heart racing at rest and
with activity. His symptoms are unchanged throughout the day. He denies abdominal pain, flushing, sweating, or diarrhea. The
patient does not take any medications or other supplements. blood pressure is 115/65 mmHg, pulse is 110/min, respiratory rate
is 14/min, and oxygen saturation of 98% in room air. On physical examination, there is a fine tremor of the hands, with slight lid
lag and mild proptosis bilaterally. There is no lymphadenopathy, and the thyroid gland is not tender or enlarged. An ECG shows
sinus tachycardia and no ST abnormalities. Laboratory results are as follows:
A) Echocardiogram
B) Thyroid ultrasound
C) Radionuclide iodine-123 thyroid uptake and scan
D) Thyroid peroxidase antibodies
Explanation
Answer Description
If patients have prolonged tachycardia from their thyrotoxicosis, then they can be at risk for developing cardiomyopathy.
However, this patient has only been having symptoms for three weeks, and the priority is to determine the cause of the
thyrotoxicosis.
A thyroid ultrasound would only be appropriate if a thyroid nodule was found on the exam.
Thyroid peroxidase antibodies are more specific for Hashimoto's thyroiditis, which usually causes hypothyroidism.
Summarized Points:
A radionuclide iodine-123 thyroid uptake and scan can differentiate between causes of hyperthyroidism.
More Detail
This patient's history, physical examination, and laboratory findings suggest thyrotoxicosis. He has hyperadrenergic
symptoms (palpitations, anxiety, and tremors), and his TSH is suppressed with an elevation in free T4 and total T3 levels.
The next step is to determine the cause of hyperthyroidism. An iodine-123 uptake and scan can help discern the cause, which
would guide treatment. The degree of uptake and its pattern would help to differentiate Graves' disease from a toxic
solitary nodule and toxic multinodular goiter. These would respond to antithyroid medication or radioactive iodine treatment.
However, if there is low uptake, the differential would include subacute thyroiditis or ingestion of exogenous thyroid
hormone, for which antithyroid medications or radioactive iodine therapy would not be useful.
References
Objective number 63 (Neck Mass, Goiter, Thyroid Disease) of Objectives for the Qualifying Examination, 3rd edition. Ross DS.
Diagnosis of hyperthyroidism. Post TW, ed. UpToDate. Waltham, MA: UpToDate Inc. Accessed February 15, 2022.
Question #4
A 29-year-old man is evaluated for gynecomastia present for the past 2 years. He also has fatigue, low libido, weight gain, and
erectile dysfunction. He and his wife have been attempting to become pregnant for the past 12 months without success. He has
no other symptoms and takes no medications or supplements. Vital signs are normal. BMI is 27 kg/m2. Muscle mass is
decreased, and axillary and pubic hair is scant. Breast palpation demonstrates subareolar glandular tissue approximately 1 cm in
diameter in the left breast and 0.8 cm in the right breast. Testicular examination reveals decreased testicular volume. Which of
the following is the most appropriate diagnostic test?
A) Breast biopsy
B) 8 AM Serum testosterone
C) Mammography
D) Testicular ultrasonography
E) Thyroid-stimulating hormone
Explanation
Answer Description
The patient's physical examination is consistent with gynecomastia rather than breast cancer. Male breast cancer would be
suspected if a unilateral, non-tender, fixed breast mass were present with nipple involvement or retraction, overlying skin
changes or ulceration, and axillary lymphadenopathy. A breast biopsy is performed only if a suspicious breast mass was
confirmed on imaging. Mammography is only indicated if physical examination findings are concerning for breast cancer.
Testicular germ cell tumours can cause gynecomastia. When the etiology of gynecomastia is not apparent, evaluation of hCG as
well as a testicular examination for testicular masses should be performed. Testicular ultrasonography is indicated if hCG is
elevated or a testicular mass is palpated on physical examination. In this case, the patient has an apparent cause of
gynecomastia (hypogonadism) and no testicular mass on examination. Therefore, testicular ultrasonography is not indicated.
The patient has classic symptoms of hypogonadism and does not have clinical features of hyperthyroidism (palpitations,
tachycardia, tremors, sweating, weight loss, hyper-defecation); therefore, a thyroid-stimulating hormone level does not need to
be obtained.
Summarized Points:
When the etiology of gynecomastia is not clinically apparent, the initial laboratory evaluation includes measurement of human
chorionic gonadotropin, luteinizing hormone, estradiol, and 8 AM fasting testosterone.
More Detail
In addition to hypogonadism, gynecomastia may be caused by substance use disorders, malnutrition, cirrhosis, testicular germ
cell tumours, hyperthyroidism, and chronic kidney disease. Gynecomastia also may result from the use of medications that affect
androgen or estrogen levels, such as spironolactone, cimetidine, ketoconazole, estrogens, antiandrogens, 5α-reductase
inhibitors, and protease inhibitors, as well as over-the-counter supplements, such as lavender oil and tea tree oil.
A thorough medication/supplement history should be taken in all patients with gynecomastia. When the etiology of gynecomastia
is not clinically apparent, the initial laboratory evaluation includes measurement of human chorionic gonadotropin (hCG),
luteinizing hormone, estradiol, and 8 AM fasting testosterone. The etiology of gynecomastia in this patient is apparent; an early-
morning testosterone level will establish the diagnosis.
References
Objective number 46(Infertility) of Objectives for the Qualifying Examination, 3rd edition. Kanakis GA, Nordkap L, Bang AK et al.
EAA clinical practice guidelines-gynecomastia evaluation and management. Andrology. 2019 Nov;7(6):778-793. Snyder PJ.
Clinical features and diagnosis of male hypogonadism. Post TW, ed. UpToDate. Waltham, MA: UpToDate Inc. Accessed May 05,
2022.
Question #11
A 48-year-old female type 2 diabetic with mild hypertension presents with a 6-week history of tiredness that has come on
gradually; there even are days when she feels that there is no point in getting up in the morning. She has been turning up late for
work, and her supervisor has “written her up” for tardiness. In addition, she states that her clothes have become tighter, that she
has difficulty getting into her shoes, and that her voice seems hoarse. For the past few days, she also has noticed pain in the
right wrist, especially when she uses the computer. She constantly fights with her husband, saying that the bedroom is cold at
night, and insists on using several blankets. Her only medications are metformin for diabetes and hydrochlorothiazide for
hypertension. Her vital signs are as follows: blood pressure, 130/90 mm Hg; pulse 60/min regular; respirations, 16/min; and a
body temperature of 36.50 0C (97.70 0F). She has rather coarse features, her face and eyelids are puffy, and pretibial nonpitting
edema is present. Examination of her neck reveals a diffuse nontender enlargement in the anterior region that moves with
swallowing. No other abnormality is noted. Which of the following set of laboratory test values best supports the proper
diagnosis?
Total Serum
Choice T4
Free Serum T4 Serum TSH
1 ↑ Normal Normal
2 ↑ ↑ ↓
3 ↓ Normal Normal
4 ↓ ↓ ↑
5 ↓ ↓ ↓
The arrow = direction of deviation from normal; T4, thyroxine; T3. triiodothyronine: TSH, thyroid-stimulating hormone
A) 1
B) 2
C) 3
D) 4
E) 5
Explanation
Answer Description
An analytic pattern of increased total T4, normal free T4, and normal TSH values typically is obtained from a person with an
increase in TG levels; for example, a woman taking oral contraceptives.
Increased total T4, increased free T4, and decreased TSH levels represent data that might be presented by a patient with
hyperthyroidism, most commonly due to Graves disease.
A decreased total T4, normal free T4, and normal TSH values is a pattern that would be obtained from a person with a decrease
in TG levels; for example, an athlete taking anabolic steroids.
A decrease in total T4, decrease in free T4, and decrease in TSH values is characteristic of a patient with a problem in either the
anterior pituitary (secondary hypothyroidism) or the hypothalamus (tertiary hypothyroidism), with decreased release of
thyrotropin- releasing hormone (TRH).
Summarized Points:
The clinical vignette describes a patient with hypothyroidism. This diagnosis is confirmed by the laboratory test profile of
decreased total T4, decreased free T4, and increased thyroid-stimulating hormone (TSH) levels.
More Detail
Additional clinical features of hypothyroidism include macroglossia, congestive cardiomyopathy, depression, constipation, slow
mentation, menstrual irregularities, and proximal muscle weakness. Elevated serum creatinine kinase should alert one to muscle
problems. Levothyroxine is the treatment of choice, with a goal of bringing serum TSH into the normal range.
References
Helfand M, Redfern CC. Clinical guideline, part 2. Screening for thyroid disease: an update. American College of Physicians.
Ann Intern Med. 1998 Jul 15. 129(2):144-58. Objective number 63 (Neck Mass, Goiter, Thyroid Disease) of Objectives for the
Qualifying Examination, 3rd edition.
Question #15
A 34-year-old woman complains of severe episodes of headache, tremulousness, palpitations, and anxiety. The patient has
noted a change in her voice, and she has difficulty swallowing solids. On physical examination, there is a palpable, nontender
swelling in front of her neck that moves with deglutition. No cervical lymphadenopathy is noted. Laboratory studies show serum
hypercalcemia. An x-ray film of the cervical region reveals irregular calcification in the mass, while magnetic resonance imaging
(MRI) of the abdomen confirms the presence of bilateral adrenal lesions. Which of the following would be the best screening test
for the thyroid mass in this patient?
Answer Description
Serum thyroid-stimulating hormone (TSH) and serum thyroxine (T4) levels are normal in patients with medullary carcinoma.
Summarized Points:
Multiple endocrine neoplasias type 2 (MEN2) is an inherited disorder characterized by the development of medullary thyroid
cancer(MTC), parathyroid tumors, and pheochromocytoma. MEN2 results from germline mutations in the RET proto-oncogene
and is transmitted in an autosomal dominant fashion. Serum calcitonin is a marker for MTC; levels correlate with tumor mass.
More Detail
This woman has bilateral pheochromocytoma (bilateral adrenal lesions), a parathyroid adenoma (hypercalcemia and ectopic
calcium deposits), and an oversized thyroid (a palpable, nontender swelling in front of her neck). This by definition is an example
of multiple endocrine neoplasias (MEN). There are three such multigland syndromes, called MEN 1 (aka, Wermer syndrome),
MEN 2a (aka, Sipple syndrome), and MEN 2b. All are inherited as autosomal dominant traits.
Among adults worldwide, the prevalence of all three types of MEN is estimated to be between 0.2 and 2 cases per 105
individuals; about 90% of cases are MEN 1, and MEN 2a makes up almost all of the remainder (MEN 2b is extremely rare).
Accordingly, the woman described in the vignette has MEN 2a. Medullary carcinomas of the thyroid derive from C cells, which
synthesize calcitonin. For MEN 2a, serum calcitonin is the best screen.
References
Wells SA Jr, Asa SL, Dralle H, et al. Revised American Thyroid Association guidelines for the management of medullary thyroid
carcinoma. Thyroid. 2015 Jun. 25 (6):567-610. Objective number 63(Neck Mass, Goiter, Thyroid Disease) of Objectives for the
Qualifying Examination, 3rd edition.
Question #18
A 47-year-old woman presents to the clinic complaining of muscle cramps and paresthesias in her hands and feet, as well as
loss of sensation around her mouth. She continues to eat and drink normally. She has no history of diabetes mellitus or
neuropathy. Her medical record has a diagnosis of hyperthyroidism, but she states it was treated with thyroidectomy about one
month ago and is now taking thyroid hormone replacement, which is still being titrated to the appropriate dose. She takes no
other medications. Which of the following is the most likely cause of her presenting symptoms?
A) Hypoparathyroidism
B) Hyperparathyroidism
C) Hypokalemia
D) Hypothyroidism
Explanation
Answer Description
Hypokalemia commonly presents with muscle cramps as described above but not typically with paresthesias or numbness.
Muscle cramps and weakness, rhabdomyolysis, and cardiac dysrhythmias (if severe) are the most common manifestations of
hypokalemia. The degree of symptoms depends upon the severity and the duration of potassium deficiency. Vomiting, diarrhea,
and medications (usually diuretics) are the most frequent causes of hypokalemia, none of which are present in history.
Hypothyroidism most often presents with fatigue, skin changes (brittle hair and nails, dry skin), constipation, bradycardia, and
modest weight gain. True hypothyroidism is diagnosed by an elevated thyroid-stimulating hormone (TSH) level and a low free T4
level. Subclinical hypothyroidism is diagnosed by a mildly elevated TSH level and a normal free T4 level in patients with
symptoms of hypothyroidism.
Summarized Points:
The acute manifestations of hypoparathyroidism (e.g., post-surgical hypoparathyroidism) are due to acute hypocalcemia. The
hallmark of acute hypocalcemia is tetany, which is a disorder of neuromuscular irritability. The symptoms of tetany may be mild
(perioral numbness, paresthesias of the hands and feet, muscle cramps) or severe (carpopedal spasm, laryngospasm, and focal
or generalized seizures, which must be distinguished from the generalized tonic muscle contractions that occur in severe tetany).
The classic physical findings in patients with neuromuscular irritability due to latent tetany are Trousseau's and Chvostek's signs.
More Detail
The correct answer is A.
The clues to the diagnosis in the question stem are muscle cramps, paresthesias in hands and feet, perioral numbness,
and recent thyroidectomy, all suggestive of a diagnosis of hypoparathyroidism. The main electrolyte derangement causing
symptoms in hypoparathyroidism is acute hypocalcemia, which is often manifested as tetany, ranging from mild to severe. Mild
symptoms of tetany include muscle cramps, paresthesias in hands and feet, and perioral numbness, whereas more
severe symptoms of tetany include seizures, carpopedal spasms, and laryngospasms. Severe tetany causes more
neuromuscular disturbances, including generalized tonic muscle contractions. Other patients may have no neuromuscular
symptoms at all and simply have vague complaints, such as anxiety, hyperirritability, fatigue, and depression. In contrast,
chronic hypocalcemia causes cataracts, dental problems, and basal ganglia calcifications, which produce extrapyramidal
symptoms. The diagnosis of hypoparathyroidism is made by a persistently low or inappropriately normal parathyroid hormone
(PTH) level and hyperphosphatemia (without hypomagnesemia).
References
Objective number 12-2 (Calcium Disorders) of Objectives for the Qualifying Examination, 3rd edition. Goltzman D.
Hypoparathyroidism. Post TW, ed. UpToDate. UpToDate Inc. Accessed June 10, 2021.
Question #21
A 42-year-old female presents to your office with a history of fatigue for the past few months. She does not have any other
complaints. After a thorough medical history, she admits to mild constipation that is not troublesome for her. She reports a 2.5 kg
weight gain over the past 6 months, which she attributes to living a more sedentary life of late due to her fatigue and lethargy.
Her physical examination is unremarkable; her thyroid does not seem enlarged, although her face is slightly puffy. Which of the
following is the best next step in management?
A) Free T4 levels
B) Thyroid-stimulating hormone (TSH) levels
C) Thyroglobulin levels
D) Radioactive iodine uptake (RAIU) scan
E) Biopsy
Explanation
Answer Description
Free T4 is important in the diagnostic workup of thyroid disorders and should be the next test ordered if TSH comes back
decreased. If free T4 is increased with a decreased TSH level, this is diagnostic of primary hyperthyroidism. If free T4 is
decreased with a decreased TSH level, then central hypothyroidism is the diagnosis and the etiology involves the pituitary gland
or the hypothalamus. Finally, if free T4 is normal with a decreased TSH, then this is subclinical hypothyroidism.
Thyroglobulin is often increased in goiter and hyperthyroidism and is also a tumor marker for thyroid cancer. It is not the best first
test in working up thyroid disorders.
Radioactive iodine uptake (RAIU) scan is the next best step once primary hyperthyroidism is diagnosed (increased free T4 with
decreased TSH) as it can help differentiate causes of hyperthyroidism (Graves vs. multinodular goiter vs. silent thyroiditis).
Thyroid biopsy is far too invasive of a test without a baseline TSH level.
Summarized Points:
Thyroid-stimulating hormone (TSH) assays are readily available and are generally the most sensitive screening tool for primary
hypothyroidism.
More Detail
References
Dayan CM. Interpretation of thyroid function tests. Lancet 2001;357:619-624. Objective number 63 (Neck Mass, Goiter, Thyroid
Disease) of Objectives for the Qualifying Examination, 3rd edition.
Question #23
A 35-year-old female presents with fatigue, cold intolerance, weight gain, constipation, dry skin, and myalgias. She has no
known medical diagnoses. Menarche was at 13, but she has never had regular menses. She is on no medications and denies
alcohol or drug use. Vital signs are normal except for a heart rate of 47/min. Physical exam reveals an obese female with clear
lung sounds, no murmurs, 2+ non-pitting pedal edema, and dry skin. Which additional abnormality is associated with this clinical
condition?
A) Hypernatremia
B) Hyperlipidemia
C) Microcytic anemia
D) Low serum muscle enzyme concentrations
Explanation
Answer Description
Summarized Points:
Hashimoto's thyroiditis is the most common cause of hypothyroidism. Screening for this disorder includes anti-thyroid peroxidase
(anti-TPO) and antithyroglobulin (anti-Tg) antibodies. Most patients with hypothyroidism have elevations in cholesterol and/or
triglycerides.
More Detail
This patient's presentation is characteristic of hypothyroidism. Common symptoms of thyroid hormone deficiency include
fatigue, cold intolerance, weight gain, constipation, dry skin, myalgia, and menstrual irregularities. Additional findings may include
goiter, bradycardia, hypertension, a delayed relaxation phase in deep tendon reflexes, and numerous others. Most patients with
hypothyroidism have elevations in cholesterol and/or triglycerides. Other conditions caused by hypothyroidism include
hyponatremia (often from inappropriate production of antidiuretic hormone), high serum muscle enzyme concentrations,
normocytic anemia, and pericardial or pleural effusions.
Patients may have had a previous thyroid injury (e.g., radioiodine therapy, thyroid or neck surgery, external radiation therapy),
and may have pituitary or hypothalamic disorders and various autoimmune diseases. The most common cause of
hypothyroidism is Hashimoto's thyroiditis. Laboratory evaluation for this condition includes assays for anti-thyroid peroxidase
(anti-TPO) and antithyroglobulin (anti-Tg) antibodies.
References
Objective number 63 (Neck Mass, Goiter, Thyroid Disease) of Objectives for the Qualifying Examination, 3rd edition. Wilson SA,
Stem LA, Bruehlman RD. Hypothyroidism: Diagnosis and Treatment. Am Fam Physician. 2021 May 15;103(10):605-613. Surks
MI. Clinical manifestations of hypothyroidism. Post TW, ed. UpToDate. Waltham, MA: UpToDate Inc. Accessed October 12,
2020.
Question #24
A 29-year-old woman is evaluated for a 6-month history of amenorrhea. Medical history is significant for schizophrenia. Her only
medication is risperidone. On physical examination, vital signs are normal. BMI is 25 kg/m2. No breast discharge is evident.
Laboratory studies:
MRI reveals a normal pituitary gland. Consultation with the patient's psychiatrist confirms that risperidone cannot be
discontinued. Which of the following is the most appropriate management?
A) Begin cabergoline
B) Repeat pituitary MRI in 6 months
C) Repeat prolactin measurement in 6 months
D) Begin estrogen-progesterone replacement therapy
Explanation
Answer Description
Initiating a dopamine agonist such as cabergoline to treat medication-induced hyperprolactinemia in cases such as described for
this patient is not recommended because it can induce psychosis.
This patient has an estrogen deficiency that requires estrogen-progesterone replacement to avoid long-term consequences.
Additionally, the elevated prolactin is unlikely to improve if the patient continues taking risperidone, and the pituitary MRI is
unlikely to change. If possible and in consultation with the patient's psychiatrist, prolactin levels should be rechecked after either
the causative medication has been withheld for 3 days or switched to medication less likely to cause hyperprolactinemia.
However, in this case, the patient's psychiatrist confirms that continuing risperidone is necessary; therefore, estrogen-
progesterone replacement therapy is indicated.
Summarized Points:
Although medication-induced hyperprolactinemia most often results in prolactin levels of 25-100 µg/L, drugs such as
metoclopramide, risperidone, and phenothiazines can lead to prolactin levels greater than 200 µg/L. Untreated
hyperprolactinemia can lead to hypogonadism and bone loss.
More Detail
This patient has risperidone-induced hyperprolactinemia. Symptoms of hyperprolactinemia in women include amenorrhea
and in some cases galactorrhea. The most common cause of non-tumour-related hyperprolactinemia is medication. Up to 40%
of patients taking typical antipsychotics will develop hyperprolactinemia because of the dopamine antagonist effect. Although
medication-induced hyperprolactinemia most often results in prolactin levels of 25-100 µg/L, drugs such as metoclopramide,
risperidone, and phenothiazines can lead to prolactin levels greater than 200 µg/L. Confirmation of medication-induced
hyperprolactinemia is often challenging. A pituitary-specific MRI is often necessary to ensure that an adenoma is not present. If
this patient's hyperprolactinemia remains untreated, she will continue to have amenorrhea and bone loss over time because of
estrogen deficiency. Estrogen-progesterone replacement therapy is necessary to avoid the sequelae of estrogen deficiency.
References
Objective number 56-1(Amenorrhea, Oligomenorrhea) of Objectives for the Qualifying Examination, 3rd edition. Montejo ÁL,
Arango C, Bernardo M, et al. Multidisciplinary consensus on the therapeutic recommendations for iatrogenic hyperprolactinemia
secondary to antipsychotics. Front Neuroendocrinol. 2017 Apr;45:25-34. Snyder PJ. Management of hyperprolactinemia. Post
TW, ed. UpToDate. Waltham, MA: UpToDate Inc. Accessed June 30, 2022.
Question #25
A 26-year-old woman comes to the office due to palpitations, increased frequency of bowel movements, and a 4.5-kg (10-1b)
weight loss despite adequate food intake. The patient’s symptoms also include redness of the eyes associated with tearing,
grittiness, and photophobia, as well as intermittent diplopia when reading or watching television. Blood pressure is 138/66 mm
Hg and pulse is 110/min and regular. The patient appears anxious. The thyroid gland shows diffuse symmetrical enlargement
without tenderness. A thyroid bruit is audible. Ocular examination shows bilateral proptosis with conjunctival injection and
periorbital edema. Laboratory evaluation shows:
The patient is started on a beta-blocker and oral prednisone. Which of the following additional interventions should be initiated at
this time?
A) Propylthiouracil
B) Radioactive iodine
C) Selenium
D) Thyroidectomy
E) Methimazole
Explanation
Answer Description
MMI is preferred over PTU due to its longer duration of action (once-daily dosing for most), faster resolution of hyperthyroidism,
and lower incidence of side effects.
Radioactive iodine (RAI) ablation can be considered for definitive therapy in patients with moderate to severe Graves’ disease, in
whom an ATD alone is unlikely to achieve remission. However, RAI can worsen ophthalmopathy and should be avoided in
patients with significant ocular symptoms.
Selenium deficiency is associated with an increased risk of Graves ophthalmopathy. Supplementation appears to reduce
symptoms in patients with mild ophthalmopathy, but the benefits are uncertain in regions where selenium deficiency is
uncommon, including the United States.
Thyroidectomy is the preferred definitive therapy for patients with large goiters, obstructive symptoms, or suspected thyroid
cancer. It is also recommended for those with significant ophthalmopathy, in whom RAI may cause an exacerbation of
symptoms. However, thyroidectomy is usually performed with the patient in a euthyroid state on an ATD.
Summarized Points:
Antithyroid drugs (e.g., methimazole) can induce remission in patients with mild Graves’ disease. They are also used to attain a
euthyroid state in preparation for definitive treatment with radioactive iodine or thyroidectomy in patients with moderate to severe
disease. Radioactive iodine can worsen Graves ophthalmopathy and should be avoided in patients with significant ocular
symptoms.
More Detail
This patient has hyperthyroidism associated with diffuse goiter and increased radioiodine uptake consistent with Graves’
disease. She also has moderate Graves ophthalmopathy, with ocular irritation, photophobia, and diplopia. Initial management of
patients with hyperthyroidism should include a beta-blocker (e.g., propranolol) to control adrenergic symptoms. Systemic
glucocorticoids are indicated for those with moderate (oral) or severe/progressive (intravenous) ophthalmopathy. Smoking
cessation is also advised as smoking is associated with increased progression of ophthalmopathy. In addition, this patient should
be started on an antithyroid drug (ATD) to inhibit the synthesis of thyroid hormone. ATDs can be used to achieve remission in
patients with mild Graves’ disease, or to attain a euthyroid state in preparation for more definitive treatment in those with
more significant disease (such as this patient). Methimazole (MMI) is usually preferred due to the risk of hepatotoxicity with
propylthiouracil (PTU), although propylthiouracil is recommended in the first trimester of pregnancy due to the teratogenic effects
of methimazole.
References
Objective number 63 (Neck Mass, Goiter, Thyroid Disease) of Objectives for the Qualifying Examination, 3rd edition. Ross DS,
Burch HB, Cooper DS, et al. 2016 American Thyroid Association Guidelines for Diagnosis and Management of Hyperthyroidism
and Other Causes of Thyrotoxicosis. Thyroid 2016; 26:1343.
Question #27
A 32-year-old woman, gravida 1 para 1, comes to the office for her 2-month follow-up on her lab results. She was diagnosed with
postpartum thyroiditis 2 months ago and since then has been taking propranolol. Laboratory results are as follows:
The patient concern about her condition, which of the following would be the long-term consequences of her postpartum
thyroiditis?
Ophthalmopathy is a significant finding in patients with Graves’ disease but not PT.
Because patients with PT are unlikely to develop chronic hyperthyroidism, there is no indication for radioiodine therapy or
thyroidectomy.
PT is not associated with thyroid cancer. Patients with classic chronic lymphocytic (Hashimoto) thyroiditis have a small risk of
thyroid lymphoma, but this appears to be rare in PT.
Patients with a history of type 1 diabetes mellitus are at increased risk of developing postpartum thyroiditis. However, a
previously healthy woman who develops postpartum thyroiditis is not at increased risk of developing diabetes.
Summarized Points:
Any woman who has had postpartum thyroiditis should be told that recurrence is likely after future pregnancies and that she is at
substantial risk for the later development of hypothyroidism or goiter, and she should be educated regarding the possible
development of symptoms. For women who have fully recovered from postpartum thyroiditis, it is recommended to measure
serum TSH levels annually, particularly within 5 to 10 years after the initial diagnosis.
More Detail
However, a significant minority of patients (20%-40%) may develop persistent or recurrent hypothyroidism with a palpable
goiter, similar to classic chronic lymphocytic (Hashimoto) thyroiditis. Patients with a more severe initial hypothyroid phase or a
high titer of antithyroid peroxidase antibody are at increased risk (as in this patient). In light of this, patients with PT
should be counselled on the symptoms of hypothyroidism, and some experts advise annual measurement of TSH.
References
Objective number 63 (Neck Mass, Goiter, Thyroid Disease) of Objectives for the Qualifying Examination, 3rd edition. Burman
KD. Postpartum thyroiditis. Post TW, ed. UpToDate. UpToDate Inc. Accessed June 24, 2021.
Question #32
A 32-year-old woman comes to the office with a small lump in her neck. She has lost 2.3 kg (5 lb) over the last 6 months but has
no other symptoms. The patient has no personal or family history of significant medical conditions. Serum TSH level is 0.01 µU/
mL (0.4-5). Thyroid ultrasound reveals a 1.1-cm, round, hypoechoic nodule in the right lobe of the thyroid. There is no lymph
node enlargement. Which of the following is the most appropriate course of action for this patient?
Answer Description
Antithyroid peroxidase antibodies suggest chronic lymphocytic (Hashimoto) thyroiditis and related disorders. They are most often
assayed in patients with goiter and elevated TSH. However, these antibodies do not exclude malignancy or alter indications for
scintigraphy or FNA.
Choice C is not correct:
If the serum TSH concentration is normal or elevated, and the nodule meets sonographic criteria for sampling, a palpation or
ultrasound-guided fine-needle aspiration (FNA) biopsy should be performed. FNA biopsy is the most accurate method for
evaluating thyroid nodules and selecting patients for thyroid surgery. Because this patient has a low TSH, thyroid scintigraphy is
the best option. Fine-needle aspiration is an unnecessary invasive test at this stage.
Repeat thyroid ultrasound to assess stability is indicated for patients who have hypofunctioning nodules that do not meet
threshold indications for FNA. However, if this patient has a hyperfunctioning nodule on scintigraphy, FNA and repeats
ultrasound are not indicated.
Radioiodine ablation or surgical lobectomy should be considered for hyperfunctioning nodules, but scintigraphy should be
performed first. Surgery is also indicated for patients with compressive symptoms, nondiagnostic cytology on multiple FNAs, or
FNA showing cytologic features of malignancy.
Summarized Points:
TSH measurement and thyroid ultrasound should be performed in all patients with thyroid nodules. Low/suppressed TSH
suggests a hyperfunctioning nodule, which should be confirmed with thyroid scintigraphy. Hyperfunctioning nodules are usually
benign.
More Detail
The initial evaluation of thyroid nodules should include a thyroid ultrasound and serum TSH measurement to determine the
likelihood of malignancy and the need for fine-needle aspiration (FNA) biopsy. Hyperfunctioning nodules are rarely
malignant. Therefore, if the initial TSH level is suppressed, as in this patient, radioiodine (or technetium) thyroid
scintigraphy (i.e., nuclear medicine thyroid scan) should be used to assess the metabolic activity of the nodule. If a
hyperfunctioning ("hot") nodule is confirmed on scintigraphy, FNA is not required.
Hypofunctioning ("cold") or indeterminate nodules have a higher malignancy risk and may require FNA based on size, ultrasound
findings, and patient characteristics:
• High-risk findings on ultrasound include microcalcifications, increased vascularity, hypoechoic appearance, elongated
shape, and indistinct or irregular nodule margins. FNA is indicated in such nodules ≥1 cm.
• Low-risk nodules on ultrasound (e.g., isoechoic, hyperechoic, partially cystic, spongiform) are monitored and biopsied
if they grow to ≥5-2 cm. Purely cystic nodules are almost always benign and do not require FNA.
• High-risk patient characteristics include a personal or first-degree family history of thyroid cancer, history of radiation
exposure in childhood, rapid nodule growth, and cervical lymphadenopathy. FNA should be considered for any size of
nodule in this population.
References
Objective number 63(Neck Mass, Goiter, Thyroid Disease) of Objectives for the Qualifying Examination, 3rd edition. Haugen BR,
Alexander EK, Bible KC, et al. 2015 American Thyroid Association Management Guidelines for Adult Patients with Thyroid
Nodules and Differentiated Thyroid Cancer: The American Thyroid Association Guidelines Task Force on Thyroid Nodules and
Differentiated Thyroid Cancer. Thyroid 2016; 26:1.
Question #33
A 57-year-old man is evaluated for decreased libido. He has normal erectile function and has fathered three children. He has
hypertension, type 2 diabetes mellitus, and hyperlipidemia. Medications are hydrochlorothiazide, metformin, and atorvastatin.
Vital signs are normal. BMI is 40 Kg/m2. The remainder of the physical examination is unremarkable. The 8 AM total
testosterone level is 6.0 nmol/L (6.1–27.1). Which of the following is the most appropriate next step?
Hemochromatosis, pituitary adenoma, and sleep apnea are all common causes of secondary hypogonadism (hypothalamic or
pituitary dysfunction). This patient, however, does not have confirmed hypogonadism; therefore, evaluation for these causes with
measurement of serum iron and total iron-binding capacity, a pituitary MRI, or a sleep study is premature and not indicated at
this time.
Testosterone replacement therapy is only indicated for men with biochemically proven hypogonadism. Measurement of 8 AM
fasting total testosterone level on two occasions is recommended in men with specific signs and symptoms of hypogonadism,
such as decreased morning spontaneous erections, decreased libido, infertility, mastodynia, gynecomastia, and decreased facial
hair or decreased axillary and genital hair. Laboratory evaluation of men with nonspecific symptoms of hypogonadism is not
recommended. Examples of nonspecific symptoms include decreased mood, energy, concentration, muscle strength and bulk,
and stamina, as well as poor sleep and memory. This patient does not have biochemically proven hypogonadism at this point in
the evaluation, so testosterone therapy should not be initiated.
Summarized Points:
Obesity lowers sex hormone-binding globulin levels and leads to a falsely low level of total testosterone, which measures free
plus protein-bound testosterone. In patients with low total serum testosterone and suspected low sex hormone-binding globulin,
free testosterone should be measured.
More Detail
Obesity lowers sex hormone-binding globulin (SHBG) and leads to a falsely low total testosterone level (which measures
free plus protein-bound testosterone); the free testosterone concentration remains normal. Measurement of free testosterone
in a laboratory capable of performing the equilibrium dialysis assay is recommended to distinguish between a suspected binding
abnormality and hypogonadism in a man with obesity. The binding abnormality caused by obesity is proportional to the degree of
obesity. If free testosterone is low, a serum luteinizing hormone (LH) measurement is indicated.
An elevated LH level and low free testosterone reflect primary hypogonadism (testicular failure), and further evaluation
should be directed toward identifying the cause. A low or normal LH level with simultaneous low free testosterone reflects
secondary hypogonadism (hypothalamic or pituitary dysfunction). Several other causes of abnormal SHBG levels should also
be considered in the evaluation for hypogonadism. Increased SHBG levels may be associated with aging, hyperthyroidism, high
estrogen concentrations, liver disease, HIV infection, and antiseizure drugs. In addition to obesity, reduced SHBG levels may be
associated with insulin resistance, type 2 diabetes mellitus, hypothyroidism, growth hormone excess, exogenous androgens or
anabolic steroids, glucocorticoids, progestins, and nephrotic syndrome.
References
Marcelli M, Mediwala SN. Male hypogonadism: a review. J Investig Med. 2020 Feb;68(2):335-356. Snyder PJ. Clinical features
and diagnosis of male hypogonadism. Post TW, ed. UpToDate. Waltham, MA: UpToDate Inc. Accessed May 05, 2022.
Question #39
A 65-year-old woman is seen in the outpatient clinic for follow-up. The patient has no current symptoms. She has a history of
hypothyroidism and takes levothyroxine replacement therapy. The patient's other medical problems include hypertension and
acid reflux disease. She underwent dual-energy x-ray absorptiometry 2 months ago, which was normal. Her TSH level is <0.01
mIU/L (0.4‐5.0). Despite a long discussion, the patient refuses to decrease the dose of levothyroxine as lower doses make her
feel "exhausted and sleepy all the time." Which of the following is a possible complication of levothyroxine overreplacement in
this patient?
A) Dementia
B) Hyperlipidemia
C) Ophthalmopathy
D) Cardiac arrhythmia
E) Thyroid cancer
Explanation
Answer Description
There is no proven association of dementia with thyrotoxicosis. Patients with severe thyrotoxicosis may have acute reversible
changes in mental status ranging from mild inattention to loss of consciousness.
Ophthalmopathy, characterized by an increase in volume of retro-orbital tissues with proptosis, is an extrathyroidal manifestation
of Graves' disease but is not seen in other causes of thyrotoxicosis. Lid lag and lid retraction without proptosis can be seen with
thyrotoxicosis due to any cause.
A high TSH level is a risk factor for thyroid cancer, but a low TSH level may be protective.
Summarized Points:
Overtreatment of hypothyroidism with levothyroxine is associated with an increased risk of excessive bone loss and atrial
fibrillation.
More Detail
A number of serious adverse effects are associated with overtreatment of hypothyroidism with levothyroxine, including
excessive bone loss and cardiac arrhythmias (e.g., atrial fibrillation). The risk of bone loss and arrhythmias from
overtreatment is higher in older patients, particularly those with markedly suppressed TSH (<0.1 mIU/L). Nonspecific
symptoms (e.g., fatigue, weight gain) in patients with hypothyroidism are frequently attributed to undertreatment, and patients
often request supratherapeutic dosing of thyroid supplements to increase energy and facilitate weight loss. As a result,
overtreatment of hypothyroidism is not uncommon in clinical practice but should be discouraged. Also, the addition of
liothyronine in a patient who is biochemically euthyroid on levothyroxine is not associated with improved outcomes and is not
indicated. However, dosing of levothyroxine to maintain TSH in the low-normal range is not associated with adverse effects and
may be adequate to satisfy patient concerns.
References
Objective number 33(Fatigue) of Objectives for the Qualifying Examination, 3rd edition. Thayakaran R, Adderley NJ. Thyroid
replacement therapy, thyroid stimulating hormone concentrations, and long term health outcomes in patients with
hypothyroidism: longitudinal study. BMJ. 2019 Sep 3;366:l4892.
Question #1
A 30-year-old woman returns to the clinic for a blood test follow-up. She came two weeks ago with symptoms such as agitation,
diarrhea, and loss of weight. She lost weight without making any dietary changes. She is happy that her recent pregnancy test
came back positive. Her blood pressure is 145/85 mm Hg and pulse is 115/minute. Physical examination shows a resting tremor,
pretibial myxedema, and a palpable neck mass. Which of the following is used to treat symptomatic hyperthyroidism in a patient
with Graves' disease during the first trimester of pregnancy?
A) Methimazole
B) Radioactive therapy
C) Thyroidectomy
D) Propylthiouracil
Explanation
Answer Description
Methimazole crosses the placenta. Birth defects have been observed in neonates exposed to maternal methimazole in the first
trimester of pregnancy and include anomalies of the upper gastrointestinal tract (esophageal atresia or tracheoesophageal
fistula), respiratory tract (choanal atresia), skin (aplasia cutis), and facial dysmorphism.
For women with symptomatic, moderate-to-severe, overt hyperthyroidism who cannot tolerate thioamides because of allergy or
agranulocytosis, thyroidectomy in the second trimester may be necessary.
Summarized Points:
Women diagnosed with symptomatic, moderate-to-severe hyperthyroidism during the first trimester of pregnancy should take
PTU. Patients may continue PTU for the remainder of pregnancy or switch back to methimazole at 16 weeks. All three
antithyroid drugs (ATDs) have been associated with possible teratogenic effects, but teratogenic effects are more severe with
methimazole and carbimazole compared with PTU. There are numerous case series of aplasia cutis, a scalp defect, in newborns
exposed to methimazole in utero.
More Detail
Women with symptomatic hyperthyroidism due to Graves' disease require treatment of hyperthyroidism during pregnancy. Such
patients will almost always have thyroid-stimulating hormone (TSH) values below 0.05 µU/L and elevations in trimester-specific
free T4 concentrations or total T4 and T3 concentrations that exceed 1.5 times the upper limit of normal for nonpregnant
patients. For women with moderate-to-severe hyperthyroidism complicating pregnancy, propylthiouracil (PTU) is
recommended in the first trimester while methimazole is recommended for women who present in the second or third
trimester. Patients taking PTU during the first trimester can either switch to methimazole after 16 weeks of gestation or continue
PTU throughout pregnancy.
References
Objective number 63 (Neck Mass, Goiter, Thyroid Disease) of Objectives for the Qualifying Examination, 3rd edition. Ross DS.
Hyperthyroidism during pregnancy: Treatment. Post TW, ed. UpToDate. UpToDate Inc. Accessed September 03, 2020.
Question #10
A 37-year-old woman is being evaluated for obesity. She says she has gradually been gaining weight in the past three years.
She has no significant past medical history and does not take any medications. She is 64 inches tall and weighs 225 pounds.
Her body mass index is 38.6 kg/m2. Which of the following is the most appropriate initial treatment for this patient's condition?
A) Bariatric surgery
B) Cognitive behavioral therapy
C) Lifestyle changes
D) Liraglutide
E) Orlistat
Explanation
Answer Description
Bariatric surgery, such as gastric bypass, is generally reserved for patients who fail lifestyle and dietary modifications.
Additionally, bariatric surgery should only be offered to patients with a body mass index ≥ 40 kg/m2 (without comorbidities). Since
this patient has a body mass index of 38.6 kg/m2, bariatric surgery is not recommended at this time.
Cognitive-behavioral therapy can be helpful in patients with eating disorders, such as anorexia nervosa and bulimia nervosa.
Although behavioral modification can be used in patients with obesity, lifestyle modifications should be implemented first.
There are currently three prescription medications available in Canada that are approved for use in adult patients with BMI ≥30
kg/m2 or BMI ≥27 kg/m2 and ≥1 weight-related condition (e.g., hypertension, type2-diabetes mellitus, dyslipidemia) who has
failed 3-6 months lifestyle modification. Liraglutide, Orlistat, and Naltrexone /bupropion can be used in obese patients (as in this
patient) but should only be considered after a trial of lifestyle modification, diet, and exercise have failed.
Summarized Points:
Obesity management and therapy aim to avoid, reverse, or decrease the complications of obesity while increasing quality of life.
Obese patients should first be counseled on adopting lifestyle changes, such as exercise and dietary modifications.
More Detail
Obesity is a chronic metabolic disorder that is associated with increased morbidity and mortality. Cancer, diabetes mellitus,
hypertension, dyslipidemia, cardiovascular disease, and other metabolic disorders have all been associated with obesity.
Obesity is defined as a body mass index ≥ 30 kg/m2. The goal of the management and treatment of obesity is to prevent,
reverse, or lessen the complications of obesity while improving the quality of life. Obese patients should initially be counseled on
making lifestyle changes, including exercise and dietary modification.
References
Objective number 118-1 (Weight Gain, Obesity) of Objectives for the Qualifying Examination, 3rd edition. Wharton S, Law DCW,
Vallis M, et al. Obesity in adults: a clinical practice guideline. CMAJ 2020;192:E875-E891.
Question #13
A 65-year-old man develops truncal obesity and a buffalo hump. He is not on any medications. Early morning cortisol is 610
nmol/L (185–624 ). Late evening cortisol is 280 nmol/L (≤ 276 ). Cortisol production does not suppress with either low or high
dexamethasone. CT scan of the head demonstrates a normal sella turcica and pituitary gland. An ectopic source of ACTH
production is suspected. A malignancy of which of the following organs is most likely to be the source of the ACTH?
A) Colon
B) Lung
C) Pancreas
D) Stomach
E) Testis
Explanation
Answer Description
Cancers of the colon can cause Gastrointestinal(GI) symptoms and anemia related to blood loss but are not a prominent cause
of endocrine-related paraneoplastic syndromes.
Islet cell cancers of the pancreas can secrete clinically significant amounts of insulin, gastrin., glucagon, and vasoactive intestinal
peptide, but do not usually secrete ACTH or corticosteroids.
Cancers of the stomach can cause symptoms such as jaundice and ascites related to direct extension into perigastric sites but
are not a source of endocrine-related paraneoplastic syndromes.
While some testicular cancers can secrete tiny amounts of hormones such as human chorionic gonadotropin (hCG), clinically
significant endocrine disease does not usually occur.
Summarized Points:
Cushing’s syndrome can result from ectopic ACTH production due to small cell lung cancer.
More Detail
Cushing syndrome is a cluster of clinical abnormalities seen in the setting of chronic exposure to high cortisol levels. Truncal
obesity, buffalo hump, increased susceptibility to infection, hypertension, and glucose intolerance are common manifestations.
Cushing syndrome can be due to administration of exogenous corticosteroids, adrenal hyperplasia or tumor, pituitary adenoma,
or ectopic ACTH production by another malignancy. The latter is most commonly a small cell carcinoma of the lung. Lung
cancers are also notorious for a wide variety of other paraneoplastic syndromes.
References
Carey RM, Varma SK, Drake CR Jr, Thorner MO, Kovacs K, Rivier J, et al. Ectopic secretion of corticotropin-releasing factor as a
cause of Cushing's syndrome. A clinical, morphologic, and biochemical study. N Engl J Med. 1984 Jul 5. 311 (1):13-20. Objective
number 43 (Hirsutism, Hypertrichosis) of Objectives for the Qualifying Examination, 3rd edition.
Question #16
A 51-year-old woman with newly diagnosed hypertension is started on chlorthalidone therapy. Thiazide diuretics are most likely
to exacerbate which of the following disorders?
A) Adrenal insufficiency
B) Crohn disease
C) Hypothyroidism
D) Osteoporosis
E) Primary hyperparathyroidism
Explanation
Answer Description
Adrenal insufficiency may be provoked by medications that inhibit cholesterol biosynthesis (e.g., ketoconazole, etomidate),
increase metabolic deactivation of cortisol (e.g., phenytoin, rifampin), or suppress adrenocorticotrophic hormone and
corticotrophin-releasing hormone (e.g., prednisone, opioids).
Nonsteroidal anti-inflammatory drugs, tetracyclines, and oral contraceptive pills have been implicated in precipitating or causing
inflammatory bowel disease.
Hypothyroidism can be caused by several medications, including lithium, amiodarone, oral tyrosine kinase inhibitors (e.g.,
sunitinib), and certain antiepileptics (e.g., carbamazepine, phenytoin).
Glucocorticoids, antiepileptics, and antidepressants (tricyclics, selective serotonin reuptake inhibitors) have all been associated
with early-onset osteoporosis. However, the decreased urinary excretion of calcium seen with thiazide use may actually slow
osteoporosis progression.
Summarized Points:
Thiazide diuretics cause a decrease in urinary calcium excretion and can result in mild hypercalcemia. In patients with
preexisting primary hyperparathyroidism, thiazide diuretics can result in additional elevation of serum calcium concentrations and
"unmask" the hyperparathyroidism.
More Detail
The correct answer is E.
Thiazide diuretics cause a decrease in urinary calcium excretion and can cause mild hypercalcemia. In patients with
undiagnosed primary hyperparathyroidism, the use of thiazide diuretics can lead to a substantial elevation of serum calcium
concentration and unmask the hyperparathyroidism. In such cases, the thiazide should be discontinued; measurement of
calcium and parathyroid hormone months after discontinuation can confirm the diagnosis of hyperparathyroidism.
Thiazides can also cause hyponatremia and hypokalemia. In patients with mild or undiagnosed hyperaldosteronism, potassium
levels may be normal at baseline, but the addition of a thiazide can trigger severe hypokalemia and unmask this condition as
well.
References
Objective number 12-2 (Calcium Disorders) of Objectives for the Qualifying Examination, 3rd edition. Griebeler ML, Kearns AE.
Thiazide-Associated Hypercalcemia: Incidence and Association With Primary Hyperparathyroidism Over Two Decades. J Clin
Endocrinol Metab. 2016 Mar;101(3):1166-73.
Question #23
A 55-year-old woman comes to the office to follow up on elevated blood pressure, which was 140/100 mmHg during a routine
preventive visit 4 weeks ago. The patient feels well and has experienced no recent weight changes, headaches, chest pain,
palpitations, or polyuria. Medical history is notable only for gastroesophageal reflux disease managed with dietary modification,
and she takes no medications. Screening colonoscopy at age 50 was normal. She drinks 1 or 2 glasses of wine most days and
does not use tobacco or illicit drugs. The patient is an only child, and her parents have no unusual health history. Blood pressure
is 138/100 mm Hg and pulse is 72/min. BMI is 27 kg/m2. The patient appears mildly overweight but comfortable. Physical
examination is otherwise unremarkable. Laboratory results are as follows:
Serum chemistries
Renal ultrasonography is useful to evaluate hypertension in patients with diffuse atherosclerosis or evidence of renovascular
hypertension (e.g., atrophic kidney, recurrent flash pulmonary edema, abdominal bruit, onset of severe hypertension at age >55)
but is not needed in patients with uncomplicated hypertension.
This patient has a normal serum calcium level, making hyperparathyroidism less likely. Although hypertension has been
associated with hyperparathyroidism (including normocalcemic hyperparathyroidism) in some studies, this is an inconsistent
observation.
Testing for primary hyperaldosteronism with plasma aldosterone to plasma renin activity ratio is appropriate for patients with
resistant hypertension (i.e., uncontrolled despite an appropriate multidrug regimen) or hypertension with significant hypokalemia.
Screening for pheochromocytoma with plasma fractionated metanephrines is appropriate for patients with severe, labile, or
resistant hypertension. These tests are not needed for routine evaluation of uncomplicated hypertension.
Summarized Points:
Both hyperthyroidism and hypothyroidism can raise blood pressure. Untreated thyroid disease can impair the response to
antihypertensive management, but many patients experience normalization of blood pressure when the underlying thyroid
disorder is treated. Patients with hypertension warrant screening for thyroid disease in the initial laboratory evaluation.
More Detail
Both hypothyroidism and hyperthyroidism can raise blood pressure: Hyperthyroidism causes increased systolic and pulse
pressure, whereas hypothyroidism typically causes increased diastolic pressure with decreased pulse pressure; thyroid
disease can also lead to lipid abnormalities. Untreated thyroid disease can impair the response to antihypertensive management,
but the treatment of the underlying thyroid condition can often normalize blood pressure without additional antihypertensive
therapy.
Therefore, according to American College of Cardiology/American Heart Association guidelines, patients with hypertension
warrant screening for thyroid disease with a serum TSH as part of the initial laboratory evaluation (regardless of symptoms)
and if they have clinical features of thyroid disease (e.g., heat/cold intolerance, weight loss/gain, warm/cold skin).
References
Objective number 63(Neck Mass, Goiter, Thyroid Disease) of Objectives for the Qualifying Examination, 3rd edition. Nerenberg
KA, Zarnke KB, Leung AA, et al. Hypertension Canada’s 2018 Guidelines for diagnosis, risk assessment, prevention, and
treatment of hypertension in adults and children. Can J Cardiol 2018;34(5):506-525.
Question #24
A 58-year-old man comes to the office due to fatigue and inability to maintain satisfactory erections during intercourse. He has
also noticed decreased early-morning erections and sexual desire. The patient has had no headaches, visual symptoms,
polyuria, head injuries, or motor weakness. He has been trying to lose weight with diet and exercise. Medical history is notable
for hypertension, and current medications include lisinopril and aspirin. Blood pressure is 142/74 mm Hg and pulse is 78/min.
BMI is 38 kg/m2. Body and facial hair distribution are normal. Visual fields are normal on confrontation. Testes are normal in size
but have a soft consistency. The remainder of the physical examination is normal. Laboratory results are as follows:
Which of the following is the best next step in the evaluation of this patient's symptoms?
A) Total testosterone level
B) LH and FSH levels
C) MRI of the pituitary
D) Nocturnal penile tumescence
E) Free testosterone level
Explanation
Answer Description
If testosterone is low, serum FSH and LH levels can distinguish primary (elevated LH and FSH) from secondary (low/normal FSH
and LH) hypogonadism. MRI of the pituitary should be considered in patients with secondary hypogonadism and for those with
severe hypogonadism (e.g., testosterone <150 ng/mL), deficiencies of additional pituitary hormones (e.g., TSH, growth
hormone), hyperprolactinemia, or mass effect symptoms (e.g., visual field defects).
Nocturnal penile tumescence can be useful in differentiating psychogenic (normal) and organic (reduced tumescence) causes of
erectile dysfunction. However, it is not commonly used in clinical practice, and the decreased morning erections in this patient
suggest an organic cause.
Although obesity and diabetes are associated with true hypogonadism, these conditions are also associated with decreased
levels of sex hormone-binding globulin, leading to spuriously low total testosterone levels. If the initial total testosterone level is
low, measurement of free testosterone levels should be considered to confirm true hypogonadism in patients with these
conditions; however, testing is not recommended in the initial workup of hypogonadism because it requires specialized assays
that can be expensive and inaccurate and are not always readily available.
Summarized Points:
Clinical features that warrant evaluation for androgen deficiency include decreased libido, erectile dysfunction, testicular atrophy,
loss of body hair, gynecomastia, hot flashes, and an unexplained decrease in bone density. Nonspecific symptoms (e.g., fatigue,
weakness, weight changes) alone do not warrant testing in most cases. The initial step is a morning serum testosterone level,
and low levels should be confirmed on repeat testing.
More Detail
Other specific features that should prompt evaluation for androgen deficiency include loss of body hair, gynecomastia, hot
flashes, and an unexplained decrease in bone density. In contrast, nonspecific symptoms (e.g., fatigue, weakness, weight
changes) alone do not warrant testing in most cases. The initial step in the evaluation of male hypogonadism is a morning
serum total testosterone level. However, testosterone shows significant variability, and low levels should be confirmed on
repeat testing.
References
Objective number 46 (Infertility) of Objectives for the Qualifying Examination, 3rd edition. Bhasin S, Brito JP, Cunningham GR, et
al. Testosterone Therapy in Men With Hypogonadism: An Endocrine Society Clinical Practice Guideline. J Clin Endocrinol Metab
2018; 103:1715.
Question #34
A 68-year-old man is evaluated after a chest CT scan incidentally revealed changes of the posterior left ninth rib consistent with
Paget's disease of bone. The remainder of the thoracic skeleton is normal. On physical examination, inspection and palpation of
the posterior chest wall are unremarkable. Laboratory studies:
A) Bone biopsy
B) Alkaline phosphatase isoenzyme test
C) Bone mineral density measurement
D) 1,25-Dihydroxy vitamin D measurement
E) Whole-body radionuclide bone scan
Explanation
Answer Description
Because the radiographic phenotype of Paget's disease of bone is diagnostic, bone biopsy is not required unless an atypical
presentation or radiographic appearance is found.
Serum alkaline phosphatase, a marker of increased bone formation, is a tissue-nonspecific enzyme. Differentiation of alkaline
phosphatase isoenzymes may identify the primary contributor to the elevated total. Disorders of the bone and liver most
commonly cause elevations in alkaline phosphatase levels. Considering the normal aminotransferase and bilirubin levels and
abnormal CT scan, the alkaline phosphatase elevation in this patient is likely secondary to Paget's disease of bone. A whole-
body bone scan is required regardless of the alkaline phosphatase isoenzyme levels.
Paget's disease of bone typically affects older adults, who have more risk factors for osteoporosis. Although measurement of
bone mineral density may be indicated for other reasons, it is unnecessary for the evaluation and management of Paget's
disease of bone.
Alkaline phosphatase may be elevated in other conditions, including vitamin D deficiency, other metabolic bone diseases such as
osteomalacia, or recent fracture. Therefore, all patients with suspected Paget disease of bone require assessment of serum
calcium and 25-hydroxyvitamin D. 25-Hydroxyvitamin D is the storage form of vitamin D in the body, and measurement of
25-hydroxyvitamin D is the most appropriate test for assessing vitamin stores. However, 1,25-dihydroxy vitamin D is more
reflective of kidney function and parathyroid hormone level and is, therefore, an inappropriate test for this patient. Finally,
treatment of Paget's disease of bone comprises potent antiresorptive drugs, including intravenous bisphosphonate therapy;
these drugs can cause hypocalcemia, especially if calcium or vitamin D deficiency or kidney disease is present at baseline.
Therefore, an assessment of 25-hydroxyvitamin D levels, calcium, and kidney function should be performed before initiating
therapy.
Summarized Points:
The radiographic phenotype of Paget's disease of bone is diagnostic, and bone biopsy is not required. In patients with Paget's
disease of bone, a bone scan followed by focused radiography of abnormal areas of radionuclide uptake is used to identify the
extent of the disease.
More Detail
Elevated total alkaline phosphatase in this patient is compatible with active disease, but it is uncertain whether the left
posterior ninth rib lesion is the sole location of the disease. Because incidentally discovered or symptomatic sites of involvement
cannot be assumed to represent the extent of disease, a bone scan followed by focused radiography of abnormal areas of
radionuclide uptake is used to identify and assess the presence of lesions other than the sentinel lesion.
References
Objective number 74[Periodic Health Examination (PHE)] of Objectives for the Qualifying Examination, 3rd edition. Tuck SP,
Walker J. Adult Paget's disease of bone. Clin Med (Lond). 2020 Nov;20(6):568-571. Charles JF. Treatment of Paget disease of
bone. Post TW, ed. UpToDate. Waltham, MA: UpToDate Inc. Accessed June 30, 2022.
Question #39
A 55-year-old woman is admitted to the hospital following a calcium level of 1.8 mmol/L (2.18‐2.58). Routine laboratory tests
the next morning show that her PTH is low, phosphorous is elevated, and magnesium is normal. Electrocardiography is normal.
The patient describes a sensation or numbness around her mouth and tingling in her fingers. Which of the following is the most
appropriate medication to administer to the patient at this time?
A) Vitamin D
B) Furosemide
C) Alendronate
D) Calcium gluconate
Explanation
Answer Description
Vitamin D, or calcitriol, therapy should be started on all patients with hypoparathyroidism. Vitamin D will not help to acutely raise
calcium levels, but maintenance therapy is recommended after levels have returned to a normal range.
Furosemide is a loop diuretic that increases calcium excretion, also resulting in hypocalcemia. Patients may also be found to
have hypomagnesemia and hypokalemia. Thiazide diuretics, in contrast, cause hypercalcemia but are not routinely prescribed
for the treatment of hypoparathyroidism.
Alendronate is bisphosphate commonly utilized to treat osteoporosis and may be incorporated into the treatment of
hyperparathyroidism. A common side effect is hypocalcemia, so it may acutely worsen this patient's symptoms.
Summarized Points:
Patients with calcium levels of < 1.9 mmol/L or symptoms such as seizures, tetany, or prolonged QT intervals on an ECG should
get emergency therapy. Standard therapy for patients is 1 to 2 g calcium gluconate intravenously administered slowly over 10
minutes.
More Detail
Serum calcium and vitamin D are controlled by parathyroid hormone via effects on the kidneys, gastrointestinal tract, and
bone. Parathyroid hormone is secreted by four parathyroid glands that are located in the anterior neck just posterior to the
thyroid gland. Hypoparathyroidism can develop after the destruction of these glands due to surgical or autoimmune
means. Other etiologies include abnormalities in parathyroid hormone production, altered parathyroid gland development, or
impaired action of the hormone. If a diagnosis of hypoparathyroidism is suspected, additional testing should include parathyroid
hormone, vitamin D, phosphorus, magnesium, and albumin levels.
The most common cause of acquired hypoparathyroidism is postsurgical. Some patients may go several years without any
symptoms, so calcium levels should be monitored regularly. Hypocalcemia occurs when there is insufficient production of
parathyroid hormone. Symptoms can range from asymptomatic to dyspnea, palpitations, numbness, and muscle spasms.
More severe cases can present with seizures, tetany, heart failure, or laryngospasm.
Treatment depends on the status of the patient and whether or not they display life-threatening issues secondary to
hypocalcemia. The primary goal is to alleviate symptoms and maintain serum calcium concentrations in a normal range, around
2 to 2.1 mmol/L. In patients with calcium levels below 1.9 mmol/L or those with symptoms such as seizures, tetany, or
prolonged QT intervals on electrocardiogram, emergency therapy is indicated. Patients are commonly treated with calcium
gluconate 1 to 2 g intravenously infused slowly over 10 minutes. Electrocardiogram monitoring should be done while
administering the medication. Patients should start oral calcitriol and calcium carbonate as soon as they can tolerate it and after
levels have returned to a normal range. Intravenous therapy can be weaned while transitioning to oral therapy.
References
Objective number 12-2 (Calcium Disorders) of Objectives for the Qualifying Examination, 3rd edition. Goltzman D.
Hypoparathyroidism. Post TW, ed. UpToDate. UpToDate Inc. Accessed June 10, 2021.
Question #35
A 17-year-old girl comes to the physician because of a 2-day history of stomach pain, shortness of breath with minimal exertion,
nausea, weakness, and fatigue. She has had urinary frequency and excessive thirst for the past 2 weeks. Her father and
maternal uncle have type 2 diabetes mellitus. She admits to being sexually active with one partner. The patient is 173 cm (5 ft 8
in) tall and weighs 66 kg (145 lb); the BMI is 22 kg/m2. Her pulse is 92/min, and blood pressure is 100/60 mm Hg while sitting;
her pulse is 124/min, and palpable systolic blood pressure is 80 mm Hg while standing. Her respirations are 36/min. Her
abdominal examination is significant for diffuse pain on palpation in all quadrants but no rebound or guarding. The rest of her
examination produces negative results. Laboratory values are:
128 mmol/L
Na+
(135‐145)
A) Insulin
B) Intravenous fluids and Insulin
C) Pregnancy test
D) Sodium bicarbonate
E) Upright x-ray of chest
Explanation
Answer Description
Insulin administration would be a step in management; however, patients in DKA are usually volume depleted. Intravenous fluids
would be administered with insulin to help correct the hypovolemia and hyperosmolarity, which are the main causes of the
patients symptoms.
Summarized Points:
The treatment for DKA consists of aggressive administration of intravenous fluids and insulin.
More Detail
Diabetic ketoacidosis (DKA) has a 100% mortality rate if left untreated. The treatment for diabetic ketoacidosis consists of
aggressive intravenous fluids and insulin. Patients often present with significant dehydration that must be reversed as quickly
as possible. This is most commonly due to undiagnosed diabetes mellitus, or in patients who have been diagnosed with
diabetes, failure to take prescribed insulin.
References
Braunwald E, Fauci AS, Kasper DL, et al. Harrison’s principles of internal medicine, vol 2. New York: McGraw Hill, 2001.
Chapter: DM; p2109-2135. Objective number 37 (Glucose Abnormal, Serum/Diabetes Mellitus/Polydipsia) of Objectives for the
Qualifying Examination, 3rd edition.
Question #37
A 6-year-old girl is brought to the physician for a routine examination. She has no complaints and is enjoying first grade. Her
favourite drink is orange juice, and she enjoys chips and cookies. The girl has no medical problems and takes no medications.
The review of the systems is negative. Several maternal relatives are obese, but no family members have heart disease or
stroke. There are no smokers in the home. Body mass index (BMI) was previously in the 80th percentile but is now in the 99th
percentile. Vital signs are normal. Examination reveals central adiposity. No acanthosis nigricans is seen. Which of the following
is the best next step in the management of this patient?
Answer Description
Routine electrocardiogram in asymptomatic children with no significant family history of cardiac disease is low yield and not
recommended.
Hypothyroidism rarely causes obesity in isolation and usually manifests with other abnormalities (e.g., constipation, cold
intolerance, fatigue).
Summarized Points:
Pediatric obesity is increasingly common and is associated with early complications such as dyslipidemia, fatty liver disease, and
hypertension. Fasting glucose, serum alanine aminotransferase, and a lipid panel should be measured in these patients. Dietary
and lifestyle modifications are the first-line treatment.
More Detail
The growing prevalence of overweight (body mass index [BMI] 85th–95th percentile) and obese (BMI ≥95th percentile) children
has resulted in increasing rates of early dyslipidemia and metabolic syndrome. Children with a BMI ≥95th percentile should
undergo fasting glucose testing to screen for diabetes mellitus, serum alanine aminotransferase (ALT) to screen for fatty
liver disease, and a serum lipid panel to screen for dyslipidemia. Ideally, total cholesterol, triglycerides, low-density lipoprotein
(LDL) and high-density lipoprotein (HDL) cholesterol are measured after the patient has fasted. Triglycerides in particular can
remain elevated for several hours after a high-fat meal, but recent studies show that the other values may not be significantly
affected by a recent meal. The first-line treatment for children with elevated LDL and triglycerides includes regular exercise, a
healthy diet low in fat and cholesterol, avoiding sugary beverages, and minimizing television time to <2 hours per day.
Addressing modifiable risk factors can delay, pause, or reverse the development of these complications.
References
Objective number 118-1 (Weight Gain, Obesity) of Objectives for the Qualifying Examination, 3rd edition. Styne DM, Arslanian
SA, Connor EL, et al. Pediatric Obesity-Assessment, Treatment, and Prevention: An Endocrine Society Clinical Practice
Guideline. J Clin Endocrinol Metab 2017; 102:709.
Question #25
A 32-year-old woman returns to the office in 6 months for a follow-up regarding her weight loss program. The patient was
instructed to follow a program that includes a calorie-restricted, low-carbohydrate diet and appropriate exercise regimen. She
reports that she has not lost any weight despite strict adherence to the program. Her body mass index is 30 kg/m2, blood
pressure is 128/72 mm Hg, and pulse is 72/min. Which of the following is the best next step in the management of this patient?
A) Bariatric surgery
B) Change to low-fat diet
C) Liposuction
D) Phentermine
E) Orlistat
Explanation
Answer Description
Bariatric surgery is indicated for patients who fail to lose weight with conservative management and have a BMI >40 kg/m2 (or
>35 kg/m2 and additional obesity-related comorbidities).
Studies comparing low-fat versus low-carbohydrate diets have been mixed, and one approach is not clearly superior. The patient
may choose to try a different dietary regimen, but this would be unlikely to yield significant weight loss.
Liposuction is used primarily for aesthetic body contouring. Large-volume liposuction can provide significant weight loss, but it
carries a risk of complications and does not reliably improve obesity-related cardiovascular risk.
Phentermine is a sympathomimetic agent that can cause fast weight reduction in the short term. However, phentermine
monotherapy is associated with considerable re-gain and little long-term effect. Additionally, Phentermine is not approved in
Canada.
Summarized Points:
Medication for weight loss is indicated for patients who fail initial dietary modification and are obese (BMI >30 kg/m2) or those
with a BMI of 27-29.99 kg/m2 and comorbidities. Orlistat has an excellent safety profile but significant gastrointestinal side
effects.
More Detail
Obesity is a common disease that is associated with multiple medical problems including diabetes mellitus, cardiovascular
disease, and pulmonary disease. Obesity is defined as a BMI > 30 kg/m2. Initial management of obese individuals includes
modifications of diet and exercise. For individuals who have not lost at least 5% of their total body weight after 3 to 6
months of diet and exercise modifications, drug therapy can be considered for those with a BMI > 30 kg/m2 or those with a BMI
of 27-29.99 kg/m2 and comorbidities.
There are now three prescription drugs available in Canada that are approved for use in adult patients with BMIs of ≥30 kg/m2
or ≥27 kg/m2 and ≥1 weight-related disease (e.g., Hypertension, diabetes mellitus type 2, dyslipidemia). It should be noted that
these drugs should be taken in conjunction with a reduced-calorie diet and increased physical exercise.
The choice of medication may be dependent on the patient's comorbidities and relative contraindications. Orlistat inhibits
pancreatic lipases, resulting in decreased fat absorption. It is frequently chosen as a first-line drug due to its excellent safety
profile. However, orlistat is associated with significant gastrointestinal side effects (e.g., diarrhea, bloating, fecal
incontinence), and some patients may elect to start with another medication.
References
Objective number 118-1 (Weight Gain, Obesity) of Objectives for the Qualifying Examination, 3rd edition. Wharton S, Law DCW,
Vallis M, et al. Obesity in adults: a clinical practice guideline. CMAJ 2020;192:E875-E891.
Question #27
A 48-year-old woman presents to the clinic with dysphagia, diarrhea, and facial flushing. Physical examination reveals a solitary
thyroid nodule and cervical lymphadenopathy. Laboratory studies show elevated serum calcitonin and elevated
carcinoembryonic antigen levels. Which of the following is used to confirm the diagnosis of this patient?
Answer Description
Choice A and C are not correct:
Although most cases of medullary thyroid cancer are acquired sporadically, genetic inheritance can be seen in cases of multiple
endocrine neoplasia type 2 (MEN2). MEN2 is associated with medullary thyroid cancer, pheochromocytoma. and primary
parathyroid hyperplasia. Therefore, additional testing for coexisting tumors such as biochemical evaluation for
pheochromocytoma and germline RET mutation testing is recommended.
Once medullary thyroid cancer is confirmed with fine-needle aspiration biopsy, further evaluation should include ultrasonography
of the neck to look for lymph node involvement.
Summarized Points:
Thyroid Carcinoma physical exam will show a solitary hard nodule. Labs will show normal thyroid function, cold nodule. Initial
Diagnosis is ultrasound, and Confirmation is Fine-needle biopsy.
More Detail
The production of calcitonin and carcinoembryonic antigen (CEA) is characteristic of medullary thyroid cancer, and these
are useful markers for diagnosis and follow-up. Patients typically present with dysphagia, diarrhea, facial flushing, and a
solitary thyroid nodule. Medullary thyroid cancer is a neuroendocrine tumor of the parafollicular or C cells of the thyroid
gland. The diagnosis of medullary thyroid cancer is made after a fine-needle aspiration biopsy of the thyroid nodule. Treatment
is with the removal of the affected thyroid lobes, and for nodules that are 4 cm or larger, diagnostic lobectomy is recommended
over fine-needle aspiration.
References
Objective number 63 (Neck Mass, Goiter, Thyroid Disease) of Objectives for the Qualifying Examination, 3rd edition. Kant R,
Davis A, Verma V. Thyroid Nodules: Advances in Evaluation and Management. Am Fam Physician. 2020;102(5):298-304.
Question #28
Further workup of this patient confirms the diagnosis of medullary thyroid cancer. Serum calcitonin is 300 pg/mL (0-10). Genetic
studies show a mutation in the RET proto-oncogene. Which of the following is the best next step in the management of this
patient?
If pheochromocytoma has been ruled out, patients with nonmetastatic MTC may proceed to thyroidectomy for definitive
treatment.
Radioactive iodine is preferentially taken up by thyroid tissue and is often used following surgery for patients with differentiated
thyroid cancer (papillary or follicular) who are at risk for recurrence. Radioactive iodine is not effective for MTC as these cells do
not take up iodine.
Anticancer drugs and external beam radiation do not play a role in the initial management of MTC. Radiotherapy is usually
reserved for postsurgery patients with extrathyroidal disease or extensive nodal metastases not amenable to surgical resection.
Chemotherapy is considered for patients with progressively metastatic MTC not responsive to surgery or radiotherapy.
Summarized Points:
Medullary thyroid cancer (MTC) is part of the multiple endocrine neoplasia type 2 syndromes. All patients with new MTC require
serum calcitonin and carcinoembryonic antigen, neck ultrasound (evaluation for regional metastases), genetic testing for
germline RET mutations, and evaluation for coexisting tumors (hyperparathyroidism, pheochromocytomas).
More Detail
Multiple endocrine neoplasia type 2 (MEN 2) is an autosomal dominant disorder caused by germline mutations involving
the RET proto-oncogene located on chromosome 10. MEN 2 can be divided into 3 classifications: MEN 2A, MEN 2B, and
familial medullary thyroid cancer (FMTC).
• MEN 2A predisposes to medullary thyroid cancer (MTC), pheochromocytoma (PCC), and hyperparathyroidism
(due to parathyroid hyperplasia or multiple adenomas).
• MEN 2B is not accompanied by hyperparathyroidism but may be associated with marfanoid habitus, mucosal
neuromas, and skeletal deformations (e.g., kyphoscoliosis, lordosis).
• FMTC is a subset of MEN 2A associated with MTC only, without PCC or parathyroid disease.
All patients with MTC require serum calcitonin and carcinoembryonic antigen, neck ultrasound (evaluation for regional
metastases), genetic testing for germline RET mutations, and evaluation for coexisting tumors (hyperparathyroidism, PCCs).
PCC is present in approximately 40% of patients with MEN 2, and undiagnosed PCC can cause life-threatening
hemodynamic complications during and after thyroid surgery. Plasma-free metanephrines are usually elevated in PCC and
confirmed with 24-hour urinary fractionated metanephrines, catecholamines, and abdominal imaging. Once PCC has been ruled
out, patients with nonmetastatic MTC may proceed to thyroidectomy for definitive management.
References
Objective number 63 (Neck Mass, Goiter, Thyroid Disease) of Objectives for the Qualifying Examination, 3rd edition. Wells SA Jr,
Asa SL, Dralle H, et al. Revised American Thyroid Association guidelines for the management of medullary thyroid carcinoma.
Thyroid 2015; 25:567.
Question #31
A 38-year-old woman has had severe right-sided flank pain for the past 2 days. She has a history of hypothyroidism, which is
treated with levothyroxine, for which you have been monitoring her for the past year. Her temperature today is 39.1° C (102.4 F),
blood pressure 100/50 mm Hg lying down and 80/40 mm Hg sitting up, pulse 120/min lying down, and 145/min sitting up. On
physical examination, she appears very ill and fatigued. Her skin appears tan. There is severe right-sided flank tenderness. As
her lab results are pending, she is hydrated in the office with Ringer's lactate solution. After 2 L of intravenous fluids, her blood
pressure is 90/45 mm Hg lying down and she still feels too weak to stand up. Laboratory studies show:
An endocrinology consult would be too time-consuming and would delay lifesaving treatment.
Checking serum cortisol levels would be helpful in solidifying the diagnosis of adrenal insufficiency, but administering IV steroids
is the higher priority.
A blood transfusion is not required for this patient's anemia; it is most likely the steroid deficiency, not the chronic anemia, is
contributing to her hypotension.
Intravenous antibiotics will be required for the pyelonephritis, but the first step in management includes stress-dose steroids.
Summarized Points:
Primary adrenal insufficiency is most commonly caused by autoimmune adrenalitis and should be suspected when there is
orthostatic hypotension, hyperpigmented skin, hypoglycemia, hyponatremia, hyperkalemia, metabolic acidosis, and
peripheral eosinophilia. Physiologic stressors such as infection can precipitate an adrenal crisis with profound hypotension.
Treat with Large volumes (1 to 3 liters) of 0.9 percent saline solution or 5 percent dextrose in 0.9 percent saline (if there is
hypoglycemia)
• In a patient without a previous diagnosis of adrenal insufficiency, dexamethasone, which is not measured in cortisol
assays, should be used rather than hydrocortisone while biochemical testing is performed.
• For patients with a previously known diagnosis of adrenal insufficiency, IV hydrocortisone or any other glucocorticoid
preparation may be used because diagnostic testing is not necessary.
More Detail
This patient likely has primary adrenal insufficiency (Addison's disease) resulting from autoimmune adrenalitis. Her urinalysis is
consistent with a urinary tract infection and, given her flank tenderness and elevated white blood cells (WBCs), she
probably has pyelonephritis. The infection has precipitated an adrenal crisis, which occurs because of the loss of both
glucocorticoid and mineralocorticoid activity, leading to hypotension and hypovolemia. Clues suggestive of primary adrenal
insufficiency are the presence of concomitant autoimmune disease (hypothyroidism), hyperpigmented skin,
hyponatremia, hyperkalemia, metabolic acidosis, hypoglycemia, and a peripheral eosinophilia.
Treatment of patients who present in possible adrenal crisis should not be delayed while diagnostic tests are performed. Blood
for serum cortisol, adrenocorticotropic hormone (ACTH), aldosterone, renin, and serum chemistry should be drawn and therapy
initiated immediately. The serum chemistry results guide initial treatment; the hormone values return later and are used to
confirm the diagnosis (cortisol and aldosterone) or evaluate the differential diagnosis (ACTH and renin).
Treatment includes 1 to 3 liters of 0.9 percent saline solution or 5 percent dextrose in 0.9 percent saline (to correct possible
hypoglycemia) should be infused intravenously (IV) within the first 12 to 24 hours based on the assessment of volume status and
urine output. Hypotonic saline should not be used because it can worsen hyponatremia.
References
Objective number 37-1 (Glucose Abnormalities) of Objectives for the Qualifying Examination, 3rd edition. Bornstein SR, Allolio B,
Arlt W, et al. Diagnosis and Treatment of Primary Adrenal Insufficiency: An Endocrine Society Clinical Practice Guideline. J Clin
Endocrinol Metab. 2016 Feb;101(2):364-89. Nieman LK. Diagnosis of adrenal insufficiency in adults. Post TW, ed. UpToDate.
Waltham, MA: UpToDate Inc. Accessed October 16, 2019.
Question #2
A 35-year-old man is seen for a follow-up visit six months after undergoing total thyroidectomy and central neck dissection for
medullary thyroid cancer. Pathology results revealed that the tumor was 3 cm in diameter, and 3 of 18 dissected nodes were
positive for malignancy. The fasting serum calcitonin level before surgery was 280 ng/ml (< 4ng/ml). The postoperative course
was uncomplicated, and he was then started on levothyroxine 0.15 mg/d. The repeat TSH level obtained after 8 weeks was
normal. The physical examination revealed no abnormal neck masses. The rest of the examination is also unremarkable. His
current calcitonin level is 170 ng/ml. What is the next best step in the management of this patient?
Ultrasonography limited to the thyroid bed will not pick up lesion(s) in other parts of the neck.
Medullary thyroid cancer is a malignancy of parafollicular C-cells that do not concentrate iodine; therefore, total body iodine scan
is not useful in the detection of metastasis of medullary thyroid cancer.
Thallium scintiscan is nonspecific and has not been studied properly in patients with medullary thyroid cancer.
Chest x-ray has a very low sensitivity for detecting metastatic lesions, and should not be used as a solitary investigative tool.
Summarized Points:
Elevated calcitonin levels in patients with medullary thyroid cancer following total thyroidectomy indicate metastatic disease. CT
scan of the neck and chest is recommended as the next step to look for metastatic disease. Medullary thyroid cancer cells do not
take up iodine; therefore, total body iodine scan is not useful.
More Detail
The patient has residual metastatic thyroid cancer, as suggested by the persistently elevated calcitonin levels. CT scan of
the neck and chest with or without high-resolution ultrasound of the neck is recommended as the next step to look for
metastatic disease. Surgical resection should be performed if a resectable lesion(s) is identified. If no lesion is identified, then
an abdominal CT scan and bone scan may be required. I111 octreotide or PET scan may be helpful if CT scans are negative
despite the presence of elevated calcitonin levels.
References
Objective number 63 (Neck Mass, Goiter, Thyroid Disease) of Objectives for the Qualifying Examination, 3rd edition. Wells SA Jr,
Asa SL, Dralle H, et al. Revised American Thyroid Association guidelines for the management of medullary thyroid carcinoma.
Thyroid 2015; 25:567.
Question #33
A 40-year-old obese man with type 2 diabetes mellitus comes to the clinic with abdominal distension, flatus, and low-volume
fecal incontinence over the past 2 weeks. He has been treated with metformin for years and was recently started on exenatide
and orlistat. His abdomen is distended, tympanic, and nontender. Which of the following is the best next step in the management
of this patient?
A) Discontinue exenatide
B) Lactose-free diet
C) Low-fat diet
D) Metoclopramide
E) Simethicone
Explanation
Answer Description
Exenatide is a glucagon-like peptide-1 analog that can improve glycemic control and induce weight loss in overweight diabetics.
Exenatide can cause acute pancreatitis, but the absence of nausea, vomiting, and significant abdominal pain makes pancreatitis
less likely.
Lactose intolerance presents with bloating, flatulence, crampy abdominal pain, and watery diarrhea after consumption of dairy
products. However, orlistat is not associated with lactose intolerance.
Metoclopramide is a promotility agent used in delayed gastric emptying. Although such disorders can cause abdominal
distension, they are also associated with nausea and vomiting, abdominal pain, and early satiety.
Although simethicone can help to reduce flatus, it would not address this patient's fecal incontinence. A low-fat diet would
eliminate the source of his symptoms.
Summarized Points:
Orlistat inhibits pancreatic lipase to reduce fat absorption and increase fecal fat wasting. Patients with high dietary fat intake can
develop side effects similar to fat malabsorption (e.g., flatus, fecal incontinence, oily spotting).
More Detail
This patient is experiencing adverse effects from orlistat. Orlistat is used for the treatment of obesity and works by inhibiting
pancreatic lipase to alter fat digestion, decrease fat absorption, and increase fecal fat wasting. As a result, patients with high
dietary fat intake can develop symptoms similar to fat malabsorption (e.g., flatus, fecal incontinence, abdominal distension. A
low-fat diet (30% of total calories) can reduce these symptoms. Orlistat can also decrease absorption of fat-soluble vitamins so
patients should take a daily multivitamin.
References
Objective number 118-1 (Weight Gain, Obesity) of Objectives for the Qualifying Examination, 3rd edition. Jensen MD, Ryan DH,
Apovian CM, et al. 2013 AHA/ACC/TOS guideline for the management of overweight and obesity in adults: a report of the
American College of Cardiology/American Heart Association Task Force on Practice Guidelines and The Obesity Society.
Circulation 2014; 129:S102.
Question #38
A 74-year-old woman is brought to the office for evaluation of a left heel wound. She had an ischemic stroke a year ago with
residual left hemiplegia and has lived in a nursing home facility since. The patient also has oropharyngeal dysphagia and is fed
through a gastrostomy tube. For the past several weeks, she has had a gradually enlarging left heel ulcer She has no pain
sensation in the left leg and has had no fever or foul-smelling drainage. Her other medical problems include hypertension, type 2
diabetes mellitus, and coronary artery disease. Left heel examination shows a 2-cm ulcer with thick, firm eschar. There is no
crepitus or surrounding redness. The lower-extremity pulses are weak. Which of the following is the best next step in the
management of this patient?
Answer Description
There is insufficient evidence that hyperbaric oxygen therapy provides benefit in treating pressure ulcers.
Surgical debridement is usually recommended for pressure ulcers with clinically evident infection, extensive tissue necrosis, or
failure to improve with conservative therapy. However, it is otherwise not recommended for heel pressure ulcers as the close
proximity of the bone can increase the risk of osteomyelitis.
Wound swab culture usually reflects skin colonization and is not useful for determining the true microbiology of an infection.
Needle aspiration or deep-tissue biopsy is more effective if needed.
Summarized Points:
Heel pressure ulcers are usually managed conservatively as removing tissue around the heel can increase the risk of
osteomyelitis. Proper positioning to redistribute the pressure is the most important intervention to improve healing.
More Detail
This patient has an unstageable heel pressure ulcer, defined as full-thickness skin loss covered by eschar that obscures the
ulcer base. Unstageable ulcers are usually stage III or IV, although the true depth cannot be determined unless eschar is
removed to expose the base of the wound. Heel pressure ulcers are managed differently from those at other sites due to the
limited thickness of subcutaneous tissue and proximity of the underlying bone. Removing tissue around the heel can
increase the risk of osteomyelitis. Stable (dry, adherent, intact without erythema or fluctuance) eschar on the heels serves as
a natural protective cover and should not be removed. Instead, patients are usually treated initially with conservative measures,
including pressure redistribution, pain control, and optimized nutrition. Proper positioning to redistribute the pressure is the
most important intervention to increase tissue perfusion, improve healing, and prevent future ulcers. Some patients may need
support surfaces or devices (e.g., orthotic device, foam boot) for adequate pressure relief. Iodine paint to keep the area dry or
enzymatic debriding agents may also be used. Measures to control pain include oral medications and topical anesthetics (e.g.,
lidocaine). Because pressure ulcers cause a chronic catabolic state, adequate caloric intake is also required for proper wound
healing.
References
Objective number 95(Skin Tumors and Ulcers) of Objectives for the Qualifying Examination, 3rd edition. Kottner J, Cuddigan J,
Carville K, et al. Prevention and treatment of pressure ulcers/injuries: The protocol for the second update of the international
Clinical Practice Guideline 2019. J Tissue Viability 2019; 28:51.
Question #5
A 58-year-old woman is evaluated for enlargement of her hands and feet. Medical history is significant for hypertension. Her only
medication is amlodipine. On physical examination, vital signs are normal. BMI is 24. The patient has a wide nose and
enlargement of hands and feet. Prognathism is noted. Which of the following is the most appropriate diagnostic test?
If a patient's IGF-1 level is elevated, then an oral glucose tolerance test can confirm the diagnosis of acromegaly. This test is
performed by administering 75 g of oral glucose and measuring GH levels every 30 minutes for 120 minutes. GH less than 0.2
µg/L is a normal response, whereas a GH nadir of 1.0 µg /L or greater is diagnostic of acromegaly.
Because the diagnosis of an endocrine disorder is always made on the basis of laboratory evaluation before imaging, a pituitary
MRI should not be obtained in this patient before measuring her IGF-1. Small incidentally noted pituitary lesions are common. In
patients undergoing MRI for nonpituitary reasons, microadenomas are found in 10% to 38% of cases whereas incidental
macroadenomas are seen in 0.2% of cases. Most pituitary incidentalomas (a pituitary lesion discovered incidentally on imaging)
are benign non-functional adenomas. Because many patients may have a pituitary incidentaloma, obtaining imaging before a
biochemical diagnosis of pituitary excess disorders may be misleading. When laboratory evaluation shows GH excess, a pituitary
MRI should be obtained.
Because GH is pulsatile, measuring a random GH level may result in a false-negative result and possibly a missed diagnosis of
acromegaly.
Summarized Points:
An insulin-like growth factor-1 level is the best screening biomarker for the diagnosis of acromegaly. In patients with an elevated
insulin-like growth factor-1 level, the diagnosis of acromegaly can be confirmed with an oral glucose tolerance test.
More Detail
Because IGF-1 is produced in the liver in response to GH stimulation, obtaining an IGF-1 level is the most appropriate method to
evaluate for acromegaly and it is the best screening biomarker for acromegaly because it is more stable than GH level. IGF-1
levels are normalized for age and sex, and aging lowers the normal range of IGF-1.
References
Objective number 9-1(Hypertension) of Objectives for the Qualifying Examination, 3rd edition. Giustina A, Barkan A, Beckers A
et al. A Consensus on the Diagnosis and Treatment of Acromegaly Comorbidities: An Update. J Clin Endocrinol Metab. 2020 Apr
1;105(4):dgz096. Melmed S, Katznelson L. Diagnosis of acromegaly. Post TW, ed. UpToDate. Waltham, MA: UpToDate Inc.
Accessed December 03, 2021.
Question #6
A 43-year-old woman was applying her make-up while also drinking her morning cup of coffee. She noticed in the mirror that a
round, 2-cm mass would move up and down in the lower part of her neck whenever she swallowed. Her physician confirms that
she has a single, mobile, thyroid nodule in the right lobe. There are no other abnormalities in the history or physical examination.
Her pulse is 82/min and regular. Thyroid stimulating hormone (TSH) is within normal limits. Which of the following is the most
appropriate next step in management?
Answer Description
Clinical observation alone would not be appropriate. Thyroid cancers grow slowly, but they still need to be diagnosed and
treated. Incidentally, they typically do not affect thyroid function, so that following the TSH would not alert you to the presence of
malignancy.
Thyroid nodules can be benign but hyperfunctioning (toxic adenoma), and therefore thyroid function must be determined. But,
that has already been done here with the normal TSH. Further pursuit of T3 and T4 is unnecessary in someone with no clinical
evidence of hyperfunction (normal pulse).
Thyroid scan would have been the answer 10 or 20 years ago, before FNA displaced it as the best way to select surgical
candidates. In the old days, a cold nodule raised suspicions of malignancy but gave low yields at surgery.
Without a diagnosis of cancer, or an indeterminate FNA, one cannot justify the extremely aggressive approach of surgery as the
next step in management. Thus, choice E is clearly wrong.
Summarized Points:
Once TSH is normal, next step is Fine needle aspiration (FNA). A needle is inserted into the nodule, and cells are aspirated and
then examined under a microscope. This is the only test that can reliably differentiate between benign and malignant nodules.
More Detail
Most thyroid nodules are benign, and surgery must be reserved for selected candidates with the highest likelihood of malignancy.
Fine needle aspiration (FNA) is the best way to make the selection. If read by an experienced pathologist as negative for
cancer, patients can be safely followed. If read as either indeterminate or positive for cancer, surgery would be required.
References
Kundel A, Zarnegar R, Kato M, et al. Comparison of microarray analysis of fine needle aspirates and tissue specimen in thyroid
nodule diagnosis. Diagn Mol Pathol. 2010 Mar. 19(1):9-14. Objective number 63(Neck Mass, Goiter, Thyroid Disease) of
Objectives for the Qualifying Examination, 3rd edition.
Question #12
An 84-year-old woman comes to the office due to fatigue. For the last 6 months since her husband died, she has had low energy
associated with decreased appetite. The patient has had no weight change, fevers, chills, or night sweats. She sleeps 5-6 hours
a night. Medical history is notable for hypertension, depression, and osteoporosis, for which she takes hydrochlorothiazide,
sertraline, and alendronate, respectively. Temperature is 36.8 C (98.2 F), blood pressure is 128/82 mm Hg, and pulse is 68/min.
BMI is 24 kg/m2. On examination, the patient is awake, alert, and cooperative. Neck examination is normal with no goiter.
Cardiopulmonary examination is normal. Neurologic examination shows normal cranial nerves and motor strength. Ankle
reflexes are absent. Laboratory studies show normal glucose, electrolytes, and blood counts.
Which of the following is the most appropriate next step in the management of this patient's symptoms?
Answer Description
T3 is produced primarily in peripheral tissues; serum levels correlate poorly with thyroid status. Free T3 levels have some utility
in evaluating hyperthyroidism but are not recommended in hypothyroidism.
Thyroid ultrasound is primarily used for the evaluation of patients with a palpable goiter or thyroid nodules. It is not indicated in
the routine evaluation of hypothyroid conditions.
Choice D is not correct:
Thyrotropin (TSH) receptor antibodies are involved in the pathogenesis of Graves disease, the most common cause of
hyperthyroidism. Serum titers have little utility in the evaluation of hypothyroidism.
TSH values as high as 8 µU/mL may indicate physiologic euthyroidism in adults over the age of 80. Older patients should be
followed with periodic retesting (e.g., every 1-3 months and every 6-12 months thereafter) in substitution of treatment, and
medication should be considered for those who develop to overt hypothyroidism.
Summarized Points:
Subclinical hypothyroidism is defined as a serum TSH above the upper limit of normal with a serum free T4 level in the normal
range. Patients age ≥70 with subclinical hypothyroidism should generally not be treated but should have periodic monitoring for
progression to overt hypothyroidism.
More Detail
This patient has subclinical hypothyroidism, defined as a serum TSH above the upper limit of normal with serum free T4
in the normal range. Subclinical hypothyroidism is often associated with symptoms that suggest a hypothyroid state, but in most
cases, symptoms are vague and highly nonspecific (e.g., fatigue, constipation). Treatment of subclinical hypothyroidism is
recommended for all patients with TSH ≥10 or those age <70 with convincing hypothyroid symptoms or features associated
with progression to overt hypothyroidism (e.g., enlarging goiter, elevated anti-thyroid peroxidase antibody titer).
In contrast, older (age 70) patients with subclinical hypothyroidism warrant more conservative management. Levothyroxine
replacement in this population confers less benefit than in younger patients and carries an increased risk for adverse events
(e.g., atrial fibrillation). In addition, some experts believe that the true normal TSH range in older patients may be higher than in
younger patients, and levels as high as 8 µU/mL may represent physiologic euthyroidism in patients age >80. In lieu of
treatment, older patients should be monitored with periodic retesting (e.g., in 1-3 months and every 6-12 months thereafter),
and treatment should be considered for those who progress to overt hypothyroidism.
References
Objective number 33(Fatigue) of Objectives for the Qualifying Examination, 3rd edition. Stott DJ, Rodondi N, Bauer DC; TRUST
Study Group. Thyroid Hormone Therapy for Older Adults with Subclinical Hypothyroidism. N Engl J Med. 2017 Oct
5;377(14):e20.
Question #28
A 52-year-old woman with mild hypertension treated with losartan is found to have a serum calcium level of 2.7 mmol/L. She is
asymptomatic. Repeat testing shows:
TSH and 25-hydroxyvitamin D levels are normal. Which of the following is the most appropriate next step in evaluating this
patient?
Answer Description
Secretion of parathyroid hormone-related peptide (PTHrP) by solid tumors (e.g., lung and breast cancer) can cause
hypercalcemia, which is PTH-independent with PTH levels <20 pg/mL. This patient has an elevated PTH level, so PTHrP
measurement is not necessary.
Nephrolithiasis, nephrocalcinosis, and chronic renal insufficiency can occur in patients with PHPT. However, routine imaging to
look for renal stones or nephrocalcinosis is not indicated in asymptomatic patients.
Radionuclide parathyroid scan and ultrasound of the neck are used to localize an adenoma in patients with PHPT in preparation
for parathyroidectomy. This patient should first undergo a bone densitometry scan to determine if surgery is indicated.
Summarized Points:
Primary hyperparathyroidism is characterized by elevated calcium with elevated parathyroid hormone. Patients should be
evaluated for indications for parathyroidectomy including:
• Age <50
• Low bone density with a T-score <-2.5 (at hip, distal radius, or lumbar spine)
• Renal insufficiency
• Calcium >1 mg/dL (0.25 mmol/L) above the upper limit of normal
More Detail
References
Objective number 12 (Calcium/Phosphate Concentration Abnormal, Serum) of Objectives for the Qualifying Examination, 3rd
edition. Bilezikian JP, Brandi ML, Eastell R, et al. Guidelines for the management of asymptomatic primary hyperparathyroidism:
summary statement from the Fourth International Workshop. J Clin Endocrinol Metab 2014; 99:3561.
Question #13
A 33-year-old woman is evaluated for fatigue and abdominal pain of 4 months duration. She lost 2.3 kg (5.1 lb) during that time.
She has no other medical problems and takes no medications. On physical examination, blood pressure is 110/72 mm Hg sitting
and 90/62 mm Hg standing. Pulse rate is 70/min sitting and 88/min standing. Remaining vital signs are normal. BMI is 19 Kg/m2.
The patient has intensely tanned skin and hyperpigmented buccal mucosa and palmar creases. Visual field testing is normal.
Laboratory studies:
A) Prednisone
B) Pituitary MRI
C) Cosyntropin stimulation test
D) Serum aldosterone measurement
E) Hydrocortisone plus fludrocortisone
Explanation
Answer Description
Adherence to multiple daily doses of hydrocortisone can be challenging. Once-daily prednisone can be used as an alternative,
but it also must be combined with fludrocortisone. Prednisone alone would not provide adequate mineralocorticoid replacement.
Given the high adrenocorticotropic hormone level, which indicates that pituitary function is intact, the etiology is confirmed to be
primary and not secondary (central) adrenal insufficiency. Therefore, a pituitary MRI is not indicated.
Although the cosyntropin stimulation test is often used to diagnose adrenal insufficiency, in this patient. the diagnosis has already
been confirmed by a morning serum cortisol level. As a result, a cosyntropin stimulation test is not required for diagnosis and will
not change management.
Serum aldosterone measurement is not required to diagnose primary adrenal insufficiency, especially because this patient
already has hyperkalemia and hyponatremia, which suggest aldosterone deficiency.
Summarized Points:
The preferred treatment of primary adrenal insufficiency is hydrocortisone two or three times daily plus fludrocortisone. The most
common cause of primary adrenal insufficiency is autoimmune destruction of all layers of the adrenal cortex leading to
progressive mineralocorticoid, glucocorticoid, and adrenal androgen deficiency.
More Detail
The correct answer is E.
This patient has primary adrenal insufficiency, as evidenced by the combination of her low morning serum cortisol and
elevated adrenocorticotropic hormone level. The most common cause of primary adrenal insufficiency is autoimmune
destruction of all layers of the adrenal cortex, which leads to progressive mineralocorticoid, glucocorticoid, and adrenal
androgen deficiency. Most patients have positive 21-hydroxylase antibodies, and approximately 50% develop another
autoimmune endocrine disorder in their lifetimes, such as celiac disease, thyroid disease, or type diabetes mellitus.
Given that this patient has primary adrenal insufficiency, she requires both glucocorticoid and mineralocorticoid therapy. The
preferred glucocorticoid for the treatment of adrenal insufficiency is hydrocortisone two or three times daily to better mimic the
circadian rhythm of endogenous cortisol secretion. The higher dose of hydrocortisone is given in the morning (typically 10-15
mg), and the lower dose is given in the afternoon (approximately 5 mg).
References
Objective number 33(Fatigue) of Objectives for the Qualifying Examination, 3rd edition. Bornstein SR, Allolio B, Arlt W, et al.
Diagnosis and Treatment of Primary Adrenal Insufficiency: An Endocrine Society Clinical Practice Guideline. J Clin Endocrinol
Metab. 2016 Feb;101(2):364-89. Nieman LK. Diagnosis of adrenal insufficiency in adults. Post TW, ed. UpToDate. Waltham, MA:
UpToDate Inc. Accessed October 16, 2019.
Question #22
A 32-year-old primigravida at 10 weeks gestation comes to the office for her first prenatal visit. She has had occasional nausea
and vomiting but otherwise feels fine. The patient has hypothyroidism that has been managed with a stable dose of levothyroxine
for the last 3 years. Vital signs are normal.
Which of the following is the best next step in the management of this patient?
A) Decrease the dose of levothyroxine and repeat thyroid function test in 4 weeks
B) Measure free T4 to adjust the dose of levothyroxine
C) Measure T3 levels to adjust the dose of levothyroxine
D) Increase the dose of levothyroxine and repeat thyroid function test in 4 weeks
E) Repeat thyroid function test in 4 weeks on the current dose of levothyroxine
Explanation
Answer Description
Given that T4 dose requirements may increase during pregnancy in women with preexisting hypothyroidism, hypothyroid women
who are newly pregnant should preemptively increase their levothyroxine dose by approximately 30 percent.
Free T4 levels may be monitored in pregnancy but can be unreliable due to assay-specific artifact and lack of pregnancy
reference ranges. Due to elevated (undertreated) TSH, this patient requires an increased dose of levothyroxine, regardless of
free T4 levels.
Total serum T3 is elevated in pregnancy due to increases in thyroxine-binding globulin; in contrast, free T3 is not increased
proportionally. Evaluating serum T3 is unnecessary in most patients with hypothyroidism, including those who are pregnant.
Dose adjustments in pregnancy are based primarily on TSH.
Summarized Points:
Pregnant patients with hypothyroidism require an increased dose of levothyroxine as soon as they become pregnant due to their
bodies' inability to increase thyroxine appropriately. The dose of levothyroxine is subsequently adjusted based on TSH levels
using pregnancy-specific norms (or maintenance of TSH between the lower reference limit of normal and <2.5 mIU/L).
More Detail
Pregnant women require more thyroxine (i.e., T4), beginning as early as 5 weeks gestation, for the increased metabolic
demands of pregnancy. This demand is met by 2 physiologic mechanisms:
1. Estrogen increasing thyroxine-binding globulin levels
2. β-hCG stimulating TSH receptors (due to the shared a-subunit of hCG and TSH)
These physiologic changes result in a decreased TSH and an increased total T4 level (and a minimally increased free T4 level),
thereby meeting the metabolic demands of pregnancy. However, in pregnant patients with pre-existing hypothyroidism, the
body is unable to increase T4 appropriately, making them at risk for worsening hypothyroidism and adverse fetal effects
(e.g., cognitive impairment). Therefore, patients with hypothyroidism who are on a stable dose of levothyroxine should have the
dose increased by approximately 30% as soon as they are pregnant. The dose is adjusted based on TSH levels (typically in
4-week increments) using pregnancy-specific norms (or maintenance of TSH between the lower reference limit of normal and 2.5
mIU/L). Total T4 may also be followed and maintained within pregnancy-specific norms (i.e., 1.5x prepregnancy levels). Most
patients with hypothyroidism require up to a 50% dose increase during pregnancy.
References
Objective number 74 [Periodic Health Examination (PHE)] of Objectives for the Qualifying Examination, 3rd edition. Lemieux P,
Yamamoto JM, Nerenberg KA, Metcalfe A, Chin A, Khurana R, Donovan LE. Thyroid Laboratory Testing and Management in
Women on Thyroid Replacement Before Pregnancy and Associated Pregnancy Outcomes. Thyroid. 2021 May;31(5):841-849.